You are on page 1of 378

Open Rubric

©  2020 University of South Africa

All rights reserved

Printed and published by the


University of South Africa
Muckleneuk, Pretoria

GEE2601/1/2021–2023

10001042

MSWord

Images from Shutterstock


CONTENTS

STUDY UNITPage
1 INTRODUCTION1
1.1 What is Geotechnical Engineering III all about?1
1.2 Module contents4
1.3 Prescribed book7
1.4 Calculator requirements8
1.5 What is expected from you8
1.6 Self-assessment activities11
1.7 Solutions to self-assessment activities11
Reference works and further reading12

2 REVISION OF GTE260113
2.1 Purpose of study unit 13
2.2 Self-assessment activities 14

3 SOIL COMPACTION17
3.1 Introduction17
3.2 Definitions17
3.3 Learning outcomes18
3.4 General principles of compaction19
3.5 Factors affecting compaction22
3.6 Structure of compacted cohesive soil24
3.7 Field compaction25
3.8 Examples with solutions29
3.9 Self-assessment activities33
3.10 Solutions to self-assessment activities38
3.11 Summary39
3.12 Video clips, slides and other web-based resources39
Reference works and further reading40

4 MOVEMENT OF WATER THROUGH SOIL41


4.1 Introduction41
4.2 Definitions42
4.3 Learning outcomes42
4.4 Occurrence and effects of water in soil43
4.5 Bernoulli's Equation43
4.6 Darcy's Law46
4.7 Laboratory determination of hydraulic conductivity50
...........
iii G EE 2 6 01/1
CO N T EN T S

4.8 Empirical relations for hydraulic conductivity59


4.9 Field determination of hydraulic conductivity by pumping
from wells60
4.10 Flow nets67
4.11 Confined flow nets71
4.12 Unconfined flow nets79
4.13 Flow nets for anisotropic flow conditions85
4.14 Requirements of hydraulic filters87
4.15 Self-assessment activities88
4.16 Solutions to self-assessment activities96
4.17 Summary98
4.18 Video clips, slides and other web-based resources98
Reference works and further reading99

5 STRESSES IN A SOIL MASS101


5.1 Introduction101
5.2 Definitions101
5.3 Learning outcomes102
5.4 Principle of effective stress103
5.5 Stresses in soil with seepage105
5.6 Stress increase caused by surface loading110
5.7 Self-assessment activities126
5.8 Solutions to self-assessment activities132
5.9 Summary133
4.10 Video clips, slides and other web-based resources133
Reference works and further reading134

6 CONSOLIDATION SETTLEMENT135
6.1 Introduction135
6.2 Definitions136
6.3 Learning outcomes136
6.4 Fundamentals of consolidation137
6.5 Laboratory consolidation test139
6.6 Void ratio pressure plots140
6.7 Primary consolidation settlement148
6.8 Secondary consolidation152
6.9 Time rate of primary consolidation155
6.10 Self-assessment activities160
6.11 Solutions to self-assessment activities168
6.12 Summary170
6.13 Video clips, slides and other web-based resources170
Reference works and further reading171

...........
iv
Co nte nt s

7 SHEAR STRENGTH OF SOIL173


7.1 Introduction173
7.2 Definitions174
7.3 Learning outcomes174
7.4 Mohr-Coulomb failure criterion175
7.5 Factors determining shear strength177
7.6 Pore water pressure changes during shear178
7.7 Direct shear test183
7.8 Triaxial shear tests190
7.9 Self-assessment activities201
7.10 Solutions to self-assessment activities209
7.11 Summary210
7.12 Video clips, slides and other web-based resources211
Reference works and further reading212

8 SUBSURFACE EXPLORATION213
8.1 Introduction213
8.2 Learning outcomes214
8.3 Self-assessment activities216
8.4 Solutions to self-assessment activities223
8.5 Summary224
8.6 Video clips, slides and other web-based resources225
Reference works and further reading225

9 LATERAL EARTH PRESSURE227


9.1 Purpose of study unit227
9.2 Definitions228
9.3 Learning outcomes229
9.4 Earth pressure at rest230
9.5 Rankine's Theory of Active and passive earth pressure 233
9.6 Coulomb's Earth Pressure Theory246
9.7 Self-assessment activities251
9.8 Solutions to self-assessment activities259
9.9 Summary261
9.10 Video clips, slides and other web-based resources261
Reference works and further reading262

10 SLOPE STABILITY263
10.1 Introduction263
10.2 Definitions264
10.3 Learning outcomes265
10.4 Factor of safety265
10.5 Infinite slopes without seepage 266
10.6 Infinite slopes with seepage267

...........
v G EE 2 6 01/1
CO N T EN T S

10.7 Finite slopes with circularly cylindrical


failure surfaces270
10.8 Method of slices276
10.9 Self-assessment activities286
10.10 Solutions to self-assessment activities292
10.11 Summary293
10.12 Video clips, slides and other web-based resources293
Reference works and further reading294

11 SHALLOW FOUNDATIONS295
11.1 Introduction295
11.2 Definitions296
11.3 Learning outcomes296
11.4 Ultimate bearing capacity of foundations297
11.5 Effect of total or partial submergence 304
11.6 Factor of safety and allowable bearing capacity307
11.7 Eccentrically loaded continous foundations310
11.8 Self-assessment activities313
11.9 Solutions to self-assessment activities319
11.10 Summary321
11.11 Video clips, slides and other web-based resources321
Reference works and further reading322

12 RETAINING WALLS323
12.1 Introduction323
12.2 Definitions323
12.3 Learning outcomes324
12.4 Types of retaining walls324
12.5 Application of lateral earth pressure theories to retaining walls327
12.6 Modes of failure of a retaining wall328
12.7 Stability analysis328
12.8 Designing for undrained conditions335
12.9 Stability of embedded retaining walls336
12.10 Self-assessment activities346
12.11 Solutions to self-assessment activities350
12.12 Summary351
12.13 Video clips, slides and other web-based resources352
Reference works and further reading352

13 PILE (DEEP) FOUNDATIONS353


13.1 Introduction353
13.2 Definitions354
13.3 Learning outcomes354

...........
vi
Co nte nt s

13.4 Types of piles, pile characteristics, pile installation, load


transfer mechanism355
13.5 Estimation of pile capacity 356
13.6 Pile load tests367
13.7 Self-assessment activities367
13.8 Solutions to self-assessment activities368
13.9 Summary369
13.10 Video clips, slides and other web-based resources369
Reference works and further reading369

...........
vii G EE 2 6 01/1
...........
viii
Study unit 1 1

INTRODUCTION

1.1 WHAT IS GEOTECHNICAL ENGINEERING ALL ABOUT?

FIGURE 1.1
Geotechnical engineers and soil technicians drilling for detection of magnetic
anomalies with borehole detection (Shutterstock 2018)

1 GEE2601/1
1.1.1 Introduction

All structures (buildings, bridges, towers, etc.) and all civil engineering works are
founded on soil or rock formations. In this module, we use principles of soil and rock
mechanics to investigate subsurface conditions and materials. The purpose of this
module is therefore to enable you to identify and explain theories and models that
describe the behaviour of soil masses in different configurations subject to a variety of
loads.

For each of the configurations, you will learn to apply mathematical models to estimate
the stresses, strains and deformations in soil masses. Furthermore, you will develop
skills to plan and perform thorough site investigations and subsurface exploration for
a construction site. Lastly, you will be able to describe, perform and report laboratory
and in situ testing techniques and sampling of representative soils.

1.1.2 Soil mechanics

This module covers only the mechanics of soil masses. Rock mechanics and rock
engineering are only covered at an advanced level Geotechnical Engineering course.

Soil mechanics is defined as the branch of science concerned with the properties and
behaviour of soil as they affect its use in civil engineering. In the past decades, soil
mechanics became a unique and separate branch of geotechnical engineering mainly
because soils have a number of special properties which differentiate them from other
construction and building materials. The development of soil mechanics has also been
stimulated by the wide range of applications of soil mechanics in civil engineering and
construction, as all structures and civil engineering works require a sound foundation
to transfer loads to the soil.

In this module you will study theories and models that describe the behaviour of soil
masses in different configurations and subject to a variety of loads. For example, soil
masses with near-horizontal surfaces may be capable of supporting a variety of
foundation loads at or near the surface, or at depth, whereas soil masses with inclined
surfaces may be barely strong enough to support their own weight. If the surface

2 GEE2601/1
becomes very steep, a retaining structure may be required to support the soil mass.
For each of these configurations, there are mathematical models that strive to estimate
the stresses and deformations in the mass.

1.1.3 Site investigation and subsurface exploration

To familiarise themselves with the structural and general geology and soil conditions
of a future construction site, the geotechnical engineer and technician have to plan
and perform a thorough site investigation and subsurface exploration. During the site
exploration certain soil mass properties are determined by means of in situ testing
methods. In addition, sampling is done of representative soils that are to be taken to
a civil engineering materials laboratory for determining their engineering properties.

One entire study unit of this module will be dedicated to site investigation and
subsurface exploration. Some of the material covered will rely on the knowledge that
you acquired in the Introductory Geotechnical Engineering course.

1.1.4 Laboratory determination of soil properties

The geotechnical engineer and technician must have knowledge of the various
engineering properties of soils and soil masses that are to be used as inputs to various
models and theories to be used during a relevant design process. While some
properties may be estimated during in situ soil testing, the geotechnical engineer and
technician make extensive use of laboratory tests for accurately determining
engineering soil properties.

You learnt about some laboratory tests, mainly for soil classification purposes, in the
Introductory Geotechnical Engineering course. In this module we will discuss a
number of additional tests that have been devised to determine specific engineering
properties as part of the different study units on engineering theories and models. You
will be required to perform and report most of these experiments in a civil engineering
materials laboratory, be able to describe the test procedures and equipment and
interpret the results.

3 GEE2601/1
You will perform the following tests as part of the associated practical module:

• permeability of a soil: constant-head and falling-head methods


• direct shear test (shear box test)
• consolidation: oedometer test

The tutorial letter contains more information on laboratory work.

1.2 MODULE CONTENTS

1.2.1 Compaction

In study unit 3, we will cover the following topics:

• general principles of compaction


• factors affecting compaction
• structure of compacted cohesive soil
• field compaction
• determination of unit weight

1.2.2 Movement of water through soil

In study unit 4, we will cover the following topics:

• occurrence and effects of water in soil


• Bernoulli's equation
• Darcy's law
• laboratory determination of hydraulic conductivity
• empirical relations for hydraulic conductivity
• field determination of hydraulic conductivity
• flow nets for confined and unconfined flow conditions
• requirements of hydraulic filters

4 GEE2601/1
1.2.3 Stresses in a soil mass

In study unit 5, we will cover the following topics:

• principle of effective stress


• stresses in soil with seepage
• stress increase by surface loading

1.2.4 Consolidation settlement

In study unit 6, we will cover the following topics:

• fundamentals of consolidation
• laboratory consolidation test
• void ratio-pressure plots
• primary consolidation settlement calculations
• secondary consolidation
• time rate of primary consolidation

1.2.5 Shear strength

In study unit 7, we will cover the following topics:

• Mohr-Coulomb failure criterion


• factors determining shear strength
• pore water pressure changes during shear
• direct shear test
• tri-axial shear tests
• in situ measurement of shear strength

1.2.6 Subsurface exploration

In study unit 8, we will cover the following topics:

• subsurface exploration programme


• exploratory borings

5 GEE2601/1
• procedures for sampling
• observation of water levels
• vane shear test
• cone penetration test
• pressure meter test
• dilatometer test
• coring of rocks
• preparation of boring logs
• soil exploration report

1.2.7 Lateral earth pressure

In study unit 9, we will cover the following topics:

• earth pressure at rest


• Rankine's theory of active and passive earth pressure
• Coulomb's earth pressure theory

1.2.8 Slope stability

In study unit 10, we will cover the following topics:

• factor of safety
• infinite slopes
• finite slopes with circularly cylindrical failure surfaces
• method of slices

1.2.9 Shallow foundations

In study unit 11, we will cover the following topics:

• ultimate bearing capacity of foundations


• effect of total or partial submergence
• factor of safety and allowable bearing capacity
• eccentrically loaded foundations

6 GEE2601/1
• foundation settlement on saturated clay
• settlement and allowable bearing pressure on sand

1.2.10 Retaining walls

In study unit 12, we will cover the following topics:

• types of retaining walls


• application of lateral earth pressure theories to retaining walls
• modes of failure of a retaining wall
• stability analysis
• drainage from backfill
• designing for undrained conditions
• stability of embedded retaining walls

1.2.11 Deep foundations

In study unit 13, we will cover the following topics:

• types of piles
• pile characteristics
• pile installation
• load transfer mechanism
• estimation of pile capacity
• pile load tests

1.3 PRESCRIBED BOOK

The textbook is as follows:

• Das, BM & Sivakugan, N. 2017. Fundamentals of geotechnical engineering. 5th


ed., international edition. Connecticut: Cengage Learning. ISBN 13: 978-1-305-
63862-4.

7 GEE2601/1
• Das, BM & Sivakugan, N. 2017. Fundamentals of geotechnical engineering:
solutions manual. 5th ed., international edition. Connecticut: Cengage Learning.
ISBN 13: 978-1-305-63862-4.

In addition to the prescribed book, the following book is also recommended:

• Das, BM & Luo, Z. 2017. Principles of soil dynamics. 3rd ed., international edition.
Connecticut: Cengage Learning. ISBN 13: 978-1-305-38944-1.

Note that prescribed and recommended books are always in a continuous process of
refinement and redevelopment by the authors. As such, prescribed books may change
to the latest versions. For example, the prescribed book for this module is currently
the 5th edition and in a few years that may change to the 6th or 7th edition. The examples
and exercises may soon refer to the latest versions of the prescribed book. As such
changes occur, updates will be made to the online version of this study guide on the
myUnisa platform. Continuously consult the online platforms (myUnisa, discussion
forums and e-tutor blog) to keep up with changes and any other new additional study
material (videos, tutorials, memorandums, exercises and study notes) for this module.
You may also use earlier versions of the prescribed book if you are unable to obtain
the version listed as the prescribed book. However, the references, page numbers and
exercises will differ.

1.4 CALCULATOR REQUIREMENTS

You need a scientific calculator. Programmable calculators are not allowed.

1.5 WHAT IS EXPECTED FROM YOU

We expect the following from you as a third-year university student:

• Be dedicated and work hard.

8 GEE2601/1
• Manage your time and schedule effectively and make sure that you complete and
submit all assignments on time (see section 1.5.2 for a suggested study schedule
that you can use as a guideline to study this module).
• Be responsible for your own learning and progress. If you are struggling to
understand a particular aspect, contact your lecturer or tutor.
• Do the following activities in your own time:
 Read the study guide and make sure you understand the content.
 Read the prescribed and recommended textbooks.
 Do some wider general reading on geotechnical engineering.
 Watch video clips (web addresses are provided under Additional
Resources on the myUnisa module site).
 Regularly check for updates and additional resources on myUnisa.
 Work on self-assessment activities, previous exam papers and assignments.
 Participate in discussion forums with other students on myUnisa.

1.5.1 Examples and self-assessment activities

You will note that there are examples and solutions in study units 2-13. You will also
find self-assessment activities at the end of study units 1-13. Please attempt each of
these examples and activities before looking at the solutions provided. It is very
important to solve these examples and problems on your own, as you might come
across similar problems in the assignments and exam.

1.5.2 Planning and managing your time

Because of your unique circumstances, each of you will plan and manage your study
programme during the year differently. You may find the following tips helpful for
planning and managing your time:

• Start by skimming through the study material to get an idea of what is covered in
the module. Use this information to divide the material into manageable sections,
and schedule times when you can study them. You may need to revise the

9 GEE2601/1
schedule as you delve deeper into the study material. Also schedule times for
doing assignments and for exam revision. Students in a contact setting are forced
to interact with study material during lectures. However, as you do not attend
lectures, in order to be successful you need to schedule times for interacting with
the study material. Make notes and summaries while you are studying and use
these when you do assignments or prepare for the exam.
• Consult the Study @ Unisa brochure for suggestions on general time management
and planning skills.
• GEE2601 is a semester module offered over 15 weeks, and it requires at least
120 hours of study time. This means that you have to study at least 8 hours per
week for this module.

Here is a suggested schedule that you could use as a guideline for studying this
module:

ACTIVITY HOURS
Reading and rereading Tutorial Letter 101 and study unit 1 1
Review the Geology and Soil Mechanics (GSM1501) module 3
Skimming through the study units and textbooks to get an
3
overview of the whole module
First reading of study units 2-13 and the textbook 10
In-depth study of study units 2-13: study the content, work through
the examples and do the self-assessment activities presented at 50
the end of each study unit
Completing three assignments 15
Preparing and performing laboratory tests 10
Completing the practicum and required portfolio 5
Revising for the examination 20
Writing the examination 3
Total 120

10 GEE2601/1
1.6 SELF-ASSESSMENT ACTIVITIES

You now have the opportunity to apply what you have learnt in this unit. Answer the
following questions:

Activity 1.1
What is geotechnical engineering and why is it important?

Activity 1.2
What are the functions of geotechnical engineering?

Activity 1.3
Do further wider reading on geotechnical engineering in general to broaden your
general understanding of this module.

Activity 1.4
Do further reading on the geotechnical engineering aspects of the following famous
engineering projects:

• Panama Canal

• Golden Gate Bridge foundations

• Petronas Towers Foundations

1.7 SOLUTIONS TO SELF-ASSESSMENT ACTIVITIES

Activity 1.1

Answer: Geotechnical engineering is a branch in civil engineering concerned with the


engineering behaviour of soils. Geotechnical engineering is very important in civil
engineering and also has applications in other engineering disciplines that are related
to building and construction. The field of geotechnical engineering makes use of rock
engineering and soil mechanics principles to:
• assess surface and subsurface in situ soil conditions

11 GEE2601/1
• determine the relevant engineering properties of these soils

• assess the stability of slopes and soil deposits

• determine and assess risks posed by site conditions

• design earthworks, structure foundations and civil engineering works

• monitor construction site conditions, earthworks and foundation construction


works

Activity 1.2

Answer: The major functions of geotechnical engineering are as follows:


• evaluation of geotechnical hazards, including the potential for landslides
• determination of bearing capacity, deformations of foundations and likely inter-
actions between soil, foundation and the structure
• assessment of earth pressure and the performance of retaining walls
• analysis of embankment behaviour
• determination of strength of excavations and tunnels
• conducting of response analysis for construction site

REFERENCE WORKS AND FURTHER READING

Terzaghi, K, Peck, RB & Mesri, G. 1996. Soil mechanics in engineering practice. 3rd
ed. New York: John Wiley & Sons

Holtz, R & Kovacs, W. 1981. An introduction to geotechnical engineering. Englewood


Cliffs: Prentice-Hall.

Verruijt, A. 2001. Soil mechanics. Delft: Delft University of Technology.

12 GEE2601/1
Study unit 1 2

REVISION OF GSM1501

2.1 PURPOSE OF STUDY UNIT

This study unit is a revision of chapters 2, 3 and 4 of the prescribed book. Read these
chapters and complete the exercises in section 2.2 of this study unit. We revise the
material covered in Geology and Soil Mechanics (GSM1501). The answers to the
theoretical exercise questions can be obtained from chapters 2, 3 and 4 of the
prescribed book. It is important and beneficial for you to make time to revise GSM1501
because the material covered there will help you to have a thorough grounding for
GEE2601 and it serves as a building block to complete the exercises and to be able
to solve the more complex problems in GEE2601.

For additional explanations of important concepts in soil mechanics and geology, you
can watch some video clips. Web addresses/links to the video clips are also available
on the module site under Additional Resources in a folder named Video clips,
slides and other web-based resources.

• Phase diagram and basic definitions | soil mechanics


https://www.youtube.com/results?search_query=soil+mechanics
• CEEN 341 - Lecture 1 - origin of rocks and soil:
https://www.youtube.com/watch?v=_arD9SDTK74&list=PLzBZ3hmMnx1KUOu8
ZQItF7J2Stdo0tjhG
• Hydrometer method - particle size analysis | soil mechanics:
https://www.youtube.com/watch?v=gQhBEjCdYvs

13 GEE2601/1
2.2 SELF-ASSESSMENT ACTIVITIES

Activity 2.1

With the aid of a sketch, briefly illustrate the rock cycle.

Activity 2.2

Briefly describe the weathering process.

Activity 2.3

Briefly discuss the hydrometer analysis and explain Stoke's law.

Activity 2.4

What are the three basic soil parameters that can be determined using the grain-size
distribution curve?

Activity 2.5

In detail, discuss the classification of rocks and give examples.

Activity 2.6

What five parameters are included in the weight-volume relationship of soils?

Activity 2.7

How is soil consistency identified?

Activity 2.8

Define the following terms:

• plastic limit (PL)


• shrinkage limit (SL)
• liquid limit (LL)

14 GEE2601/1
Activity 2.9

Briefly describe the AASHTO soil classification system and give examples.

Activity 2.10

What information does a geotechnical engineer require in order to properly classify a


soil sample?

15 GEE2601/1
Notes

....................................................................................................................................

....................................................................................................................................

....................................................................................................................................

....................................................................................................................................

....................................................................................................................................

....................................................................................................................................

....................................................................................................................................

....................................................................................................................................

....................................................................................................................................

....................................................................................................................................

....................................................................................................................................

....................................................................................................................................

....................................................................................................................................

....................................................................................................................................

....................................................................................................................................

....................................................................................................................................

....................................................................................................................................

....................................................................................................................................

....................................................................................................................................

....................................................................................................................................

....................................................................................................................................

....................................................................................................................................

16 GEE2601/1
Study unit 1 3

SOIL COMPACTION

3.1 INTRODUCTION

In civil engineering construction works (roadworks, dams and bridge construction,


etc.) loose soils have to be compacted to increase their unit weights. By compacting
soils, the strength characteristics and bearing capacities are increased. This reduces
undesired settlements and improves stability. In addition, resistance to compression
and shear stresses and to water seeping through the pores is increased. Compaction
is therefore defined as the densification of soil by removing air and rearranging soil
particles close to each other through the use of mechanical energy (Das 2010).

The geotechnical engineer and technician will experience compaction and


compacted soils during construction works, including road pavements, dam, building
foundations and earth fills behind retaining walls, and it is important for them to know
more about the processes involved in compaction, the controlling factors and the
effects on the material being compacted. This study unit is covered in chapter 5 (pp.
104-134*) of the prescribed book.

3.2 DEFINITIONS

You will come across the following terms in this study unit:

• Compaction: The densification of soil by removing air and rearranging soil


particles close to each other through the use of mechanical energy.

17 GEE2601/1
• Maximum dry density: The highest density that is obtainable through a
laboratory compaction test when the compaction is carried out on a soil at varied
moisture contents.
• Optimum moisture content: The moisture content at which the maximum dry
density is achieved.
• Soil: A random mixture of grains, which may have a wide range of size, shape
and plasticity properties.

3.3 LEARNING OUTCOMES

On completion of this study unit, you should be able to explain and apply the
fundamental principles of compaction and methods of increasing the density of soil
masses. More specifically, you should be able to

• explain the general principles of compaction


• calculate and interpret the results of a compaction test
• identify and describe the various factors that determine the compaction
characteristics of soil, as well as the effects that compaction conditions have on
the soil structure
• identify and explain the standard methods that are used to determine the
compaction characteristics of soil
• distinguish and select compaction equipment that should perform well in specific
situations
• determine the maximum dry unit weight of compaction and the optimum moisture
content

To achieve the learning outcomes in this unit, work through the following topics:

• general principles of compaction


• factors affecting compaction
• the structure of compacted soils
• standard methods used to determine the compaction characteristics of soil

18 GEE2601/1
• field compaction and specifications for field compaction
• field compaction equipment and techniques

3.4 GENERAL PRINCIPLES OF COMPACTION

This topic is covered in detail in chapter 5 (pp. 104-134*) in the prescribed book.
Read through section 5.2 of the prescribed book. Compaction is any process in
which mechanical energy is applied to soil to remove air from the voids, with the result
that the mineral grains move closer together. In this way more mineral grains can be
forced into a given volume, and the dry density, or dry unit weight, of the soil mass is
increased.

There are five main reasons for compacting soil masses:


• to increase load-bearing capacity
• to prevent soil settlement and frost damage
• to provide stability
• to reduce water seepage, swelling and contraction
• to reduce settling of soil

Important facts
(a) Dry density or unit weight is used as a means to evaluate the efficiency of
compaction, not bulk density.
(b) When water is added to the soil during the process of compaction, it acts as
lubrication between soil particles. This enables the particles to slip over each
other into a densely packed position. It also helps the clay minerals to act as
binding medium for soil grains.
(c) The moisture content of the soil is a very important factor that controls the dry
unit weight after compaction. There is a curved relationship between dry unit
weight and compaction moisture content as shown in figure 5.5 (p. 109*) in
the prescribed book.
(d) At a moisture content of zero, the moist unit weight is equal to the dry unit
weight.

19 GEE2601/1
(e) The maximum dry unit weight is achieved at the optimum moisture content.
(f) The zero-air-void curve shown in figure 5.4 (p. 108*) represents the absolute
maximum dry unit weight that soil may be compacted to at any given value of
the moisture content, and that is when absolutely all the air has been driven
from the voids and the soil is saturated as a result. This dry unit weight is
impossible to achieve in practice. Its exact position on the figure is a function
of the specific gravity (Gs) and can be determined by using equation 5.3 (p.
108*) in section 5.3 in the prescribed textbook.

The equation for the zero-air-void curve, equation 5.3 in the prescribed book, can be
derived from the basic phase relationships you studied in Geotechnical Engineering
II (GTE2601):

Gs γ w Gs γ w
γd = = (3.1)
1+ e wG s
1+
Sr
Where:
• γd = Dry unit weight
• γw = Unit weight of water
• Gs = Specific gravity of soil solids
• e = Void ratio
• Sr = Degree of saturation

For 100% saturation, e = wGs. With all air forced out, in the zero-air-void condition,
the soil is saturated and so Sr = 1. The equation then becomes

Gs γ w Gs γ w γw
γ zav = = = (3.2)
1 + e 1 + wG s 1
w+
Gs
Where:

• γzav = Zero-air-void dry unit weight

Equation 3.1 can be used to draw curves of dry unit weight at any chosen constant
value of the degree of saturation, Sr. These curves are parallel to the zero-air-void

20 GEE2601/1
curve and are shifted to the left on the figure. The procedure that is applicable to
achieve the theoretical

γzav is as follows:

• Determine the specific gravity of soils.


• Know the unit weight of water (γw).
• Assume several values of w, for example 5%,10%, 15%, 20% and so on.
• Use equation 3.2 (equation 5.3 in the prescribed book) to calculate for various
values of w.

Example 3.1

Carefully work through example 5.1 in chapter 5 (pp. 113-114*) in the prescribed
book to become familiar with the calculations and interpretation of the results of a
compaction test. Try to add the zero-air-void curve to the figure by calculating the dry
unit weight of the soil at moisture contents of 16, 18, 20 and 22%. Assume that
Gs = 2,65 for this purpose.

Solution γw 10kN / m 3
γ zav = = = 18.61kN / m 3
1  16  1
w+
• For 16% moisture content:
Gs 100  + 2.65

γw 10kN / m 3
γ zav = = = 17.94kN / m 3
• For 18% moisture content: 1  18  1
w+ 100  + 2.65
Gs  

γw 10kN / m 3
γ zav = = = 17.32kN / m 3
• For 20% moisture content: 1  20  1
w+ 100  + 2.65
Gs  

γw 10kN / m 3
γ zav = = = 16.74kN / m 3
• For 22% moisture content: 1  22  1
w+ 100  + 2.65
Gs  

21 GEE2601/1
Plot the values of γzav on the graph that you have drawn after working on example 5.1
in the prescribed book.

3.5 FACTORS AFFECTING COMPACTION

Read through section 5.4 (pp. 109-110*) in the prescribed book. There are a few
important factors that determine the dry density achieved by compaction. In the
preceding section we discussed the effect of moisture content, and in this section we
will look at the effects of soil type and compaction effort.

3.5.1 Effect of soil type

No two soils behave the same during compaction and a unique relationship will be
obtained between dry density and moisture content for each soil. The soil type
(distribution of grain size, soil grain shape, specific gravity and the clay mineral type)
has a considerable impact on the optimum moisture content and the maximum dry
density. Figure 5.5 (p. 109*) in the prescribed book shows the relationships, called
compaction curves, for four soil types (A, B, C and D). You must understand, though,
that soil is a random mixture of grains, which may have a wide range of size, shape
and plasticity properties. This means that the compaction curve for a given soil can
only be determined by performing standardised laboratory compaction tests.

3.5.2 Effect of compaction energy

A standard way of compacting soils must always be used in order to be able to plot
and properly evaluate the compaction curve, compare it with those obtained for other
soils and use it in contract documents etc. Laboratory compaction is usually
performed using a hammer and mould, like the ones in figure 5.2 (p. 106*), and the
soil is compacted in layers. It is primarily the amount of mechanical energy used per
unit volume of soil that determines the dry density achieved on a given soil at any
given moisture content. This is called the compaction effort and it must have a
standard value. The effect of compaction effort on the compaction curve of sandy
clay is illustrated in figure 5.6 (p. 111*) in the prescribed book.

22 GEE2601/1
The compaction energy per unit volume (kN.m/m3) as shown in equation 5.7 (p. 110*)
in the prescribed book is given as

 Number of   Number   Weight  Height 


blows per  × of  × of  × of drop of 
       
layer  layers  hammer hammer 
E= (3.3)
Volume of mould

A standardised laboratory compaction method is the standard Proctor test, described


in section 5.3 (pp. 105-109*) in the prescribed book. To keep up with the development
of heavy rollers, the modified Proctor test, also called the modified AASHTO test, was
devised. Its compaction effort is 4,56 times that of the standard Proctor test and you
will find a detailed description of the method in section 5.5 (pp. 111-115*) in the
prescribed book.

Note that in South African practice these tests have been modified slightly and are
called the Proctor method and the modified AASHTO method, respectively. The tests
are described together with the NRB method as Method A7 in the TMH1 manual
published by the Council for Scientific and Industrial Research (CSIR) on behalf of
the South African Committee of State Road Authorities (CSRA).

Important facts

(a) The maximum dry unit weight of a given soil increases if the compaction effort
is increased, but not proportionally.
(b) The optimum moisture content decreases somewhat if the compaction effort
is increased.

23 GEE2601/1
3.6 STRUCTURE OF COMPACTED COHESIVE SOIL

The effect that compaction has on soil with an appreciable amount of clay minerals
is more than just an increase in dry density, but conditions during compaction also
influence the clay’s structure, as you can see in figure 3.1 below. Almost all of clay’s
engineering properties depend to some extent on its structure. In addition to moisture
content, the action of the compaction equipment has a marked influence on structure.
Kneading compaction results in a more dispersed structure, whereas tamping
compaction results in a more flocculated structure.

FIGURE 3.1
Effect of compaction on structure of clay soils (Das 2010)

Important facts
(a) Compaction at moisture contents below optimum, using a tamping action and
low compaction effort, leads to a more flocculated structure, which has higher
strength, brittleness and hydraulic conductivity (see figure 3.1).

24 GEE2601/1
(b) Compaction at moisture contents above optimum, using a kneading action and
high effort, results in a more dispersed structure, which has lower strength,
brittleness and hydraulic conductivity (see figure 3.1).

3.7 FIELD COMPACTION

Read through section 5.7 (pp. 118-120*) in the prescribed book.

3.7.1 Compaction equipment

The material in section 5.7 of the prescribed book covers the equipment and
techniques that are used in the field to compact soils in civil engineering construction
works (earth fills, earth dams, roadworks, etc.) and the determination of the dry unit
weight achieved by compaction.

Most of the compaction that takes place in the field is done using the following
equipment, depending on the types of materials to be compacted and the stage of
construction:

• smooth-wheeled rollers
• pneumatic rollers
• sheepsfoot rollers
• falling weight impact (drop or impact weight)
• vibratory rollers
• pedestrian rollers

There are four types of compaction efforts on soil masses:

• vibration
• impact
• kneading
• pressure

It is clear that field compaction is a lot more complicated than the standard
compaction tests performed in the laboratory. The compaction effort is virtually

25 GEE2601/1
impossible to calculate exactly but is determined by the weight and physical
dimensions of the roller, lift thickness (e.g. layer thickness) and the number of passes.
Vibration of a roller may be beneficial to improve compaction in some cases.

A complicating factor is the variation in compaction achieved over the lift depth. Trial
runs in the field may be the only reliable way of determining the optimum combination
of roller, moisture content, number of passes and lift thickness, as shown in figures
5.10 to 5.12 (p. 119*) in the prescribed book.

You should be able to distinguish between the different types of rollers and the soil
types that each could be used on, but you don’t need to remember all the detailed
information on each roller. The following pictures show the different types of rollers:

FIGURE 3.2 FIGURE 3.3


Smooth-wheeled roller (Shutterstock 2018) Pneumatic roller (Shutterstock 2018)

FIGURE 3.4 FIGURE 3.5


Sheepsfoot roller (Shutterstock 2018) Vibratory roller (Shutterstock 2018)

26 GEE2601/1
FIGURE 3.6
Pedestrian roller (Shutterstock 2018)

3.7.2 Compaction specification

The material in section 5.8 (pp. 120-121*) in the prescribed book covers the
specifications for field compaction. In its simplest form a specification for field
compaction usually involves a compaction curve of the soil that has been determined
in the laboratory using standard compaction test methods (e.g. modified Proctor test).
Depending on what is to be constructed, the following items are specified:

• the moisture content at which compaction should be performed using standard


compaction test methods (e.g. modified Proctor test), taking into account the
effect this has on the soil structure and properties
• the dry unit weight that should be achieved, usually as a percentage (the relative
compaction (R)) of the maximum value determined from the laboratory
compaction test

The relative compaction, as shown in equation 5.23 (p. 120*) in the prescribed book,
is given by

γ d ( field ) (3.4)
R (% ) = × 100
γ d (max −lab )

27 GEE2601/1
In addition to these specifications, the type of roller, lift thickness, etc. may sometimes
be stipulated, usually after some trial runs in the field, if specific requirements are to
be met. Contractors are expected to achieve a minimum specified unit weight
regardless of the field procedure adopted. The most economical compaction
condition can be explained with the aid of figure 5.13 (p. 121*).

3.7.3 Determination of unit weight in the field

The unit weight of compacted soil may be measured in various ways, as described
in section 5.9 (p. 122*) in the prescribed book. With the exception of the nuclear
methods, the principle is to determine the mass of soil that occupies a certain volume,
as well as its moisture content, from which bulk and dry unit weights may then be
calculated. A hole is dug into the compacted layer, all the soil is carefully removed,
and the mass and moisture content are determined. The volume of the hole is then
measured by filling the hole with sand of known unit weight in what is known as the
sand cone method (sand replacement method), or with water in the rubber balloon
method. From the three values (volume, mass and moisture content) the bulk and
dry unit weights may be calculated.

The dry weight of the soil can be calculated using the following formula:

W2
W3 =
w (%) (3.5)
1+
100
Where:

• W2 = Weight of the moist soil excavated


• W3 = Dry weight of the soil
• w = Moisture content

The calculation involved in determining the unit weight in the field is explained in
example 3.3.

28 GEE2601/1
The nuclear method uses different radioactive radiations to measure bulk unit weight
and moisture content directly. The test is relatively easy and fast, and the method
seems to be the preferred one in modern-day practice, although it is believed that the
sand cone method is more accurate. The rubber balloon method is not used in South
Africa. Figure 5.17 (p. 125*) in the prescribed book shows a picture of the nuclear
density meter. You should be familiar with the procedures of the nuclear and sand
replacement test and be able to calculate dry unit weight from the measurements of
the sand replacement test. The tests are designated Methods A10 (a) and (b) in the
TMH1 Manual, or SANS 3001.

3.8 EXAMPLES WITH SOLUTIONS

3.8.1 Example 3.2

The laboratory test results of a standard Proctor test in a road project in Liberia are
given in the table below. Determine the maximum dry unit weight of compaction and
the optimum moisture content (take note of the non-metric measurement system
which is used in Liberia).

Volume of mould Weight of moist soil in mould Moisture content, w


(ft3) (lb) (%)
3.63 10
3.86 12
4.02 14
3.333 x 10-2
3.98 16
3.88 18
3.78 20

29 GEE2601/1
Solution

The following table can be prepared:

Weight of
Moist unit Moisture
Volume of moist soil Dry unit weight
weight Ɣ content, w
mould (ft3) in mould (lb/ft3)
(lb/ft3) (%)
(lb)
3.63 108.9 10 99.0
3.86 115.8 12 103.4
4.02 120.6 14 105.8
3.333 x 10-2 3.98 119.4 16 102.9
3.88 116.4 18 98.6
3.78 111.9 20 93.3

Note the following: W and γ


Moist Unit Weight = Dry Unit Weight =
V  w (%) 
1+ 
Plot the dry unit weight vs the moisture content.  100 

FIGURE 3.7
Dry unit weight vs moisture content

30 GEE2601/1
3.8.2 Example 3.3

The laboratory compaction test results for a clayey-silt soil from a road project in
Giyani (Limpopo) are given in the table below:

TABLE 3.1: Laboratory tests for example 3.3

Dry unit weight (kN/m3) Moisture content, w (%)


14.80 6
17.45 8
18.52 9
18.9 11
18.5 12
16.9 14

Also, a field unit weight determination test was conducted on site using the sand cone
method. The results are as follows:

• Calibrated dry density of standard sand = 1 570 kg/m3


• Calibrated mass of standard sand to fill the cone = 0.545 kg
• Mass of jar + cone + sand (before use) = 7.590 kg
• Mass of jar + cone + sand (after use) = 4.780 kg
• Mass of moist soil from hole = 3.007 kg
• Moisture content of moist soil = 10.2%

Determine the dry unit weight of compaction in the field and the relative compaction
in the field.

Solution

In the field:

• Mass of sand to fill the hole and cone = 7.59 kg - 4.78 kg = 2.81 kg
• Mass of sand used to fill the hole = 2.81 kg – 0.545 kg = 2.265 kg
2.265 kg 2.265𝑘𝑘𝑘𝑘
• Volume of the hole = Dry density of Ottawa sand = 1570𝑘𝑘𝑘𝑘/𝑚𝑚3 = 0.0014426 𝑚𝑚3

31 GEE2601/1
Mass of moist soil 3.007𝑘𝑘𝑘𝑘
• Moist density of compacted soil = Volume of hole
= 0.0014426𝑚𝑚3 = 2084.4 𝑘𝑘𝑘𝑘/𝑚𝑚3
2084.4kg/m3 ×9.81
• Moist unit weight of compacted soil = 1000
= 20.45 𝑘𝑘𝑘𝑘/𝑚𝑚3
𝛾𝛾 20.45𝑘𝑘𝑘𝑘/𝑚𝑚3
• 𝛾𝛾𝑑𝑑 = 𝑤𝑤 (%) =
1+
10.2 = 18.56 𝑘𝑘𝑘𝑘/𝑚𝑚3
1+ 100
100

To determine the relative compaction in the field you will require to calculate the
laboratory maximum dry density.

Hence,
Plot the graph using the data given in Table 3.1

FIGURE 3.8
Dry unit weight vs moisture content

From the results and the graph above, the maximum dry density is 18.9 kN/m3,
rounded off to 19 kN/m3. From equation 3.4

γ d ( field ) 18.56kN / m 3
R (% ) = × 100 = = 97.7%
γ d (max −lab ) 19kN / m 3

32 GEE2601/1
3.9 SELF-ASSESSMENT ACTIVITIES

You now have the opportunity to apply what you have learnt in this unit. Answer the
following questions:

Activity 3.1: Prescribed book exercises

Do problems 5.1 to 5.17 (pp. 128-133*).

Activity 3.2: Multiple-choice questions

1. Compaction, in general, is …
(a) densification of soil by removal of water from the voids of the soil.
(b) densification of soil by removal of air from the voids of the soil.
(c) densification of soil by removal of both air and water from the voids of the
soil.
(d) the movement of rollers over the soil subgrade.

2. The degree of compaction of a soil is measured in terms of its …


(a) moisture content.
(b) bulk unit weight.
(c) dry unit weight.
(d) shear strength.

3. During compaction, with an increase in moisture content, the dry unit weight of
soil …
(a) always increases.
(b) always decreases.
(c) first increases, reaches a peak value and then decreases.
(d) remains unchanged.

33 GEE2601/1
4. The moisture content of soil at which the maximum dry unit weight is attained is
generally referred to as …
(a) optimum moisture content.
(b) maximum moisture content.
(c) minimum moisture content.
(d) None of the above.

5. The laboratory soil compaction test is used to obtain the …

(a) decrease in volume of the soil.


(b) maximum unit weight of the soil.
(c) optimum moisture content of the soil.
(d) Both (b) and (c).

6. The standard Proctor compaction mould has a volume of …

(a) 943.3 cm3.


(b) 1 000 cm3.
(c) 2 124 cm3.
(d) None of the above.

7. In the modified Proctor compaction test, the soil is compacted in layers.

(a) 3
(b) 5
(c) 7
(d) 9

8. In the standard Proctor compaction test, the mass of the hammer is

(a) 2.5 kg.


(b) 4.536 kg.

34 GEE2601/1
(c) 5 kg.
(d) 10 kg.

9. In the modified Proctor compaction test, the drop of the hammer is …


(a) 101.6 mm.
(b) 304.8 mm.
(c) 457.2 mm.
(d) None of the above.

10. The number of hammer blows for each soil layer in the standard Proctor
compaction test is kept at …
(a) 25.
(b) 50.
(c) 75.
(d) 125.

11. The compaction energy per unit volume of soil used for the standard Proctor
compaction test is …
(a) 591.3 N-m/m3.
(b) 591.3 kN-m/m3.
(c) 2 696 kN-m/m3.
(d) 2 696 N-m/m3.

12. A compaction curve is a plot of …


(a) bulk unit weight versus moisture content.
(b) dry unit weight versus moisture content.
(c) maximum dry unit weight versus optimum moisture content.
(d) None of the above.

35 GEE2601/1
13. If, for a compacted soil bulk, unit weight is 18.3 kN/m3, and moisture content is
12%, its dry unit weight will be …

(a) 16.3 kN/m3.


(b) 18.3 kN/m3
(c) 20.5 kN/m3.
(d) None of the above.

14. For a given moisture content, the maximum dry unit weight is obtained when
degree of saturation is …

(a) 0%.
(b) 25%.
(c) 50%.
(d) 100%.

15. If specific gravity of soil solids is 2.68, and the water content of soil is 10%, its
zero-air void unit weight will be …

(a) 3.7 kN/m3.


(b) 7.1 kN/m3.
(c) 20.7 kN/m3.
(d) None of the above.

Activity 3.3: Typical exam questions

1. A soil sample was taken from a borrow pit in a road construction project in
Majeje village in the Phalaborwa district. The soil sample has a dry unit weight
of 19.5 kN/m3, moisture content of 8% and the specific gravity of solids particles
is 2.67. Calculate the following:

(a) The void ratio


(b) Moisture content and saturated unit weight

36 GEE2601/1
(c) The mass of water to be added to cubic metres of soil to reach 80%
saturation
(d) The volume of solids particles when the mass of water is 25 g for saturation

2. A soil sample taken from a mining borrow pit in Welkom has an avoid ratio of
0.72, a moisture content of 12% and a specific gravity of 2.72. Determine the
following:

(a) Dry unit weight and the moist unit weight (kN/m3)
(b) Weight of water in kN/m3 to be added for 80% degree of saturation
(c) Is it possible to reach a water content of 30% without changing the present
void ratio?
(d) Is it possible to compact the soil sample to a dry unit weight of 23.5 kN/m3?

3. The results of a standard compaction test for a soil sample in Lulekani township
near Namakgale in Limpopo are shown in the table below (Gs = 2.5):

Water content (%) Unit weight (KN/m3)


6.2 16.9
8.1 18.7
9.8 19.5
11.5 20.5
12.3 20.4
13.2 20.1

Compute the following:

(a) The optimum water content


(b) The maximum dry unit weight
(c) The void ratio (e)
(d) Degree of saturation (S)
(e) Moisture unit weight

37 GEE2601/1
(f) The weight of water need to be added to 1 m3 to reach 100% degree of
saturation

3.10 SOLUTIONS TO SELF-ASSESSMENT ACTIVITIES

Activity 3.1: Prescribed book exercises


See prescribed solution book for answers.

Activity 3.2: Multiple-choice questions

1. (b)
2. (c)
3. (c)
4. (a)
5. (d)
6. (a)
7. (b)
8. (a)
9. (c)
10. (a)
11. (b)
12. (b)
13. (a)
15. (c)

Activity 3.3: Typical exam questions

1. (a) e = 0.343; (b) w = 12.85% & γsat = 22 kN/m3; (c) 44.85 kg; (d) vs = 72.84
cm3)
2. (a) 15.51 kN/m3 & 17.374 kN/m3; (b) 1.428 kN/m3; (c) Not possible; (d) Not
possible)
3. (a) 11.5%; (b) 18.4 kN/m3; (c) e = 0.334; (d) S = 86%; (e) 20.86 kN/m3; (f) 0.36
KN

38 GEE2601/1
3.11 SUMMARY

Having studied this study unit, you should be able to determine and evaluate the
compaction characteristics of soil in the laboratory and use these to specify and
measure the compaction achieved in the field.

3.12 VIDEO CLIPS, SLIDES AND OTHER WEB-BASED


RESOURCES

For additional explanations on compaction, you can watch some video clips. Web
addresses/links to the video clips are available on the module site under Additional
Resources in a folder named Video clips, slides and other web-based resources.

• Introduction to soil compaction:


https://www.youtube.com/watch?v=qq09VuGYE1E
• General principles of compaction:
https://www.youtube.com/watch?v=Q69a_LiqC3s
• Soil compaction:
https://www.youtube.com/watch?v=ls-idY3uSBc
• Soil compaction:
http://slideplayer.com/slide/5714744/
• Compaction:
http://slideplayer.com/slide/9411880/
• How to calculate a compaction test report:
https://www.qualityengineersguide.com/how-to-calculate-a-compaction-test-
report
• The Standard Proctor compaction test:
https://www.youtube.com/watch?v=rhvcy3112sQ
• Dynamic compaction part I:
https://www.youtube.com/watch?v=zjcT1cO9th4
• Compactor/roller fitted with a sheepsfoot drum compacting dirt at a work site:
https://www.youtube.com/watch?v=yqfLPnEgoy0

39 GEE2601/1
REFERENCE WORKS AND FURTHER READING

Council for Scientific and Industrial Research (CSIR). 1986. Technical manual for
highways (TMH1): standard methods of testing road construction materials:
method A7: the determination of the maximum dry density and optimum
moisture content of gravel, soil and sand. Pretoria.

Das, BM. 2010. Principles of geotechnical engineering. 7th ed. Connecticut: Cengage
Learning.

Das, BM. 2013. FundameWntals of geotechnical engineering. 4th ed. Connecticut:


Cengage Learning.

Das, BM & Sivakugan, N. 2017. Fundamentals of geotechnical engineering. 5th ed.,


international edition. Connecticut: Cengage Learning.

Multiquip. 2011. Soil compaction handbook. Revision A. California.

* Page, figure, table and chapter numbers may differ on updated and revised versions of the prescribed
book. See online version of study guide for updates.

40 GEE2601/1
Study unit 1 4

MOVEMENT OF WATER THROUGH SOIL

4.1 INTRODUCTION

Soils have interconnected voids through which water can flow from points of high
energy to points of low energy. Water in soil is free to migrate through the
interconnected pores and will do so if there is a hydraulic energy gradient across a
body of soil. The flow created in this way is called seepage. To the geotechnical
engineer and technician, seepage is a very important aspect of geotechnical
engineering since soils are very often saturated and water movement is likely to be
encountered in almost all investigations and designs that involve soil. A few examples
of seepage are:

• losses under concrete dams


• losses through earth dams
• excavation dewatering
• stability of natural and human-made slopes
• losses through earth-retaining structures
• Stability effect on subgrade soil

It may be said that the presence of seeping water almost always complicates design,
decreases stability and means additional calculations for the engineering team. The
study of water movement through porous media is very important in geotechnical
engineering for

• estimating underground seepage quantities under varying hydraulic conditions


• investigating groundwater

41 GEE2601/1
• stability analysis for earth-retaining structures, earth dams and foundations

4.2 DEFINITIONS

You will come across the following terms in this study unit:

• Discharge velocity: The quantity of water flowing in unit time through a unit cross-
sectional area of a given soil.
• Hydraulic conductivity/permeability: The ease with which water can flow
through soil.
• Hydraulic energy gradient: The profile of water streaming in an open channel or
a pipe streaming in part full.
• Hydraulic gradient: The change in head per unit distance.
• Seepage: Movement of water through soils.

4.3 LEARNING OUTCOMES

On completion of this study unit, you should be able to determine the hydraulic
conductivity of soil and perform calculations related to movement of water through soil.
More specifically, you should be able to

• determine the hydraulic conductivity of different types of soil


• calculate the flow rates in natural soil layers
• draw flow nets for two-dimensional isotropic seepage conditions in the vertical
plane in confined and unconfined situations
• calculate seepage rates and pore water pressures within the soil
• transform anisotropic cross-sections to isotropic equivalents

To achieve the learning outcomes in this unit, work through the following topics:

• the determination of the hydraulic gradient, which is the driving force behind
seepage across soil bodies, through the use of Bernoulli's equation

42 GEE2601/1
• the use of Darcy's law to determine the coefficient of permeability of soil using the
results of constant head and falling head permeameter tests in the laboratory and
well-point tests in the field
• the determination of seepage loss through pervious soil under impermeable
structures and through earth dam walls, using flow net techniques
• the determination of pore water pressures within a soil volume subject to seepage,
using flow net techniques
• the conflicting requirements of hydraulic filters
• laboratory practicals (constant head and falling head permeameter tests)

This study unit is covered in detail in chapters 6 and 7 (pp. 135-178*) in the prescribed
book. Chapter 6 (pp. 135-162*) covers hydraulic conductivity and chapter 7 (pp. 163-
178*) covers seepage. Since it is important for the geotechnical engineer and
technician to be able to construct flow nets through earth-fill dams, we will examine
this aspect in detail in this study unit. In addition, we will describe the technique of
considering anisotropic soil properties when constructing flow nets.

4.4 OCCURRENCE AND EFFECTS OF WATER IN SOIL

The occurrence and effects of water in soils were covered in Geology and Soil
Mechanics (GSM1501). The emphasis was on static soil water, and you should refresh
your memory by doing some revision of the relevant work covered in the previous
module (GSM1501).

4.5 BERNOULLI'S EQUATION

This area is covered in detail in sections 6.1 and 6.2 (pp. 135-137*) in the prescribed
book.

Water, including soil water, experiences different kinds of energy. The most important
of these are potential energy due to its elevation above some datum level, pressure
energy and kinetic energy because it is not stationary. The total energy is simply the

43 GEE2601/1
sum of these forms of energy. In fluid mechanics, energy is expressed per unit weight
of liquid, resulting in units of length, and the term “head” is used to describe this.
Bernoulli's equation, equation 6.1 (p. 135*) in the prescribed book, thus states that
total head is equal to the sum of pressure, velocity and elevation heads.

𝑢𝑢 𝑣𝑣 2
ℎ= + + 𝑍𝑍 (4.1)
𝛾𝛾𝑤𝑤 2𝑔𝑔

Where:
h = Total head
u = Pressure
v = Velocity
g = Acceleration due to gravity
γw = Unit weight of water

Note the following:


𝑢𝑢
• = Pressure head
𝛾𝛾𝑤𝑤

𝑣𝑣 2
• = Velocity head
2𝑔𝑔

• Z = Elevation head

In pipe and open stream flow, the velocity of the water may be quite high, but that of
water moving through soil is so low that kinetic energy, or velocity head, is usually
ignored. The result is that only two terms remain in the equation, as shown by
equation 6.2 (p. 136*) in the prescribed book.

u
h= +Z (4.2)
γw

It is important to note the physical meanings of the terms. As indicated in figure 6.1
(p. 136*) in the prescribed book, the water will rise to a certain level inside a tube,
called a standpipe piezometer, if it is inserted into the soil at a given position, like A or
B in figure 4.1 below.

44 GEE2601/1
FIGURE 4.1
Pressure, elevation and total heads for flow of water through soil (Das 2014)

The following interpretation is clear:

• The elevation of the position above the chosen datum is the elevation head
(Z).
• The level above the position to which the water rises inside the piezometer is
the pressure head (u/γw).
• The level above the datum to which the water rises inside the piezometer is
the total head (h).

Water always moves from a position of higher total head to a position of lower total
head, as illustrated by figure 6.1 (p. 136*) in the prescribed book, in this case from A
to B. In fact, it is the change in head per unit distance, called the hydraulic gradient,
which is the driving force behind water movement. Equation 6.4 (p. 137*) is for
computing the head loss. Per definition then

∆h ∂h
i= = (4.3)
L ∂L

45 GEE2601/1
Where:
i = Hydraulic gradient
L = Length over which the total head loss occurs

Section 6.1 (pp. 135-137*) in the prescribed book includes a discussion on the
relationship between flow velocity and hydraulic gradient. The conclusion is that in
most types of soil the water flow is laminar and the relationship is linear. This is well
illustrated in figure 6.2 (p. 137*) in the prescribed book.

4.6 DARCY'S LAW

Darcy's law is covered in detail in section 6.3 (p. 138*) in the prescribed book. A linear
relationship between seepage velocity and hydraulic gradient was discussed in the
previous section. The relationship, although linear, is not the same for all types of soil,
and the slope of the straight line in Zone I of figure 6.2 (p. 137*) in the prescribed book
may be different for every material. Efforts to determine the relationship from basic
principles proved unsatisfactory because of the complex maze of interconnected soil
pores through which the water has to flow.

Darcy proposed an empirical law, based on observations, to relate the discharge


velocity of water through soil to the hydraulic gradient. Note that the discharge velocity
is not the same as the actual velocity that a water molecule has as it passes through
the soil, but rather the quantity of flow in unit time through a unit cross-sectional area
at right angles to the direction of the flow. Discharge velocity is related to true velocity
by the porosity or void ratio of the soil, as derived in equations 6.7 to 6.10 (pp. 138-
139*) in the prescribed book.

Darcy's law may be expressed in one of the following ways:


v = ki (4.4)

q = kiA (4.5)

46 GEE2601/1
Where:
v = Discharge velocity in m/s
k = Hydraulic conductivity, also called coefficient of permeability, in m/s
q = Flow rate in m3/s
A = Area perpendicular to flow direction in m2

Hydraulic conductivity has units of m/s in the SI system.

Hydraulic conductivity is a measure of the ease with which water can flow through the
pores of a soil and its actual value for a given soil depends on many factors, or soil
properties, such as pore size distribution, void ratio, particle shape and roughness, as
well as on the viscosity and unit weight of the water. In general, the coarser the soil is,
the higher its hydraulic conductivity. Typical values are indicated in table 4.1 below
(table 6.1 in the prescribed book, p. 140*).

TABLE 4.1: Typical values for hydraulic conductivity for saturated soils

(Source: Das 2010)

The hydraulic conductivity of compacted soil, especially if it contains an appreciable


amount of clay, is extremely sensitive to the compaction water content and the mode
of compaction. The value of the hydraulic conductivity may be a thousand times
smaller after kneading compaction above optimum water content than after
compaction at water contents below optimum.

47 GEE2601/1
Since the viscosity of water is temperature dependent and it is customary to express
hydraulic conductivity at 20 0C, a correction must be applied when other temperatures
are involved, for example in laboratory tests. The hydraulic conductivity at temperature
T (oC) is related to that at 20 0C through the equation

k 20 = κ T k T (4.6)

Where:
k20 = Hydraulic conductivity at 20 0C
KT = Temperature correction coefficient
kT = Hydraulic conductivity at temperature T

Table 4.2 below gives values of kT at different temperatures. Table 6.2 (p. 141*) in the
prescribed book also contains the same values.

TABLE 4.2: Values of temperature correction coefficient

0C KT 0C kT

10 1,299 22 0,953

15 1,135 23 0,931

16 1,106 24 0,910

17 1,077 25 0,889

18 1,051 26 0,869

19 1,025 27 0,850

20 1,000 28 0,832

21 0,976 30 0,797

Example 4.1 (example 6.2 on pp. 144-145 in the prescribed book)

A permeable soil is underlain by an impervious layer, as shown in figure 4.6(a) (p.


145*) in the prescribed book. With k = 4,8 × 10-3 cm/s (4,8 × 10-5 m/s) for the permeable

48 GEE2601/1
layer, calculate the rate of seepage through it in m3/hr per m width if H = 3 m and
α = 5°. Note that H is the vertical elevation difference between the groundwater table
and the bottom of the permeable layer. The angle α is the slope, or inclination, of the
layer.

Solution

Determine the following:

• The dimensions perpendicular to and along the direction of seepage as shown in


figure 4.6(b) (p. 145*) in the prescribed book.

Known information:
• The “thickness” of the stream of water = H cos α
• The area perpendicular to flow = 1 x H cos α per metre width of the layer
• The head loss between two vertical sections spaced at a distance L', ∆h =
L' tan α
• The distance between same sections, but measured along the flow direction =
L'/cos α

Determine the hydraulic gradient:


∆h L' tan α
i= = = sin α
L' / cos α L' / cos α

Apply Darcy's law:

q = kiA = ( 4,8 × 10 −5 ) × (sin 5) × (1× 3 × cos 5) = 1,25 × 10 −5 m 3 / s / m

In 1 hour there are 3 600 seconds, so multiply q by 3 600 to change to m/hr:


q = 0,045 m3/hr/m

Example 4.2

A permeable layer is confined between two impervious layers. With H = 3 m, H1 = 1.1


m, h = 1.4 m, α = 14° and k = 0.5 × 10-3 m/s, find the flow rate through the permeable
layer.

49 GEE2601/1
Solution

Determine the following:

• The dimensions perpendicular to, and along, the direction of seepage:

Known information:

o The “thickness” of the stream of water = H1 cos α


o The area perpendicular to flow = 1 x H1 cos α per metre width of the layer
o The head loss between two vertical sections spaced at a distance L, ∆h = L tan α
o The distance between same sections, but measured along the flow direction =
L/cos α

The hydraulic gradient:


Since the water levels in the two piezometers indicate the total heads at the two
sections, h is the head loss. Therefore

h h cos α
i= =
L / cos α L

Apply Darcy's law (equation 4.5):

 1,4 × cos 14° 


q = kiA = (0,5 × 10 −3 ) ×  −5 3
 × (1× 1,1× cos 14°) = 1,812 × 10 m / s / m
 40 

q = 1,812 × 10-5 × 3 600 = 0,065 m3/hr/m

4.7 LABORATORY DETERMINATION OF HYDRAULIC


CONDUCTIVITY

This section is covered in section 6.5 (pp. 141-146*) in the prescribed book. Hydraulic
conductivity can be determined in the laboratory using one of the following tests:

• constant head permeameter test for relatively coarse-grained soil


• falling head permeameter test or relatively fine-grained soil
• hydraulic cell test for fine-grained soil

50 GEE2601/1
In this module (GTE3601), we will only cover the constant and falling head tests. The
accuracy of laboratory tests depends largely on how well the test conditions can
simulate the field situation. It is therefore essential to use undisturbed representative
soil samples if the in situ hydraulic conductivity of a natural soil deposit is required.
Complete saturation of the specimens is essential for a representative value of the
hydraulic conductivity. If air is present, it will be in the form of millions of small bubbles
which will pose obstructions to the water movement, thereby decreasing the hydraulic
conductivity of the soil.

4.7.1 Constant head permeability test

Figure 6.4 (p. 142)* in the prescribed book shows a sketch of the constant head
permeameter apparatus. You must learn the sketch and the essentials of the test
procedure for assignment and examination purposes. Note that two standpipes
(piezometers) are sometimes attached to the body of the tube containing the specimen
to improve the accuracy o f the test result. The calculation procedure is a direct
application of both Darcy's law and Bernoulli's equation and is an excellent way to
illustrate both.

Carefully study figure 4.2 below. It is similar to figure 6.4 (p. 142*) in the prescribed
book, but a few extra dimensions and text have been added to aid with the explanation.
The datum is taken as the bottom of the lower tank. Now the derivations can be made.

51 GEE2601/1
FIGURE 4.2
Constant head permeability test (Das 2010)

Determine the total heads at positions A and B:


uA
hA = + ZA
γw

uB
hB = + ZB
γw

Next determine the hydraulic gradient between A and B:


h A − hB h
i= =
L L

Apply Darcy's law:


h Q
q = kiA = k A=
L t

52 GEE2601/1
Where:
• A = Cross-sectional area of the soil specimen
• Q = Total volume of water collected
• t = Duration of water collection

After rearranging the variables


QL
k= (4.7)
Aht

Note:
• Equation 4.7 can only be used for the calculation of k using test results from the
constant head test.
• The constant head permeability test is primarily for coarse-grained soils.
• The test is based on the assumption of laminar flow; therefore, Darcy's law applies.

The simplified procedure of the test is as follows:

• Measure the quantity of water, Q, flowing through the soil sample.


• Measure the length of the soil sample.
• Measure the head of the water, h.

Figures 4.3 and 4.4 below show the laboratory test and apparatus, respectively, for
conducting the constant head test.

53 GEE2601/1
FIGURE 4.3 FIGURE 4.4
Constant head permeability lab test Constant head permeability apparatus
(Cemmlab 2010) (Civilblog 2015)

Example 4.3

A constant head test has been done on a sand sample 250 mm in length and 75 mm
in diameter. With a head loss of 500 mm, the total volume of water collected was 267
ml in 2 minutes and 6 seconds. The temperature was 25 °C. Determine the hydraulic
conductivity (coefficient of permeability) of the sand.

Solution

Convert all units to the SI system:

• Q = 267 ml = 267cm3 = 267 x 10-6 m3


• L = 250 mm = 0,250 m
• A = π/4 × (75)2 = 4 417,867 mm2 = 4 417,867 × 10-6 m2
• h = 500 mm = 0,500 m
• t = 126 s

54 GEE2601/1
Apply equation 4.7:
QL 267 × 10 −6 × 0,250
k= = = 2,398 × 10 − 4 m / s
Aht 4417,9 × 10 × 0,500 × 126
−6

Since the test was performed at 25 °C, there is a correction for temperature. Apply
equation 4.6:
k 20 = κ T k T = 0,889 × 2,398 × 10 −4 = 2,13 × 10 −4 m / s

Study example 6.1 (pp. 143-144*) in the prescribed book. The solution is included.

4.7.2 Falling head permeameter test

Figure 6.5 (p. 143*) in the prescribed book shows a sketch of the falling head
permeameter apparatus. You must learn the sketch and the essentials of the test
procedure for examination purposes. In this test the head at the outflow end of the
sample is held constant by means of a tank, while the head at the inflow end is
governed by the water level in the standpipe. It should be clear that the head loss
across the specimen is equal to h, the level of the water in the standpipe above the
constant water level in the tank. As water passes through the specimen, the water
level in the standpipe drops and the head at the top decreases, hence the name of the
test. The diameter of the standpipe and the rate of drop of the water level are used to
calculate the flow rate, q.

Carefully study figure 4.5. It is similar to figure 6.5 (p. 143*) in the prescribed book.

55 GEE2601/1
FIGURE 4.5
Falling head permeability test (Das 2014)

Apply Darcy's law at time t when the head loss across the specimen is h:

h dh
q = kiA = k A = −a (4.8)
L dt

56 GEE2601/1
Where:
• a = Cross-sectional area of the standpipe
• A = Cross-sectional area of the specimen
• L = Specimen length

Since head decreases with time, and q must have appositive value, the negative sign
is imported in the last part of equation 4.8. After some rearrangement of the equation
to separate variables, both sides may be integrated between the limits 0 and t for time
and h1 and h2 for head loss. The result is as follows, again after rearrangement:

aL h1 aL h
k= ln = 2,303 log 2 (4.9)
At h 2 At h1

Where:
• t = Duration of test
• h1 = Initial head loss across the specimen
• h2 = Final head loss across the specimen

In the equation, “ln” and “log” refer to logarithm to the bases exponential constant (e)
must not be confused with the void ratio, and 10, respectively.

Note:
• Equation 4.8 can only be used to calculate hydraulic conductivity from data
obtained in the falling head test.
• The flow through the standpipe equals the flow through the soil.

The simplified procedure of the test is as follows:

• Record initial head difference, h1 at t = 0.


• Allow water to flow through the soil specimen.
• Record the final head difference, h2, at t = t2.

Figures 4.6 and 4.7 show the laboratory test and apparatus, respectively, for
conducting the constant head test.

57 GEE2601/1
FIGURE 4.6 FIGURE 4.7
Falling head permeability test (Uvm 2010) Constant head permeability apparatus
(Uvm 2010)

Example 4.4

A falling head test was performed on a clay specimen and the following results were
obtained:

• Initial head (above tank level) in the standpipe = 1,5 m


• Final head in the standpipe = 0,605 m
• Duration of test = 4,7 minutes
• Specimen length = 150 mm
• Specimen diameter = 55 mm
• Standpipe diameter = 5 mm
• Temperature = 20 °C

Determine the hydraulic conductivity.

58 GEE2601/1
Solution

Convert all units to the standard SI equivalents:

• t = 4,7 × 60 = 282 seconds


• a = π × 52/4 mm2 = π × 52/4 × 10-6m2
• A = π × 552/4 mm2 = π × 552/4 × 10-6m2
• L = 0,150 m
• h1 = 1,500 m
• h2 = 0,605 m

Apply equation 4.9:

2
aL h2 π×5 × 10 −6 × 0,150 1,500
k = 2,303 log = 2,303 4 log = 3,99 × 10 −6 m / s
At h1 π× 55 2 −6
× 10 × 282 0,605
4

Note:

• No temperature correction is necessary since the test was performed at 20 °C.

Did you know?

The constant head permeability test is used primarily for coarse-grained sands.

The falling head test is used for both coarse-grained soils as well as fine-grained soils.

4.8 EMPIRICAL RELATIONS FOR HYDRAULIC CONDUCTIVITY

This topic is covered in detail in section 6.6 (pp. 146-152*) in chapter 6 in the
prescribed book. Many empirical relations have been established between the
hydraulic conductivity of coarse-grained soils and other soil parameters. These
relations can be used to estimate hydraulic conductivity when accurate values are not
required. Probably the most useful is the method proposed by Hazen (1930) which
relates hydraulic conductivity to effective size of fairly uniform sand.

59 GEE2601/1
For granular soils:
2
k = cD10 (4.10)

Where:
• K = Hydraulic conductivity (m/s)
• D10 = Effective size of the sand (mm)
• c = A constant that varies between 1,0 × 10-2 and 1,5 × 10-2

For cohesive soils, Tavenas et al (1983) propose the following method:


𝒆𝒆𝒏𝒏
𝒌𝒌 = 𝑪𝑪 �𝟏𝟏+𝒆𝒆� (4.11)

Where:
c = A constant determined in the laboratory through an experiment

n = A constant determined in the laboratory through an experiment

e = Void ratio

4.9 FIELD DETERMINATION OF HYDRAULIC CONDUCTIVITY BY


PUMPING FROM WELLS

This topic is covered in detail in section 6.8 (pp. 155-157*) in chapter 6 in the
prescribed book. The average hydraulic conductivity of a large volume of soil may be
determined by the well pumping test, which makes it a much better method than
laboratory methods. Also note that well pumping tests determine the horizontal
hydraulic conductivity of soil since flow is mainly horizontal. Laboratory tests on
undisturbed samples taken from the surface give vertical hydraulic conductivity.
Unfortunately, in situ testing is expensive and time-consuming and is usually only done
if the hydraulic conductivity of the soil is really of great importance.

Pumping tests involve a central borehole from which water is pumped, plus a series
of observation holes in a concentric pattern, at different distances, around it. The
observation holes are used to measure the total head at their respective positions and
from these readings, and the rate of pumping, the hydraulic conductivity can be

60 GEE2601/1
calculated once the phreatic surface (water table) has become stationary. Figure 4.8
below shows a typical pumping well field test.

FIGURE 4.8
Typical pumping well field test

4.9.1 Pumping from a well in an unconfined aquifer

This version of the pumping test is used when the phreatic surface, or water table, is
within the permeable layer. This is usually the case when the permeable layer is the
topmost one in the soil profile. The test setup is shown in figure 4.9 of the prescribed
book. You must learn the sketch and the essentials of the test procedure for
examination purposes. Note that h is defined relative to the bottom of the pervious
layer. Figure 4.9, which is similar to figure 6.11 (p. 155*) in the prescribed book, shows
a pumping test from a well in an unconfined permeable layer underlain by an
impermeable stratum.

61 GEE2601/1
FIGURE 4.9
Pumping test from a well in an unconfined permeable layer
underlain by an impermeable stratum (Das 2014)

Water is pumped from the central hole at a constant rate q. The hydraulic gradient at
a distance r away from the central borehole is given by the slope of the phreatic
surface:
dh
i=
dr

The area across which the water flows towards the central well is the circumferential
area of a cylinder with radius r and height h:
A = 2πrh

Darcy's law can now be applied:


dh
q = kiA = k 2πrh
dr

62 GEE2601/1
Rearranging the equation to separate variables and integration between two
observation wells, that is between the limits r2 and r1, and h2 and h1, lead to the
expression for k:

2,303q log r1
r2
k= (4.12)
π (
h12 − h 22 )
Note:
• It is recommended that values of h and r from several sets of observation wells be
used for the calculation in order to determine an average value of the hydraulic
conductivity.
• Equation 4.12 can only be used to calculate hydraulic conductivity from data
obtained in the unconfined aquifer pumping test.

Example 4.5

A 9.15 m thick layer of sand overlies impervious rock. The water table, or phreatic
surface, was at 1.22 m below the surface before a field test for hydraulic conductivity
commenced. During the test, water was pumped from a central well at a rate of 5 680
l/min and the drawdown of the water table was noted in two observation wells on a
radial line and at distances of 3.05 and 30.5 m from the central well. The water table
dropped to 4.57 m below the surface in the nearest well and to 2.13 m in the furthest
well. Determine the hydraulic conductivity of the soil in m/s if the temperature of the
soil water was 15 °C at the time of testing.

Solution

Convert and list all variables in SI units:

• q = 5 680 l/m = 5,680 m3/m = 0.0947 m3/s


• r1 = 30.5m
• r2 = 3.05 m
• h1 = 9.15 – 2.13 m = 7.02 m
• h2 = 9.15 – 4,57 = 4.58 m

63 GEE2601/1
Apply equation 4.12:

2,303q log r1 2,303 × 0,0947 log 30,5


r2 3,05
k= = = 0,00245m / s
π (h − h
1
2 2
2 ) π (7,02 − 4,58
2 2
)
Apply equation 4.6 and table 4.2 for temperature correction:

k 20 = κ T k T = 1,135 × 0,00245 = 2,78 × 10 −3 m / s

4.9.2 Pumping from a well in a confined aquifer

This version of the pumping test is used when the permeable layer is confined between
two impervious layers. The test setup is shown in figure 6.12 (p. 156*) in the
prescribed book. You must learn the sketch and the essentials of the test procedure
for examination purposes. Note that h is defined relative to the water level in the central
borehole, and not relative to the bottom of the permeable layer. From the final equation
you will see that the datum relative to which h is defined is not important since it is only
the difference in the h-values that matters.

FIGURE 4.10
Pumping test from a well penetrating the full depth in a confined aquifer (Das 2014)

64 GEE2601/1
The derivation of the expression for hydraulic conductivity, k, is the same as was
performed for the unconfined aquifer, but the cylindrical surface across which the
water flows is independent of the radius and is given by

A = 2πrH

After integration, using the same limits as before, the following expression is derived
at:

2,303q log r1
r2
k= (4.13)
(
2πH h1 − h 2 )
Note:

• Equation 4.13 can only be used to calculate hydraulic conductivity from data
obtained in the confined aquifer pumping test.
• Impervious layer = Impermeable layer.
• A clay layer is considered an impermeable layer.
• Always take the maximum value of r as r1→ the value of h1 must be larger than h2.
• r is the distance from the centre of the well to the centre of the observation well.
• h is the elevation of water in the observation well.

Example 4.6

A layer of sand 3.0 m thick is confined between impermeable rock below and a 5.0 m
thick impermeable clay on top of it. The water table, or phreatic surface, was at 1.0 m
below the surface before a field test for hydraulic conductivity commenced. During the
test, water was pumped from a central well at a rate of 100 l/min and the drawdown of
the water table was noted in two observation wells on a radial line and at distances of
3.0 and 30.0 m from the central well. The water table dropped to 3.0 m below the
surface in the central well, to 2.0 m in the nearest well and to 1.5 m below the surface
in the furthest well. Determine the permeability of the soil in m/s if the temperature of
the soil water was 20 °C at the time of testing.

65 GEE2601/1
Solution

Relate the dimensions given in the problem statement to figure 4.10 in the prescribed
book:

• r1 = 30.0 m
• r2 = 3.0 m
• h1 = 1.5 m above the level in the central hole
• h2 = 1.0 m above the level in the central hole
• q = 100 l/m = 0,00167 m3/s
• H = 3.0 m

Apply equation 4.12:

r1
2,303q log
r2 2,303 × 0,00167 × log 30,0
3,0
k= = = 4,08 × 10 −3 m / s
(
2πH h1 − h 2 ) 2 × π × 3,0 × (1,5 − 1,0 )

Note:

• With h1 = 6.5 m and h2 = 6.0 m, the same result would be obtained.


• There is no correction required for temperature.

4.9.3 Other types of field tests for hydraulic conductivity

Pumping from wells is not the only type of test that can be used in the field to determine
hydraulic conductivity. Many other tests, which involve much smaller volumes of soil,
have been developed. Most of these involve infiltration into the soil, from the surface
or from open-ended or perforated pipes penetrating some distance into the soil. These
tests generally do not have a sound theoretical basis and rely on empirical relations
or numerical techniques. They are useful, however, for specific purposes, and you
should know about their existence. Descriptions can be found in many textbooks and
other literature but will not be covered in this module (GEE2601).

66 GEE2601/1
4.10 FLOW NETS

This topic is covered in section 7.3 (pp. 165-173*) in chapter 7 in the prescribed book.
The movement of water through soil, or of any liquid through any porous medium, is
governed by Laplace's equation of continuity. Section 7.2 (pp. 163-165*) in the
prescribed book contains the derivation of Laplace's equation in detail, but for our
purpose it is sufficient to know that it was derived from the fact that the amount of
water in any given soil element subject to seepage stays constant, since the element
is, and remains, saturated. The equation is repeated below in three dimensions:

∂ 2h ∂ 2h ∂ 2h
kx + ky + kz =0
∂x 2 ∂y 2 ∂z 2

We will only be considering two-dimensional flow, in the x-z plane, which represents
a vertical cross-section parallel to the direction of water movement. Seepage is forced
to be two-dimensional by the geometry of the situation in most cases. This means that
the middle term in the equation may be dropped.

∂ 2h ∂ 2h
kx + kz =0 (4.14)
∂x 2 ∂z 2

Initially we will only consider isotropic soils for which kx = kz, so that equation 4.14
simplifies to

∂ 2h ∂ 2h
+ =0 (4.15)
∂x 2 ∂z 2

The solution of Laplace's equation may be represented graphically by a flow net, a


form of curvilinear net made up of a set of flow lines and equipotential lines, as shown
by the examples in figure 7.2 (p. 166*) in the prescribed book.

4.10.1 Flow lines and flow channels

Water moves from positions of high total head to positions of low total head. A flow
line represents the path that a molecule of water follows as it migrates through the soil,
smoothed into a curved shape. Minor deviations, as the molecule winds its way around

67 GEE2601/1
soil particles, are not included. The flow path is not straight, because the whole soil
volume is saturated and the water molecules move in a side-by-side manner, like a
group of marching soldiers, each following in the “footsteps”, or flow line, actually flow
curve, of the molecule just ahead, within the physical seepage boundary conditions.
Of course, there are an infinite number of these “parallel” flow lines, but we only draw
a few on a flow net, not to clutter the cross-section.

A prerequisite of Laplace's equation is that flow lines do not meet, join up, or cross.
The flow lines that we do draw may be seen as the boundaries of streams, or flow
channels, within the soil, and the positions of the flow lines are chosen in such a way
that each flow channel carries the same flow, ∆q. For Nf +1 flow lines, which include
the physical boundaries of the flow region, there will be Nf or flow channels and the
total flow would be

q = Nf ∆q

4.10.2 Equipotential lines and equipotential drops

As the water moves along the different flow lines, it experiences a continuous loss of
total head. If the points with the same total head are joined, also called potential, on
the cross-section in the same way as when drawing contour lines on a topographical
map, the result would be a set of equipotential lines. Since they are part of the solution
of Laplace's equation, equipotential lines must not meet, cross, or join up in any way.

An important principle is illustrated by figure 7.2(a) (p. 166*) in the prescribed book.
Although the total head is constant along the equipotential, as shown by the identical
levels of the water in the two standpipes, the individual components, pressure head
and elevation head, are not constant. As (Z) decreases with position, the pressure
head (u/γw) increases by an equal amount.

An infinite number of equipotential lines may be drawn, but that would make the
graphical solution of Laplace's equation impossible to interpret. Just a sufficient
number of lines are chosen to represent the variation of total head on the cross-
section. A good choice would be to have the loss in total head constant between all

68 GEE2601/1
equipotential lines, say at a value of ∆h. This is again similar to contour lines on a map
drawn at, say, 10 m height intervals. With Nd+1 equipotential lines, there will be Nd
equal potential drops and ∆h is calculated from

H
∆h =
Nd

Where:

• H = Difference in potential across the flow region

4.10.3 Flow nets for isotropic flow conditions

In the preceding sections we discussed the flow lines and equipotential lines that make
up the graphical solution of Laplace's equation. If the soil is isotropic, that is the
hydraulic conductivity is the same in the x and z directions, equation 4.15 applies. As
a result, a mathematical requirement is that the two sets of curves in the graphical
solution must be perpendicular to each other, that is, they must cross at right angles.
This can clearly be seen in figures 7.4 and 7.5 (pp. 168-169*) in the prescribed book.

4.10.4 Calculation of seepage from a flow net

Since we have decided to draw the flow lines to form flow channels that each carry the
same flow, ∆q, and to draw the equipotential lines to have equal potential drops, ∆h,
we can now proceed to calculate the value of ∆q, as shown by figure 7.5 (p. 169*) in
the prescribed book. Apply Darcy's law to each of the three four-sided elements shown
in the figure, and you will see that

∆h ∆h ∆h
∆q = kiA = k b1 = k b2 = k b3 (4.16)
l1 l2 l3

Where:

• b1, b2, and b3, = Area in each element, perpendicular to the flow direction, across
which the flow takes place if a unit width is considered

69 GEE2601/1
𝑏𝑏1 𝑏𝑏2 𝑏𝑏3
From equation 4.16 it is clear that = = = constant within the flow channel, and
𝑙𝑙1 𝑙𝑙2 𝑙𝑙3

that the value of ∆q is a function of this ratio between the dimensions of the elements.
𝑏𝑏
The ratio may take on any value, but if all the flow channels are to have the same
𝑙𝑙
𝑏𝑏
flow, ∆q, this ratio should be the same for all elements. A good choice is to have 𝑙𝑙 =

1, that is, the four-sided elements of the flow net should all be made “squares” as
shown in figure 7.4 (p. 168*) in the prescribed book.

The total flow of water may be calculated in the following way, if all flow net elements
are squares, all flow channels carry the same flow, ∆q, and all potential drops are
equal, as described above.

From the previously determined relationships

q = Nf ∆q

H
∆h =
Nd

b
=1
l

The derivation for total flow can be derived at

∆h H
q = Nf ∆q = Nf k b = Nf k∆h = Nf k
l Nd

so that

Nf
q = kH (4.17)
Nd

In every flow net there is usually one flow channel that cannot be made to have square
elements, like the bottom one in figure 7.6 (p. 170*) in the prescribed book. This flow
𝑏𝑏
channel will then carry a fraction of ∆q, proportional to the ratio 𝑙𝑙 , but this ratio must

be the same everywhere in the flow channel, otherwise the graphical solution is

70 GEE2601/1
incorrect. Nf will then not be an integer, in order to accommodate this smaller flow
channel.

4.11 CONFINED FLOW NETS

Flow through soil may be of the confined type or the unconfined type. For purposes of
clarity, we will study flow nets for these two flow types separately.

4.11.1 Boundary conditions of a confined flow net

When a flow region is such that two potential boundary conditions as well as two flow
boundary conditions are dictated by the geometry, the flow is said to be confined. This
is typical of seepage through soil below human-made structures, such as dam walls
made of relatively impervious materials like concrete or steel.

The potential boundary conditions are given by known and usually constant total
heads at the ground surfaces where the water enters and exits the flow region, like
surfaces ab and de in figure 7.2(b) (p. 166*) in the prescribed book. In this figure, let’s
choose the datum to be the surface of the water level above de. The following values,
shown in table 4.3 below, can then be determined:

TABLE 4.3: Potential boundary conditions

At surface ab At surface de
Pressure head = u/γw H1 H2
Elevation head = Z -H2 -H2
Total head = h H1 - H2 H2 - H2 = 0

The potential difference for the cross-section is

H = hab - hde = H1 - H2, which is the difference between the water levels above the two
surfaces.

71 GEE2601/1
The flow boundary conditions are determined from the impermeable material surfaces
that surround the flow region. In figure 7.2(b) (p. 166*) in the prescribed book fg is the
lower flow boundary, or bottom flow line, formed by the impervious layer. The upper
flow boundary, or top flow line, is formed by the surface of the sheet pile, acd, since
water cannot flow through the sheet pile.

4.11.2 Construction of a confined flow net

Constructing flow nets takes several trials, so you need to keep the boundary
conditions in mind. It is important that you keep the following in mind:

• Flow nets have four boundary conditions, two flow boundaries and two potential
boundaries.
𝑏𝑏
• The elements of the net must be square, that is 𝑙𝑙 = 1, or constant and < 1 in one

flow channel for which squares are impossible to draw. This is the “square rule”.
• Flow lines and equipotential lines intersect at right angles. This is the “right angles
rule”.
• Equipotential lines never meet, cross, or join one another.
• Flow lines never meet, cross, or join one another.
• All flow channels between flow lines carry the same flow, except the odd one for
𝑏𝑏 𝑏𝑏
which 𝑙𝑙 < 1. For this odd flow channel, the flow is proportional to 𝑙𝑙 .

• All potential drops between equipotential lines are the same.

A series of steps are involved when a flow net is constructed. We will use example 4.7
below to demonstrate these. Only by studying example flow nets in textbooks and
doing some exercises will you become good at drawing flow nets.

72 GEE2601/1
Example 4.7

A riverbed consists of sand on top of impervious rock. The sand is 7.5 m thick and has
a hydraulic conductivity of 4 × 10-5 m/s. A 5 m wide concrete wall is constructed to dam
the water. The bottom of the wall is at a depth of 2.5 m below the sand's surface and
the water level on the upstream side rises to 2.5 m above the sand's surface. On the
downstream side the water level is 0.5 m above the sand's surface. Draw a flow net
and determine the amount of water seeping below the wall. The solution is shown in
the next page.

Solution

Draw the cross-section of the flow region to scale, to cover at least half an A4-size
sheet when doing tutorials, or much larger when dealing with complicated real-life
problems. Use ink for doing this because extensive erasing is going to be done later.
See figure 4.11 below for the cross-section of the situation described in example 4.7.

73 GEE2601/1
FIGURE 4.11
Geometry of confined flow problem

Determine the two potential boundary conditions, decide on a datum and calculate the
boundary potentials. The potential boundaries in figure 4.11 are AB and CD. Choose
the datum at the water surface level above CD. The following are then determined:

At surface AB At surface CD

Pressure head = u/γw 2,5 m 0,5 m


Elevation head = Z -0,5 m -0,5 m
Total head = h 2,0 m 0m

Determine the two flow boundary conditions (top and bottom flow lines). The top flow
boundary on figure 4.11 is the surface of the impervious concrete, BEFC, whereas the
bottom flow boundary is the surface of the impervious rock, GH.

74 GEE2601/1
In a flow net, the flow lines closest to a specific flow boundary will approximate its
shape, and the further it is away from the boundary, the less the boundary will influence
its shape. Using a soft pencil, draw a flow line about halfway between the top and
bottom flow lines, but a little closer to the top one. This flow line is shown on
figure 4.12. Remember that the flow line must meet the two potential boundaries at
right angles.

FIGURE 4.12
First flow line and first equipotential line

Start at either the left- or the right-hand side of the cross-section and draw an
equipotential line that meets or intersects the existing flow lines, including the flow
boundaries, at right angles and forming a square element with the top flow line, the
flow line that you drew in the previous step, as well as with the nearest potential
boundary condition. This equipotential line is shown on figure 4.12 for the current
example.

Draw a second equipotential line that forms a square with the top flow boundary, the
flow line and the equipotential from the previous steps, again intersecting all flow lines
at right angles. Add more equipotential lines, while obeying the squares rule and the
right angles rule, until you end up near the second potential boundary. Don't worry too

75 GEE2601/1
much if the last element is not square because some adjustments will be made later.
The equipotential lines are shown on figure 4.13 below.

FIGURE 4.13
Add equipotential lines to form square elements above first flow line.

Draw one more flow line or two if there is room, below the first one, again obeying the
squares rule and the right angles rule for the elements above each flow line. See figure
below 4.14 for the results.

76 GEE2601/1
FIGURE 4.14
Add flow line to form square elements below first flow line.

The flow net can now be given smaller elements by subdividing the existing elements
into smaller squares. This is done by adding flow lines and equipotential lines between
the ones that you have drawn previously, always intersecting at right angles. Figure
4.15 shows the subdivided flow net for example 4.7.

FIGURE 4.15
Subdivide square elements and adjust to refine flow net.

77 GEE2601/1
You now have a rough flow net which may be adjusted by repositioning some or all of
the flow lines and equipotential lines in order to make the elements better squares, or
of a constant b/l ratio in the bottom flow channel if necessary. Sometimes slight
movement of a line, or only part of it, is required, and in other cases it may be
necessary to shift more than one line, or replace one line with two, or two with three,
and sometimes you have to erase everything and start all over.

Not too much time should be spent trying to perfect a flow net, though, but the rules
must be obeyed. If you cannot clearly see that elements in the net are not square, or
intersections between flow lines and equipotential lines are not at right angles, and no
violations of the other flow net properties listed above exist, the flow net may be
regarded as accurate enough to be used for calculations. The flow net in figure 4.15
will be regarded as the solution to example 4.7.

Once the flow net is a good graphical solution of Laplace's equation, the flow may be
calculated. Count the number of equipotential drops and flow channels: Nd = 12 and
Nf = 4,333. Determine the potential drop over the cross-section:

H = hAB – hCD = 2.0

From equation 4.15

Nf 4,333
q = kH = 4 × 10 −5 × 2,0 × = 2,89 × 10 −5 m 3 / s
Nd 12

The total head at any position in the flow region may be determined from the potentials
at the boundaries and the equipotential line that intersects the position, because the
potential drop between equipotential lines has a constant value ∆h. Interpolation may
be required if the position is between two equipotential lines. The pressure head, and
hence the pore pressure, at the position can be calculated from the total head and the
elevation head. To illustrate this, take bottom corner E of the wall:

hCD = 0,0

hAB = 2,0

∆h = H/Nd = 2/12 m

78 GEE2601/1
E is at 9 equipotential drops higher than CD:

hE = 0,0 + 3 × ∆h = 1,5 m

Or, E is at 3 equipotential drops lower than AB:

hE = 2.0 – 3 × ∆h = 1.5 m

ZE = -3.0 m because E is situated at a level 3,0 m below the datum

From Bernoulli's equation:

u/γw = h –Z = 1.5 – (-3,0) = 4.5 m

γw = 9.81 kN/m3, so that

uE = 44,15 kN/m2

4.12 UNCONFINED FLOW NETS

Unconfined flow is very common in natural soil structures, especially when the
topography is sloped. Another case where unconfined flow occurs is through human-
made water-retaining structures like earth dams and dykes. Note that unconfined flow
is not covered in the prescribed book, only in the study guide.

4.12.1 Boundary conditions of an unconfined flow net

The difference between confined and unconfined flow is that the structural boundaries
on the cross-section do not confine the flow from above. The upper flow boundary
condition is governed by the geometry in an indirect way and must be determined
before a flow net can be constructed.

Since unconfined flow can become very complicated, we will consider only the
relatively simple case of an earth dam resting on a horizontal impervious base and
with a horizontal toe drain in this course. The function of the filter is to prevent water
from discharging on the downstream face of the dam, which is detrimental to the

79 GEE2601/1
stability of the downstream slope. The water discharges into the filter, which, if properly
designed, will also serve to protect the soil from erosion.

Figure 4.16 shows a cross-section of a typical earth-fill dam on a horizontal impervious


foundation with a horizontal toe filter. The known boundary conditions are as follows:

AB is the bottom flow boundary condition.

AD is a potential boundary condition, total head kept constant by the free water
surface.

BE is a potential boundary, since the filter is full of water at atmospheric pressure.

FIGURE 4.16
Cross-section through earth-fill dam with horizontal filter on impervious foundation

Take BE as the datum. The heads at the potential boundaries may then be determined
as follows:

At surface BE At surface AD
Variable:
Pressure head = u/γw 0 = Hw at A
= 0 at D
Variable:
Elevation head = Z 0 = 0 at A
= Hw at D
Total head = h 0 = Hw

80 GEE2601/1
The top flow boundary condition, or top flow line, must start at D, at right angles to AD,
and end in the filter, at right angles to BE to obey the right angles rule. It has been
established that the top flow line can, for almost its entire length, be presented by a
parabola with its focus at B. The parabola passes through a point F on the external
free water surface where

DF = 0.3 × CD (4.18)

The equation of the parabola, in terms of the axes shown on figure 4.17, is

z2
x = x0 − (4.19)
4x 0

Using the coordinates (x, z) of F, the point through which the parabola is known to
pass, the value of x0 may be determined from equation 4.16 as

x 0 = 21  x + x 2 + z 2  (4.20)
 

The value of x0 can then be substituted into equation 4.19 to calculate coordinates of
points on the parabola, in order to draw the position of the top flow line. Since the
parabola does not pass through D, a correction must be made, in the form of a curve
that starts at D, at right angles to AD, and joins the parabola at G. The position of G is
not fixed by any rule and may be adjusted to assist with the shape of the elements of
the flow net. Figure 4.17 below shows what the top flow line should look like.

FIGURE 4.17
Top flow line for unconfined flow

81 GEE2601/1
The pore water pressure, u, is equal to zero (atmospheric) along the top flow line and
it acts as phreatic surface, or water table. This means that the total head is equal to
the elevation head, and there must be equal vertical intervals between the points of
intersection between successive equipotential lines and the top flow line. If this is not
obeyed, the flow net is incorrect, even if all other rules have been complied with. This
is illustrated in the example below.

4.12.2 Constructing the unconfined flow net

The process of drawing the top flow line and the rest of the flow net will be explained
step-by-step by means of example 4.8. The properties of the flow net that we
discussed in section 4.11.2 apply to unconfined flow nets too, and the rules must
always be adhered to. In addition, there is the rule of equal vertical intervals for the
intersections of the equipotential lines with the top flow line.

Example 4.8

An earth-fill dam is constructed on an impervious rock foundation and a horizontal filter


is provided at the toe. The dam will be 35 m high with a crest width of 10 m and both
faces will slope at 2:1 horizontal to vertical. The water level in the dam will be 30 m
above the soil-to-rock contact and the filter will extend for 40 m below the toe. Draw a
complete flow net in the dam and calculate the amount of water seeping through the
dam wall if the hydraulic conductivity = 2.5 × 10-7 m/s.

Solution

Draw the cross-section to scale as shown in figure 4.18. Use the same scale in the
vertical and horizontal directions, even if the cross-section is very wide and flat.

82 GEE2601/1
FIGURE 4.18
Geometry of earth-fill dam

Determine the coordinates of the point F through which the parabola of the top flow
line passes by means of equation 4.18:

DF = 0.3 × CD = 0.3 × 60 = 18 m

The coordinates are x = - (18 + 10 + 10 + 30) = -68 m

z = 30 m

Solve for x0 using equation 4.19:

x 0 = 21  x + x 2 + z 2  = 21  − 68 + ( −68) 2 + 30 2  = 3,16 m
   

Substitute this into equation 4.18:

z2 z2
x = x0 − = 3,16 −
4x 0 4 × 3,16

Choose values of z, at constant intervals between 0 and 30 m, to substitute into the


equation and calculate the corresponding value of x in order to obtain coordinate pairs
to plot the position of the parabola.

83 GEE2601/1
This is shown in table 4.4 below.

TABLE 4.4: X coordinates

Z 0,0 5,0 10,0 15,0 20,0 25,0 30,0


X 3,16 1,19 - 4,75 - 4,63 - 28,47 - 46,20 - 68,00

The parabola can now be drawn and the correction made to have the top flow line start
at D. Figure 4.19 shows the corrected top flow line.

FIGURE 4.19
Top flow line and rough flow net

The missing boundary condition has been determined now and all that remains is to
complete the flow net. Remember that equipotential lines must meet the top flow line
at equal vertical intervals. Use the coordinate points calculated for the parabola to
determine the starting points of the equipotential lines since they were chosen at
constant z-intervals specifically for this purpose. The points between F and C on the
parabola move horizontally towards the right to new positions on the corrected curve
DG. The equipotential lines are shown in figure 4.19. Remember that they must meet
the bottom flow line at right angles and that all equipotential lines must end on the flow
line, and not in the filter.

84 GEE2601/1
Starting from the top, add on more flow lines to intersect existing equipotential lines at
right angles and to form square elements. The flow lines must start on the upstream
face and end in the filter. In figure 4.19, there is only room for one flow line as shown.

The flow net can now be refined by subdividing and shifting lines to get better squares
and intersections, but the starting points of the equipotential lines on the top flow line
must obey the equal vertical intervals rule. In addition, all flow lines must end in the
filter and all equipotential lines must end on the bottom flow line. The flow net in figure
4.20 is a good example of what can be achieved without using too much time. Note
that figure 4.20 was drawn to a different scale than the previous ones in order to make
the flow net detail more visible.

FIGURE 4.20
Refined flow net

The seepage can now be calculated from equation 4.17:

Nf 3
q = kH = 2,5 × 10 −7 × 30,0 × = 1,875 × 10 −6 m 3 / s
Nd 12

4.13 FLOW NETS FOR ANISOTROPIC FLOW CONDITIONS

In section 4.10 of this study unit the equation of Laplace was derived for conditions of
anisotropic flow in which the soil's hydraulic conductivity in the horizontal direction is
different from that in the vertical direction (equation 4.14) and for isotropic flow in which

85 GEE2601/1
the hydraulic conductivity is the same in all directions (equation 4.15). All flow nets
constructed in later sections were for isotropic flow.

In reality, almost all soils have anisotropic hydraulic properties caused by natural
layering in transported sediments, or by compaction of coarse- and fine-grained
materials. The permeability, or hydraulic conductivity, in the horizontal direction, kx,
may be up to several times the permeability in the vertical direction, kz.

The result is that the flow net is distorted from square elements and right angle
intersections between flow lines and equipotential lines, and the flow net is very
complex to construct. There is an easy way out, fortunately, and that is by transforming
the flow area to an equivalent isotropic area.

All x coordinates, i.e. all x dimensions, of the flow area are transformed into xt
coordinates by means of equation 4.21 below, keeping z dimensions as they are:

kz
xt = x (4.21)
kx

The equivalent transformed hydraulic conductivity, as shown in equation 4.22 below,


is k':

k' = k x k z (4.22)

The flow net on the transformed cross-section may now be constructed in the normal
way, and all calculations, like flow rate and pore water pressure, done as before.

Example 4.9

Assume the same cross-section as for problem 7.6 (p. 177*) in the prescribed book,
but with kz = 6.4 × 10-6 m/s and kx = 1 × 10-5 m/s. Draw the flow net on the transformed
section and calculate the rate of seepage.

86 GEE2601/1
Solution

Transform the cross-section to the equivalent isotropic one by means of equation 4.21:

kz 6,4 × 10 −6
xt = x = x× = 0,8 x
kx 1,0 × 10 −5

All z dimensions retain their original values. The only x dimension on the cross-section
is the width of the concrete weir.

Note:

• It is actually not the weir that is transformed but the 29.5 m section of soil below it.

Transformed width = 0.8 x original width = 0.8 × 29.5 = 23.6 m. We have left this as
an exercise for you to draw the transformed cross-section and add the flow net. The
equivalent hydraulic conductivity is calculated from equation 4.22:

k' = k x k z = 6,4 × 10 −6 × 1,0 × 10 −5 = 8 × 10 −6 m / s

The flow rate can now be calculated from equation 4.17:

Nf N
q = kH = 8 × 10 −6 × 8,3 × f m 3 / s
Nd Nd

4.14 REQUIREMENTS OF HYDRAULIC FILTERS

Filters are often used in drainage, water-pumping operations and in the construction
of earth dams. The function of the filter material is to prevent soil particles from being
carried away by the flowing water into pipes or into the pores of adjacent coarse
materials. In this way pumping equipment is protected and internal erosion of the soil
is prevented. Filters may be constructed from natural materials like sand, or from
layers of artificial materials, commonly known as geofabrics.

To perform its duty, a filter material has to have seemingly contradicting properties. Its
hydraulic conductivity should be relatively high to allow easy seepage, and natural
materials must therefore be coarse-grained, or a geofabric must have large openings.

87 GEE2601/1
But its openings must also be small enough to prevent soil particles from being washed
into or through the filter. A third requirement is that the filter material itself should not
be carried away. For the last reason a filter between the clay core and shell of a rock-
fill dam is often constructed in several layers of increasing particle size, each layer
acting as a filter to the previous one.

There are many design practices for filters of natural materials, but the following
requirements seem to be as efficient as any:

• The D15 size of the filter material should be between four times the D15 size of the
soil, or material to be filtered, and four times the D85 size of the soil.
• The grading curve of the filter material should have approximately the same shape
as that of the soil.
• If pumping equipment is involved, the D85 size of the filter material should be not
less than twice the pipe diameter or screen-mesh.

4.15 SELF-ASSESSMENT ACTIVITIES

You now have the opportunity to apply what you have learnt in this unit. Answer the
following questions:

Activity 4.1: Prescribed book exercises

1. Read the summary in section 6.9 (p. 157*).


2. Read the summary in section 7.6 (p. 175*).
3. Do problems 6.1 to 6.9 (pp. 157-159*) and consult the solution book for
answers.
4. Do problems 7.1 to 7.6 (pp. 175-177*) and consult the solution book for
answers.
5. Work through examples 6.3 and 6.4 (pp. 145-146*) without looking at the
calculations. See if you get the same answers. Note that none of these examples
mention temperature, which means that the temperature may be assumed as 20
°C and no correction is required.

88 GEE2601/1
6. Attempt problems 6.3 to 6.4 (p. 158*) at the end of chapter 6 . Again, no
temperature correction is required.
7. Do problems 6.10 and 6.11 (p. 160*) at the end of chapter 6.
8. Work through example 6.5 (p. 149*).
9. Use the geometry of the concrete wall in example 7.1 (pp. 170-171*) but add a
2 m deep sheet pile that protrudes vertically downwards into the sand from the
centre of the foundation. Draw the flow net, calculate the seepage and determine
the pore water pressure at the same position E. Observe how the flow net and
the calculated values are changed by the presence of the sheet pile.
10. Redo the previous problem but move the sheet pile to a position near the
upstream or the downstream face of the wall, keeping it vertical downwards. A
different length may be used this time.
11. Do problem 7.7 (p. 178*) and consult the solution book for the answers.

Activity 4.2: Multiple-choice questions

1. The discharge velocity of water …

(a) is the quantity of water flowing in unit time through a unit gross cross-
sectional area of the soil.
(b) is the quantity of water flowing in unit time through a unit gross cross-
sectional area of the soil at right angles to the direction of flow.
(c) has the SI unit as m/s.
(d) Both (b) and (c).

2. Select the incorrect statement.

(a) Elevation head at a given point is its vertical distance above or below a
datum plane.
(b) Pressure head at a given point is the water pressure at this point divided
by the unit weight of water.

89 GEE2601/1
(c) The SI unit of head is m.
(d) None of the above.

3. For the flow of water through a porous soil medium, the total head at any point
can be adequately represented by the sum of …
(a) elevation head and pressure head.
(b) elevation head and velocity head.
(c) pressure head and velocity head.
(d) None of the above.

4. The hydraulic gradient is


(a) dimensionless.
(b) a ratio of head loss between two points to the distance between these
points.
(c) Both (a) and (b).
(d) similar to the velocity gradient.

5. In which of the following flow zones does the discharge velocity bear a linear
relationship to the hydraulic gradient?
(a) laminar flow zone
(b) transition flow zone
(c) turbulent flow zone
(d) All of the above.

6. For the flow of water through the void spaces of most saturated soils, discharge
velocity is
(a) proportional to the hydraulic gradient.
(b) proportional to the square of the hydraulic gradient.
(c) inversely proportional to the hydraulic gradient.
(d) inversely proportional to the square of the hydraulic gradient.

90 GEE2601/1
7. Darcy's law relates the discharge velocity (v) to the hydraulic gradient (i) and the
hydraulic conductivity (k) as …
(a) v = k/i.
(b) v = i/k.
(c) v = ki.
(d) None of the above.

8. The SI unit of hydraulic conductivity is …


(a) m.
(b) m/s.
(c) m/s2.
(d) m/s3.

9. The actual velocity of water through the void spaces of soil is called …
(a) flow rate.
(b) discharge velocity.
(c) seepage velocity.
(d) seepage pressure.

10. If discharge velocity of water through a soil is 24 cm/hr and soil porosity is 30%,
the seepage velocity will be …
(a) 24 cm/hr.
(b) 72 cm/hr.
(c) 80 cm/hr.
(d) None of the above.

11. The hydraulic conductivity of soil does not depend on …


(a) unit weight of soil solids.
(b) unit weight of water flowing through the void spaces of the soil.

91 GEE2601/1
(c) viscosity of water flowing through the void spaces of the soil.
(d) void ratio of the soil.

12. The hydraulic conductivity of soil is …


(a) proportional to the unit weight of water flowing through the soil.
(b) inversely proportional to the unit weight of water flowing through the soil.
(c) inversely proportional to the viscosity of water flowing through the soil.
(d) Both (a) and (c).

13. The hydraulic conductivity of fine sand typically ranges from …


(a) 100 to 1 cm/s.
(b) 1.0 to 0.1 cm/s.
(c) 0.01 to 0.001 cm/s.
(d) 0.001 to 0.0001 cm/s.

14. The absolute permeability of soil is expressed in …


(a) m.
(b) m2.
(c) m/s.
(d) m3/s.

15. The laboratory test used for determining permeability of fine-grained soils is
the …
(a) falling head permeability test.
(b) constant head permeability test.
(c) pumping test.
(d) All of the above.

92 GEE2601/1
16. For fairly uniform sand, the hydraulic conductivity k (in cm/s) is related to the
effective size D10 (in mm) of the soil particles as … where c is a constant that
varies from 1.0 to 1.5.
(a) 𝑘𝑘 = 𝑐𝑐𝐷𝐷10
2
(b) 𝑘𝑘 = 𝑐𝑐𝐷𝐷10
(c) 𝑘𝑘 = 𝑐𝑐/𝐷𝐷10
2
(d) 𝑘𝑘 = 𝑐𝑐/𝐷𝐷10

17. The hydraulic conductivity k of soil is related to its void ratio e as …


𝑒𝑒
(a) 𝑒𝑒 ∝ 1+𝑒𝑒.
𝑒𝑒 2
(b) 𝑒𝑒 ∝ 1+𝑒𝑒.
𝑒𝑒 3
(c) 𝑒𝑒 ∝ 1+𝑒𝑒.
𝑒𝑒 4
(d) 𝑒𝑒 ∝ 1+𝑒𝑒.

18. If the hydraulic conductivity of sand at a void ratio of 0.6 is 0.03 cm/s, its value
at the void ratio of 0.75 will be …
(a) 0.03.
(b) 0.05.
(c) 0.07.
(d) None of the above.

19. If a soil deposit consists of two layers with a thickness of 1 m and 1.5 m with
hydraulic conductivity values of 3 × 10-3m/s and 5 × 10-4m/s, respectively, the
equivalent hydraulic conductivity of the soil deposit in the horizontal direction
will be …
(a) 1.5 × 10-3 m/s.
(b) 1.5 × 10-4 m/s.
(c) 7.5 × 10-4 m/s.
(d) 7.5 × 10-3 m/s.

93 GEE2601/1
Activity 4.3: Typical exam questions

1. A pumping well test was made in sands extending to a depth of 15 m where an


impermeable stratum was encountered. The initial groundwater level was at the
ground surface. Observation wells were sited at distances of 3 and 7.5 m from
the pumping well. A steady state was established at about 20 hours when the
discharge was 3.8 l/s. The drawdowns at the two observation wells were 1.5 m
and 0.35 m. Calculate the coefficient of permeability.

2. A layer of sand 6 m thick lies beneath a clay stratum 5 m thick and above a bed
of thick shale. To determine the permeability of sand, a well was driven to the top
of the shale and water pumped out at a rate of 0.01 m3/sec. Two observation
wells were driven through the clay at 15 m and 30 m from the pumping well and
water was found to rise to levels of 3 m and 2.4 m below the groundwater surface.
Calculate the permeability of soil.

3. For the flow net shown below in figure 4.21, do the following:

(a) Calculate the uplift force at the base of the weir, per foot of length points A
and B at the corners of the concrete dam.

(b) Calculate the driving head at lines 5 and 12.

94 GEE2601/1
FIGURE 4.21
Flow net

4. For the flow net shown in figure 4.22 below, calculate the following:

(a) Flow rate per unit length


(b) Uplift force per unit length under the dam
(c) Effective stress at points a and b
(d) The seepage loss for 8 m length of sheet pile (additional)
(e) Pressure head at point c (from ground surface) (additional)
(f) Total head at point a (additional)
(g) The exit gradient (additional)

(k = 1 × 10−5 m/s, Ɣsat = 19 kN/m3)

95 GEE2601/1
FIGURE 4.22
Flow net

4.16 SOLUTIONS TO SELF-ASSESSMENT ACTIVITIES

Activity 4.1: Prescribed book exercises

3. See prescribed solution book.


4. See prescribed solution book.
5. See prescribed book for solutions.
6. See prescribed solution book.
7. See prescribed solution book.
8. See prescribed solution book.
9. See prescribed solution book.
10. See prescribed solution book.
11. See prescribed solution book.

96 GEE2601/1
Activity 4.2: Multiple-choice questions

1. (d)
2. (d)
3. (a)
4. (c)
5. (a)
6. (a)
7. (c)
8. (b)
9. (c)
10. (c)
11. (a)
12. (d)
13. (c)
14. (b)
15. (a)
16. (b)
17. (c)
18. (b)
19. (a)

Activity 4.3: Typical exam questions

1. 0.123 m/hr

2. 1.1 m/hr

3. (a) 188 806.8 fIb/ft; (b) 3 ft

4. (a) 2.668 × 10−5 m3/s.m; (b) 2 067.38 kN/m2; (c) 106.42 kN/m2; (d) 21.344 × 10−5
m3/s; (e) 7.99 m; (f) 22.333 m; (g) 0.741

97 GEE2601/1
4.17 SUMMARY

On completion of this study unit, you should be able to determine the hydraulic
conductivity of soil by means of laboratory or field tests, calculate flow rates in natural
soil layers and draw flow nets for two-dimensional isotropic seepage conditions in the
vertical plane in confined or unconfined situations. In addition, you should also be able
to perform calculations of seepage rates and pore water pressures within the soil, and
transform anisotropic cross-sections to their isotropic equivalent.

Since seepage is an important factor in the stability of almost all structures built from,
on, or in soil, your acquired knowledge will be of great value for the remainder of this
module (GTE3601), and also in your future career.

4.18 VIDEO CLIPS, SLIDES AND OTHER WEB-BASED


RESOURCES

For additional explanations on movement of water through soil, you can watch some
video clips. Web addresses/links to the video clips are available on the module site
under Additional Resources in a folder named Video clips, slides and other web-
based resources.

• Water movement in soil:


https://www.youtube.com/watch?v=vmo0FRAVgkM
• Saturated hydraulic conductivity: double ring infiltrometer:
http://hydropedologie.agrobiologie.cz/en-dvouvalec.html
• Bernoulli's equation:
https://www.youtube.com/watch?v=ytCuHh5PwwY
• Fluid Flow in the Subsurface (Darcy's Law):
https://fracfocus.org/groundwater-protection/fluid-flow-subsurface-darcys-law
• Seepage:
http://slideplayer.com/slide/7732863/

98 GEE2601/1
• Seepage and flow net:
http://slideplayer.com/slide/6397614/
• Flow of water through soil:
people.eng.unimelb.edu.au/stsy/geomechanics_text/Ch5_Flow.pdf

REFERENCE WORKS AND FURTHER READING

Das, BM. 2010. Principles of geotechnical engineering. 7th ed. Connecticut: Cengage
Learning.

Das, BM. 2013. Fundamentals of geotechnical engineering. 4th ed. Connecticut:


Cengage Learning.

Das, BM & Sivakugan, N. 2017. Fundamentals of geotechnical engineering. 5th ed.,


international edition. Connecticut: Cengage Learning.

Das, B.M. & Sobhan, K. (2014). Principles of Geotechnical Engineering, 8th Edition,
SI Edition. CENGAGE Learning. ISBN 13: 9788131521588 Connecticut, United
States of America.
Hazen, A. 1930. Water supply. American civil engineers' handbook. New York: Wiley.

Tavenas, F, Jean, P, Leblond, P & Leroueil, S. 1983. The permeability of natural soft
clays. Part II: Permeability characteristics. Canadian Geotechnical Journal,
20(4):645-660.

* Page, figure, table and chapter numbers may differ in updated and revised versions of the prescribed
book. See online version of study guide for updates.

99 GEE2601/1
Notes

....................................................................................................................................

....................................................................................................................................

....................................................................................................................................

....................................................................................................................................

....................................................................................................................................

....................................................................................................................................

....................................................................................................................................

....................................................................................................................................

....................................................................................................................................

....................................................................................................................................

....................................................................................................................................

....................................................................................................................................

....................................................................................................................................

....................................................................................................................................

....................................................................................................................................

....................................................................................................................................

....................................................................................................................................

....................................................................................................................................

....................................................................................................................................

....................................................................................................................................

....................................................................................................................................

....................................................................................................................................

100 GEE2601/1
Study unit 1 5

STRESSES IN SOIL MASS

5.1 INTRODUCTION

Stress plays a prominent role in the behaviour of all civil engineering structures, and
this includes structures consisting of volumes of soil. Soil cannot sustain tensile normal
stress because of its particulate nature, with the result that only compressive normal
stress and shear stress are normally studied in courses similar to this one. These two
types of stress are equally important to geotechnical engineers and technicians. The
bearing capacity of all types of foundations and the stability of natural and human-
made slopes are governed by the soil's ability to resist shear stress. Normal stress is
important in the calculation of strain, which determines the amount of movement or
displacement that a foundation experiences. In addition, a soil's resistance to shear
stress is governed largely by the normal stress, so these two types or components of
stress are not independent. In this study unit we will concentrate on compressive
normal stress acting perpendicular to the horizontal plane in the soil. This is commonly
referred to as vertical stress and is especially important because it governs
settlements of foundations.

5.2 DEFINITIONS

You will come across the following terms in this study unit:

• Effective stress: The effective stress within a soil sample is equal to the total
stress minus the pore pressure.

101 GEE2601/1
• Neutral stress: The stresses transmitted by a fluid that fills the voids between
particles of a soil.
• Total stress: Total stress (σ) is equal to the sum of effective stress (σ') and pore
water pressure (u).

5.3 LEARNING OUTCOMES

On completion of this study unit, you should be able to distinguish between total normal
stress, effective normal stress and neutral stress, and determine the effects of gravity,
seepage and a variety of applied surface loads on the vertical compressive normal
stress at any position in a soil profile. More specifically, you should be able to

• explain the concepts of effective stress and total normal stress and distinguish
between effective normal stress and neutral stress
• calculate vertical effective normal stress caused by self-weight of the soil
• determine the effects of gravity, seepage and a variety of applied surface loads on
vertical compressive normal stress at any position in a soil profile
• determine the increase in vertical compressive stress at some depth below the
surface of a soil mass as a result of various applied loads at the surface, using the
theory of elasticity

To achieve the learning outcomes in this unit, work through the following topics:

• the concepts of total and effective stress


• the calculation of vertical effective normal stress caused by the self-weight of the
soil
• the effects of vertical seepage on effective normal stress
• the increase in vertical compressive stress at some depth below the surface of a
soil mass as a result of various applied loads at the surface, using the theory of
elasticity

102 GEE2601/1
5.4 PRINCIPLE OF EFFECTIVE STRESS

Chapter 8 (pp. 179-210*) in the prescribed book covers this study unit. Figure 8.1(a)
(p. 181*) in the prescribed book shows a soil column that consists of soil of saturated
unit weight γsat. Above the soil there is some free water of unit weight γw. If you look at
the position marked (A) on the figure, it is clear that the vertical stress across the wavy
plane is given by equation 8.1 (p. 180*) in the prescribed book. The vertical stress is
caused by the weight of all matter, in this case saturated soil and water, above (A) and
it is carried in part by the pore water pressure, or neutral stress, working on the wavy
plane. The rest of the total stress is carried by the solid particles at their points of
contact. The sum of all the vertical components of the particle contact forces per unit
cross-sectional area is called the effective stress, σ'. Mathematically this may be
expressed as

• σ = σ'+u (5.1)
Where:

• σ' = Effective tress


• σ = Total stress
• u = Pore water pressure

This relationship is known as Terzaghi's principle of effective stress.

During extensive research, Terzaghi (1936) found that the compressive strain and
shear strength of soil were proportional to the effective stress and not to the total
stress, and it was only after his formulation of the principle of effective stress that soil
mechanics, or geotechnical engineering, developed into an engineering science.
Before that, people relied considerably on past experience when foundations and soil
structures were designed and constructed. The value of u in equation 5.1 is calculated
from Bernoulli's law, using the water surface as datum, as discussed in study unit 4.
You should realise that the calculations would have been the same had the free water
surface existed within the soil in the form of a water table. The example that follows
demonstrates this, and so does example 8.1 (pp. 182-183*) in the prescribed book.

103 GEE2601/1
Don’t confuse effective stress with interparticle contact stress. The sum of the areas
of all the mineral contact points is very small relative to the total cross-sectional area,
and the average contact stress is much higher than σ'.

Example 5.1

A soil column consists of a 4 m layer of sand on top of clay. The water table is at 1 m
below the surface. Above the water table the sand is moist with a bulk unit weight of
16 kN/m3, and below the water table the sand has a saturated unit weight of 20 kN/m3.
The clay has a saturated unit weight of 21 kN/m3. Assume that the unit weight of water
is 10 kN/m3 and calculate the total vertical stress, neutral stress and effective vertical
stress at position A, which is 6 m below the surface.

Solution

The total vertical stress is determined by all matter above A: 1 m of moist sand, 3 m
of saturated sand and 2 m of saturated clay. The water in saturated soil is considered
part of the soil and not a separate material.

• σ = (1 × 16) + (3 × 20) + (2 × 21) = 118 kN/m2

Neutral pressure, or pore water pressure, is governed by Bernoulli's equation. Take


the datum to be at the water table level, so that total head is zero everywhere since
there is no flow and no potential differences. It should be clear that

• u = 5 × 10 = 50 kN/m2
From equation 5.1

• σ' = σ - u = 118 – 50 = 68 kN/m2

Alternative solution

The same result may be obtained by calculating effective stress directly from buoyant
unit weights of submerged materials as suggested by equation 8.5 (p. 182*) in the
prescribed book. In this way

• σ' = 1 x 16 + 3 x (20 – 10) + 2 x (21 – 10) = 68 kN/m2

104 GEE2601/1
The material, which includes both the solid and liquid phases, below the water table
experiences an uplift force just as a person in a swimming pool does, and this reduces
effective stress to below total stress. The water pressure, u, carries the weight of a
volume of water equal to the submerged volume of material.

5.5 STRESSES IN SOIL WITH SEEPAGE

The principle of effective stress applies whether the water in the soil is stationary or
moving. In section 5.4 this was derived and demonstrated with stationary water. In this
section we will investigate the effect of seepage. In example 5.1 we mentioned that a
submerged volume of material experiences uplift just as a person or object in a
swimming pool does. This uplift works against gravity and reduces the effective stress
within the volume. You can now see that if water moves through a porous material,
the solid phase, or mineral grains in the case of a soil, experiences an additional force,
caused by the frictional resistance between water and solids. The same effect is
experienced by a person or object floating in a stream. The object is moved along with
the stream, and needs an anchor force to keep it stationary. In a porous medium this
force, if expressed per unit volume, is called the seepage pressure, and acts in the
direction of seepage as given by the flow lines in a flow net.

Read through section 8.3 (pp. 183-189*) in the prescribed book, which covers this.
From the discussions of flow nets in study unit 4, you should be able to calculate the
pore water pressure, or neutral pressure as it is also known, by the application of
Bernoulli's law at any position in a soil volume. Since water pressure at a point is the
same in all directions, it can be subtracted from the total vertical stress at the point to
obtain the effective vertical stress. We will investigate methods of calculating the total
vertical stress below a structure, like a weir, later in this study unit.

Of special importance to the geotechnical engineer and technician is the case where
the direction of seepage is vertically upward, such as next to a sheet pile, because this
may lead to instability. Figure 8.3(a) (p. 184*) in the prescribed book shows a situation
where upward flow in soil is caused by the water entering the tank from below, while

105 GEE2601/1
a constant head is maintained at the top by the free water surface. The free surface is
used as datum and the total heads at A and B are given by the water levels in the
standpipes.

At A the total stress and the water pressure are caused by the weight of the water
above A, so the effective stress is zero. At B the total stress is caused by the weight
of the column of water plus the column of saturated soil, and the water pressure is
given by the water level in the standpipe relative to B:

• σB = H1γw + H2γsat
• uB = (H1 + H2 + h)γw

Where the symbols are defined in figure 8.3(a) (p. 184*) in the prescribed book so
that

σB' = σB – uB = H1γw + H2γsat - (H1 + H2 + h)γw

= H2γsat - (H2 + h)γw

= (H2γsat - H2γw) - hγw

= H2γ' - hγw (5.2)

Where:

• γ' = Buoyant unit weight of soil

Imagine now that the valve at the bottom of the tank is closed so that flow stops. The
result would be that the water level in the standpipe connected to level B would drop
to the datum. In this purely hydrostatic situation the effective stress at B would be

• σB' = H2γ'

The difference between the stationary and seepage cases is clearly given by the
second term in equation 5.2. Upward seepage thus has the effect of reducing effective
stress above the reduction caused by buoyancy. Let's manipulate equation 5.2:

• σB' = H2γ' - hγw = H2γ' – (h/H2) γwH2

106 GEE2601/1
Recalling the definition of hydraulic gradient from chapter 4, and recognising that in
the figure

• i = h/H2

the equation for effective stress at B becomes

• σB' = H2γ' - iH2γw (5.3)

In the same way it can be derived that for point C, which is situated distance z below
the soil surface, effective stress is given by equation 5.6 in the prescribed book:

• σC' = zγ' - izγw (5.4)

In equation 5.4 the seepage pressure would be given by iγw. Seepage pressure,
multiplied by the height of the column of soil, gives the contribution to effective stress
in the same way the buoyant unit weight does, but in the opposite direction due to the
upward flow. For downward flow, the seepage pressure would increase the effective
stress and the negative signs in equations 5.3 and 5.4 would change to positive. It is
clear from equations 5.3 and 5.4 that the seepage pressure is calculated from the
hydraulic gradient in the soil above the point at which effective stress is to be
calculated. When the gradient varies, the average value above the point may be used.

Earlier we mentioned possible instability due to upward seepage. From inspection of


equation 5.4 it is clear that the effective stress may become negative if the hydraulic
gradient becomes too high. A critical value would be reached when the effective stress
is exactly zero:

• σC' = zγ' - icrzγw = 0

So that

γ'
• i cr = (5.5)
γw

Instability may occur in many soils when the hydraulic gradient is equal to or exceeds
the critical value. This instability is visible in the form of “sand boils” at the surface of
coarse-grained materials. The shear strength of soil depends on effective stress, and

107 GEE2601/1
no effective stress means very low or zero shear strength. A blow-out (also called
ground-heave, bottom-heave, or hydraulic fracturing) may occur if the hydraulic
pressure at the bottom of a layer of very low hydraulic conductivity exceeds its weight.

Note:

• Seepage changes effective stresses in the soil by exerting body forces on the soil
mass in the direction of flow. These body forces are similar to those caused by
gravity, but do not modify total stresses. Seepage does have an influence on the
neutral pressures.
• The effect of seepage on effective stress may be taken into account by subtracting
neutral stress (pore water pressure) from total stress, or from the combination of
gravity and seepage pressure, as is illustrated by the development of equation 5.2.

Example 5.2

Consider the flow situation around the sheet pile in figure 7.7 (p. 171*) of example 7.2
(pp. 170-171*) in the prescribed book. The sheet pile was driven to a depth of 5 m into
the 10 m layer of sand. Assume that the saturated unit weight of the sand is 20 kN/m3.
Determine whether there is a possibility of instability of the sand due to the upward
flow.

Solution

Calculate the potential drop between equipotential lines:

• ∆h = H/Nd = (5.00 – 1.67)/6 = 0.555 m

Consider the right-hand side of the pile where seepage is upwards. There are 2.5
potential drops between the bottom of the pile and the surface:

• i = ∆h/L = 2.5 x 0.555/5 = 0.2775

Calculate the critical hydraulic gradient from equation 5.5:

γ' 20 − 9,81
i cr = = = 1,04
γw 9,81

108 GEE2601/1
Since the true gradient is much smaller than the critical value, the effective stress
should remain positive, with no fear of instability occurring.

Example 5.3

A situation of bottom-heave is considered in example 8.3 (p. 188*) in the prescribed


book. The water pressure at the bottom of the clay is governed by the artesian
condition in the confined sand layer and is measured by the water column in the drill
hole, i.e. 3.6 m. The saturated unit weight of the clay is calculated from the given soil
properties as 18 kN/m3 and the clay layer is 9 m thick. How deep can an excavation
into the clay be made before bottom-heave will occur?

Solution

Take an excavation of Hexc. The weight of a column of unit area of clay below the
excavation is W = 18.76 × (8 - Hexc). Bottom-heave will not occur until the water force
at the bottom of the column exceeds its weight, that is, when neutral stress exceeds
total stress and effective stress becomes zero or negative:

• 18 × (9 - Hexc) < 3.6 × 9.81

From which

• Hexc > 7.04 m

Alternative solution

Consider seepage from the sand layer into the excavation, assuming that the
excavation will be kept free of water by pumping. Take the datum at the sand-clay
interface and determine the potentials at the interface and at the bottom of the
excavation:

• At the interface: Z = 0; u/γw = 3.6 m; so that hint = 3.6 m


• At the excavation: Z = 9 - Hexc; u/γw = 0; so that hexc = 9 - Hexc

This drop in potential is over a distance of 9 - Hexc, so that

• i = {3.6 – (9 - Hexc)} / (9 - Hexc) = (Hexc – 5.4)/(9 - Hexc)

109 GEE2601/1
Calculate icr:

• icr = (18,76 – 9,81)/9,81 = 0.9327

Bottom heave will occur as soon as i > icr:

Hexc > 7.04 m

Note:

• The same result is obtained from investigating the bottom-heave and from
employing the critical hydraulic gradient.
• The reason is that both methods are actually determining effective stress at the
point under investigation.
• The second method was rather cumbersome when applied to the example
because of the specific situation in which the potential at the bottom of the
excavation was a function of the depth of excavation.
• In other situations, with constant potentials at both ends of the soil column, the two
methods require the same effort to apply.

5.6 STRESS INCREASE CAUSED BY SURFACE LOADING

Up to now we have considered the stress in the soil caused by its own weight in
combination with seepage effects. The remainder of this study unit will deal with the
stresses caused within the soil by some applied load at or near the earth's surface.
You will see that these stresses are localised, because the area over which the load
is applied is generally not large. An exception to this is when the load is in the form of
a large area fill, which is treated as if it were part of the soil profile. The same applies
to a large area excavation which actually modifies the soil profile.

The stress increase caused by a load is calculated by means of the theory of elasticity.
Now, this is not correct, because soil is not an elastic material. If the load is relatively
small and the soil is not stressed to near to its failure condition, the error is small. In
fact, there is no other way of conveniently calculating the stresses. Another benefit of
assuming linear elasticity is that superposition is allowed. The stresses caused by

110 GEE2601/1
different loads may be added and are in addition to those caused by unit weight and
seepage.

5.6.1 Stress caused by a point load


Section 8.6 (pp. 194-195*) in the prescribed book covers this topic. Boussinesq (1883)
calculated the stresses produced by a point load applied on the surface of a
homogeneous, elastic, infinite half-space. The formulae for the stresses in the different
directions are given by equations 8.17 to 8.20 (pp. 194-195*) in the prescribed book.
Equation 8.20 is especially important to engineers because it gives the vertical stress
necessary to calculate settlements.

Boussinesq's is not the only solution available. Westergaard (1939) derived a similar
set of equations, but he assumed the soil to be infinitely stiff in the horizontal direction,
which means that Poisson's ratio is zero. The modern-day feeling is that the solution
for natural soil deposits is somewhere in the middle of the two, but Boussinesq's
solution seems to be the more popular. Notice that vertical stress is independent of
the material properties. Also, the stress is inversely proportional to the square of the
depth, which means that at the point of application, the stress would tend towards
infinity. This is to be expected for a true point load.

If a load is applied over a small area, and the point at which the stress is to be
determined is not close to the loaded area, it may be regarded as a point load for
purposes of calculation.

5.6.2 Vertical stress caused by a line load


Section 8.7 (pp. 195-196*) in the prescribed book covers this topic. An infinite number
of point loads placed side-by-side form a line load that is flexible in the vertical plane,
that will follow vertical movements of the contact surface. The stress caused by such
an infinitely long line load can therefore be determined by summation or integration of
the stresses from these point loads. The result is equations 8.22 and 8.23 (pp. 195-
196*) in the prescribed book. A very narrow loaded area, such as a track in direct
contact with the earth's surface, may be regarded as a line load. The prerequisite is

111 GEE2601/1
that the track must be very long relative to the depth at which the stress is to be
calculated, to approximate the infinite length, and it must carry the same load per unit
length everywhere.

5.6.3 Vertical stress caused by an infinitely long strip load


A strip load may be regarded as a large number of line loads placed alongside each
other to cover a width B, as shown in figure 5.1 below. Summation, or rather
integration, is again used to determine the cumulative effect of the resulting flexible
loaded strip. The vertical stress is given by equation 5.6. Study this figure carefully
together with the equation to see how the angles β and δ are defined.

q
• ∆σ D = [β + sin β cos(β + 2δ)] (5.6)
π
Where:

• q = Load/unit area

FIGURE 5.1
Strip load (Cengage Learning 2014)

112 GEE2601/1
The numerical values of the angles β and δ require special attention. It is very
important to note that the values must be expressed in radians to be used in the
equation, so it is wise to set your pocket calculator to work in radians before attempting
any calculations. The angles are shown positive in the figure. Now, if A, the point at
which the stress is to be calculated, moves to the left, angle δ between the vertical and
the line connecting A to the nearest edge of the load will become smaller, until it
reaches a value of zero when A is directly below the edge of the strip. Further
movement to the left reduces δ some more, so that its value becomes negative.

The maximum negative value is reached when A is below the centre of the strip, on
the z-axis. After that, δ is measured relative to the other edge and its value increases
again, reaches zero below the edge and becomes positive beyond, just as if the figure
is viewed from the back of the page. If in doubt, use the influence factors in table 5.1
below, but regard x as a dimension relative to the z-axis rather than a coordinate, so
that x is always positive.

TABLE 5.1: Variation of ∆δz/q with 2z/B and 2x/B

2x/B
2z/B
0 0.5 1.0 1.5 2.0
0 1.000 1.000 0.500 - -
0.5 0.959 0.903 0.497 0.089 0.019
1.0 0.818 0.735 0.480 0.249 0.078
1.5 0.668 0.607 0.448 0.270 0.146
2.0 0.550 0.510 0.409 0.288 0.185
2.5 0.462 0.437 0.370 0.285 0.205
3.0 0.396 0.379 0.334 0.273 0.211
3.5 0.345 0.334 0.302 0.258 0.216
4.0 0.306 0.298 0.275 0.242 0.205
4.5 0.274 0.268 0.251 0.226 0.197
5.0 0.248 0.244 0.231 0.212 0.188
(Source: Cengage Learning 2014)

113 GEE2601/1
An interesting aspect of stress below a strip footing is illustrated in figure 5.2 below.
The contour lines, or isobars, show how the stress decreases with distance and depth.
If the strip has a width of B, the stress at a depth of 3B is only 20% of the applied load
per unit area, as shown in figure 5.2(a) below. This means that soil deeper than 3B
will not substantially influence the bearing capacity or settlement of the strip. For a
square loaded area this “20% pressure bulb” is only about 1,5B deep, as shown by
figure 5.2(b) below.

FIGURE 5.2
Contours of equal vertical stress under (a) strip area (b) under square area
(Cengage Learning, 2014)

The vertical stresses caused by a strip load are shown below in figure 5.3. The angles
that are measured in counterclockwise direction are considered positive.

114 GEE2601/1
FIGURE 5.3
Vertical stresses caused by a strip load (Cengage Learning, 2010).

Example 5.4

A 200 kN/m2 flexible strip load is 4 m wide and infinitely long, as shown in figure 5.3.
Calculate the vertical stress caused by the load at a depth of 2 m, at positions A to D.
Note that the positions have been chosen symmetrically on opposite sides of the
centreline in order to illustrate the variation of the sign of the angle δ in equation 5.6.

115 GEE2601/1
FIGURE 5.4
Stress caused by strip load for example 5.4

Solution

Point D in figure 5.4 has the same position relative to the load as point A in figure 5.3.
Note that δD and βD are both positive, since the right-hand side edge of the load is
used as the reference (D). The values of the angles can be determined from the
geometry:

• δD = tan-1(2/2) = 45o = 0,7854 radians


• βD + δD = tan-1(6/2) = 1,2490 radians
• βD = 1,2490 – 0,7854 = 0,4636 radians

116 GEE2601/1
Apply equation 5.6:

q
∆σ D = [β + sin β cos(β + 2δ)] = 200 [0,4363 + sin 0,4363 cos(0,4363 + 2 × 0,7854)]
π π

=16.8 kN/m2

Point C in figure 5.4 is to the left of the reference edge of the load and, as discussed
above, the value of angle δD is negative when used in the equation, while βD remains
positive.

Determine the values of the angles from the geometry:

• δC = tan-1(1/2) = 0.4636 radians (use -0.4636 in the equation)


• βC – δC = tan-1(3/2) = 0.9828 radians
• βC = 0.9828 – 0.4636 = 1.4464 radians

Apply equation 5.6:

q
∆σ C = [β + sin β cos(β + 2δ)] = 200 [1,4464 + sin1,4464 cos(1,4464 + 2 × −0,4636)]
π π

=146.9 kN/m2

When the point at which the stress is to be calculated is to the left of the centreline of
the load, the left-hand side edge of the load should be used as the reference, as
discussed above. This means that at point B the angle δ will be negative, and at point
A it will be positive. Because of the symmetry of the points relative to the centreline of
the load, the stress increase at A will be the same as at D, whereas the stress increase
at B will be the same as at C.

Alternative solution

Calculate the stress at D and A using the coefficients in table 5.1:

• 2x/B = 2.0
• 2z/B = 1.0
• ∆σD = 0.078 x 200 = 15.6 kN/m2

117 GEE2601/1
Calculate the stress at C and B using the coefficients in table 5.1:

• 2x/B = 0.5
• 2z/B = 1.0
• ∆σC = 0.753 x 200 = 150.6 kN/m2

The small differences between the answers obtained from the two solutions were
caused by rounding off of angles and of coefficients in the table.

5.6.4 Vertical stress below the centre of a uniformly loaded circular area

Section 8.7 (pp. 195-196*) in the prescribed book covers this topic. Boussinesq's
solution for the stress below a point load may be integrated over a flexible circular area
to determine the stress at a depth z below the centre, as shown in figure 8.14 (p. 197*)
in the prescribed book. This is an excellent approximation of a liquid-filled reservoir
with a flexible bottom. The stress below the centre is given by equation 8.25 (p. 197*)
in the prescribed book. The influence factors in table 8.3 (p. 198*) in the prescribed
book may be used instead of the equation. Unfortunately, no mathematical expression
has yet been derived for the stress at any other point than below the centre, but some
specialist books have tables of influence factors for points below the edge.

5.6.5 Vertical stress caused by a loaded rectangular area

Section 8.9 (pp. 199-202*) in the prescribed book covers this topic. Integration of
Boussinesq's equation for stress below a point load may also be integrated over a
flexible rectangular area to obtain the stress in the underlying soil. Fadum (1948) came
up with the brilliant idea to choose the integration limits such that the stress below one
of the corners of the rectangle is obtained, as shown in figure 8.15 (p. 199*) in the
prescribed book. The result is the influence factor in the rather complicated equation
8.30 (p. 200*) in the prescribed book. m and n are the ratios of the lateral dimensions,
L and B, of the loaded area to the depth, z, at which the stress is calculated.

To perform calculations using Fadum's influence factor, your calculator must be set to
radians. It is good practice to always correlate your answer with the values from figure
8.15 in the prescribed book, because the equation may get unstable for relatively large

118 GEE2601/1
values of m and n. If a negative influence factor results from the equation, simply add
0.25 to correct it. The value of the influence factor is always between 0 and 0.25.
Fadum's brilliance becomes clear when stresses are required at points not below a
corner of the rectangular area, when different parts of an area carry different loads per
unit area, or when a loaded area is made up of rectangles. Superposition of several
loaded areas is then simply employed as in figure 8.17 (p. 202*) in the prescribed
book. The power of Fadum's method can only be illustrated by means of an example.

Example 5.5
An L-shaped flexible area carries a 300 kN/m2 load. Its dimensions are shown in figure
5.5 below. Calculate the stress increase caused by the load at a depth of 4 m below
points A, B, C, E, I and J.

FIGURE 5.5
L-shaped loaded area for example 5.5

Solution

Since the area is not a rectangle, the principle of superposition will have to be
employed to calculate the stress at the required positions. To aid with the
superposition, the loaded area has been divided into rectangles, using dotted lines, as
you can see in figure 5.6.

119 GEE2601/1
FIGURE 5.6
Application of Fadum's method

First do the calculation for the point 4 m below I which is on the corner of each of the
three loaded rectangles IABC, ICEF and IFGH that make up the L-shaped loaded
area. Since these three rectangles have the same shape and size, they will cause the
same stress increase at 4 m below I. This stress increase will now be calculated for a
rectangle of 4 m × 4 m.

• First calculate m and n: L = B = 4, and z = 4 m, therefore m = n = 1.0.


• From equation 8.30 (p. 200*) in the prescribed book I3 = 0.1752.
• The stress increase caused by one rectangle = ∆σ = 300 × 0.1752 = 52.56 kN/m2.

The stress increase cause by three identical rectangles = ∆σ = 3 × 52.56 = 157.7


kN/m2.

Next, do the calculation for the point 4 m below C on the corner of the 8 m × 4 m
rectangle CEGH, and of the 4 m × 4 m rectangle CIAB. The stress increase will be
calculated for these two rectangles and added to get the total effect.

120 GEE2601/1
• Calculate m and n for CEGH: L = 8 m, B = 4 m, z = 4 m, so that m = 2.0 and n =
1.0.

From equation 8.30 (p. 200*) in the prescribed book I2 = 0.1999. The stress increase
caused by CEGH is ∆σ = 300 × 0.1999 = 59.97 kN/m2. Calculate m and n for CIAB:
L = B = 4 m, and z = 4 m, therefore m = n = 1.0

• From equation 8.30 (p. 200*) in the prescribed book I2 = 0.1752.


• The stress increase caused by CIAB is ∆σ = 300 x 0.752 = 52.560 kN/m2.
• The stress increase at 4 m below C is ∆σ = 59.97 + 52.56 = 112.5 kN/m2.

Now do the calculation for the point 4 m below J. J is not on any corner of the L-shaped
loaded area, but the stress at a depth below J will be affected by the load. The problem
is solved by artificially extending the loaded area by including JAIH, so that it does
have a corner at J, as shown by the dotted lines, and then subtracting the effect of the
load on area JAIH.

The stress increase will be calculated for the extended area JBEG first, and then the
effect of JAIH will be subtracted to get the true effect.

• Calculate m and n for JBEG: L = 8 m, B = 8 m and z = 4 m, so that m = n = 2.0.


• From equation 5.30 in the prescribed book I2 = - 0.0175.
• Add 0.25 to remove the error from the equation to get I2 = 0.2325.
• The stress increase caused at J by JBEG is ∆σ = 300 × 0.2325 = 69.75 kN/m2.
• Calculate m and n for JAIH: L = B = 4 m and z = 4 m, therefore m = n = 1.0.
• From equation 8.30 (p. 200*) in the prescribed book I2 = 0.1752.
• The stress increase caused by CIAB is ∆σ = 300 × 0.1752 = 52.56 kN/m2.
• The stress increase at 4 m below J is ∆σ = 69.75 – 52.56 = 17.2 kN/m2.

To do the calculations for the point 4 m below A, a similar procedure to the one just
described for J is followed. Part IFGH of the loaded area is first extended to include
AIHJ and then the effect of AIHJ is subtracted. This is the only way to include the effect
of IFGH. The answer is 67.4 kN/m2. See if you can get it. The values of I2 that you will
need are listed at the end of the solution.

121 GEE2601/1
The calculation for the point below B is worthy of some discussion. The problem is to
include the effect of the loaded part IFGH. Consider the following method:

• First extend the L-shaped area to an 8 m × 8 m area by including AIHJ (∆σ = 300
× 0.2325 = 69.75 kN/m2).
• The effect of AIHJ cannot be subtracted directly, because it does not have a corner
at B, so subtract the effect of BCHJ (∆σ = 300 × 0.1999 = 59.97 kN/m2) and then
add the effect of BCIA (∆σ = 300 × 0.1752 = 52.56 kN/m2) which was subtracted
as part of BCHJ. The final result then is that ∆σ = 69.75 – 59.95 + 52.56 = 62.4
kN/m2.

Another interesting calculation is for the stress at 4 m below E. The only way of
including the total loaded area in the calculation is to consider rectangles EGHC and
EFAB because each has a corner at E. But, by doing this, the area EFIC is included
twice, causing an error in the result. To remove this error, the effect of EFIC is
subtracted once to get to the correct answer. Do the calculation and check if your
calculation shows that ∆σ = 67.4 kN/m2. Use the values if I3 in table 5.2 below.

TABLE 5.2: Variation of I3 example 5.5

Length = L Width = B Depth = z m n I3


8 8 4 2,0 2,0 0,2325
8 4 4 2,0 1,0 0,1999
4 4 4 1,0 1,0 0,1752

5.6.6 Newmark's chart for vertical stress

Newmark (1942) used the equation for vertical stress below the centre of a loaded
circular area, equation 8.25 (p. 197*) in the prescribed book, to derive a general
formula that can be used to determine the stress below a flexible loaded area of any
shape. He subdivided the circular area into elements, by using radii and concentric
circles, in such a way that each element has the same contribution to the stress below
the centre. The subdivided circular area is called Newmark's influence chart, and an

122 GEE2601/1
example is shown in figure 5.7. You are not required to know how to construct a
Newmark's chart, but you are expected to know how to use it to determine the stress
below a loaded area. A close look Newmark's chart reveals that it has an influence
value, IV, (usually 0,005 or 0,002) and a scale line of length SL. The latter is the
horizontal line in the bottom right (sometimes left) corner of the figure. This scale line
represents the depth z at which the stress is to be determined, and the influence factor
is the contribution from each element to the stress at this depth, for a load intensity of
1 kN/m2. Because of these properties, the chart is more accurate to use.

FIGURE 5.7
Newmark's chart for vertical stress below loaded areas, influence value = 0,005

To use the Newmark chart, the loaded area is drawn onto it to a scale of SL:z and
positioned in such a way that the centre of the chart coincides with the point, in plan

123 GEE2601/1
view, at which the stress is desired. Since each element has the same contribution to
the stress, you merely have to count the number of elements covered by the load and
multiply this number by the influence value and the load intensity, q, to calculate the
stress caused by the loaded area. Fractions of elements must be taken into account
to the best of your ability.

It should be clear that the power of Newmark's method is that the loaded area may
have any shape, no matter how odd or irregular, because the elements covered by
any such area can be counted. Even if different parts of an area have a different load
intensity, the method can still be used because superposition applies. If the stress at
the same depth is desired for a number of points, as in example 5.5 above, you should
draw the loaded area on a piece of transparent paper. You can then move this around
so that each successive point of interest is at the centre, and the elements counted for
each. But when the stress is required at different depths, you need a separate drawing,
to a different scale, for each depth.

Example 5.6

Use the Newmark chart to calculate the stress caused by the L-shaped loaded area
from example 5.5, at a depth of 4 m below point B.

Solution

• The length of the scale line in figure 5.4 is SL = 40 mm.


• The depth of interest is z = 4 m.
• Therefore, the scale of the drawing is 50 mm:4 m, which is 40:4 000 = 1:100.
• Figure 5.8 shows the loaded area drawn to this scale and positioned with B at the
centre of the chart.
• Count the number of elements covered by the loaded area: M = 210 (including
fractions of elements in an approximate way).
• Apply the equation: ∆σ = (IV)qM = 0,001 × 300 × 210 = 63.0 kN/m2.
• This value compares very well with the stress at the same point, 4 m below B,
calculated by Fadum's method in the solution of example 5.5.

124 GEE2601/1
FIGURE 5.8
Application of Newmark's chart, influence value = 0.001

Note:

• Solutions are available for the determination of stress increases in the soil caused
by loads and loaded areas at or near the surface. Specialist textbooks describe
many more solutions than the prescribed book and these study units, including
solutions for horizontal stresses.
• The stress increase at a point is determined by the shape of the loaded area, the
load intensity and distribution, and the depth and position of the point relative to
the loaded area.
• The stress increase caused at a depth of three times the smaller dimension of a
loaded area is usually less than 10% of the surface stress.

125 GEE2601/1
• The theory assumes that the soil is an elastic, homogeneous and semi-infinite
mass and that the loaded area is completely flexible. In practice, the theory is used
even if these assumptions are not really correct to calculate the stress increase in
layered soils and below rigid concrete footings, for example. The error is relatively
small, and there is no other convenient theory for routine calculations.
• Stress increases caused by surface loads are independent of and in addition to
stresses caused by the self-weight of the soil and the effects of seepage.
• Stress increases caused by surface loads are total stresses. Whether these have
an effect on effective stresses and/or neutral stresses (pore water pressures)
depends on the drainage conditions within the soil, as we will discuss in the next
study unit.

5.7 SELF-ASSESSMENT ACTIVITIES

You now have the opportunity to apply what you have learnt in this unit. Answer the
following questions:

Activity 5.1: Prescribed book exercises

1. Read the summary in section 8.10 (pp. 203-204*).


2. Do examples 8.1 to 8.6.
3. Do problems 8.1 to 8.10.
4. Do problem 8.16.
5. Do problems 8.22 to 8.25.

Activity 5.2: Multiple-choice questions

1. For the calculation of foundation settlement, which of the following is estimated?

(a) increase of vertical stress in soil


(b) net increase of vertical stress in soil
(c) increase of pore water pressure
(d) None of the above.

126 GEE2601/1
2. The sum of the vertical components of the forces developed at the points of
contact of the solid particles per unit cross-sectional area of the soil mass is
called … stress.

(a) vertical
(b) total
(c) effective
(d) neutral

3. The effective stress principle for saturated soils provides an expression involving
total stress σ, effective stress σ′ and pore water pressure (or neutral stress) u
as …

(a) 𝜎𝜎 ′ = 𝜎𝜎 + 𝑢𝑢.
(b) 𝜎𝜎 ′ = 𝜎𝜎′ + 𝑢𝑢.
(c) 𝑢𝑢 = 𝜎𝜎′ + 𝜎𝜎.
(d) None of the above.

4. The principle of effective stress was first developed by Karl Terzaghi in …


(a) 1925.
(b) 1936.
(c) 1943.
(d) 1948.

5. Which of the following cannot be determined experimentally?


(a) total stress
(b) pore water pressure
(c) effective stress
(d) All of the above.

127 GEE2601/1
6. The fluctuation of the water table when it remains above ground level causes …
(a) no changes in the total stress and pore water pressure at any point below
ground level.
(b) no changes in the effective stress at any point below ground level.
(c) equal increase or decrease in total stress and pore water pressure at any
point below ground level.
(d) Both (b) and (c).

7. If the groundwater table coincides with ground level and the saturated unit
weight of soil is 19 kN/m3, the effective stress at a depth of 3 m below ground
level will be approximately …
(a) 19 kN/m2.
(b) 28 kN/m2.
(c) 57 kN/m2.
(d) None of the above.

8. In Q. 7, the pore water pressure at a depth of 3 m below ground level will be


approximately …
(a) 10 kN/m2.
(b) 19 kN/m2.
(c) 29 kN/m2.
(d) 57 kN/m2.

9. If the groundwater table fluctuates, but it remains below ground level, the
effective stress at any point below the groundwater table …
(a) increases with a rise of the groundwater table.
(b) decreases with a rise of the groundwater table.
(c) tends to become zero with a rise of the groundwater table.
(d) remains constant with a rise or fall of the groundwater table.

128 GEE2601/1
10. If water is seeping through a soil layer in the vertically upward direction, the
effective stress at any point within the soil …
(a) will be lower than its static case without seepage.
(b) will be higher than its static case without seepage.
(c) may decrease to zero for a specific hydraulic gradient.
(d) Both (a) and (c).

11. For the upward water seepage through a soil mass, boiling or quick condition
occurs when the hydraulic gradient equals … where all the symbols have their
usual meaning.
𝛾𝛾′
(a) 𝑖𝑖𝑐𝑐𝑐𝑐 = 𝛾𝛾
𝑤𝑤
𝛾𝛾𝑤𝑤
(b) 𝑖𝑖𝑐𝑐𝑐𝑐 = 𝛾𝛾′
𝛾𝛾𝑠𝑠𝑠𝑠𝑠𝑠
(c 𝑖𝑖𝑐𝑐𝑐𝑐 = 𝛾𝛾𝑤𝑤
𝛾𝛾𝑑𝑑𝑑𝑑𝑑𝑑
(d) 𝑖𝑖𝑐𝑐𝑐𝑐 = 𝛾𝛾𝑤𝑤

12. For most soils, the critical hydraulic gradient that causes quick condition is
approximately …
(a) 0.
(b) 0.5.
(c) 1.0.
(d) None of the above.

13. The seepage force per unit volume of soil is equal to … where all the symbols
have their usual meaning.
(a) 𝑖𝑖𝛾𝛾𝑤𝑤 .
(b) 𝑖𝑖𝑖𝑖.
(c) 𝑖𝑖𝛾𝛾𝑠𝑠𝑠𝑠𝑠𝑠 .
(d) 𝑖𝑖𝑖𝑖′.

129 GEE2601/1
14. The factor of safety against heave on the downstream side of a hydraulic
structure can be obtained using the concept of …
(a) structural design.
(b) consistency limits.
(c) seepage force.
(d) pore water pressure.

15. The vertical normal stress at a point caused by the point load is …
(a) dependent on Poisson's ratio.
(b) independent of Poisson's ratio.
(c) directly proportional to the point load.
(d) Both (b) and (c).

16. The vertical normal stress caused by a point load of 10 kN acting on the ground
surface at a point 1 m vertically below its point of application is …
(a) 0.
(b) 4.775 kN.
(c) 5 kN.
(d) 10 kN.

17. The vertical stress increase at a depth of 1 m below the centre of the flexible
circular area of 2 m diameter subjected to a pressure of 100 kN/m2 is …
(a) 0.
(b) 29.28 kN/m2.
(c) 64.65 kN/m2.
(d) 91.06 kN/m2.

130 GEE2601/1
Activity 5.3: Typical exam questions

1. For the shown figure below, if the value of Δσz at point “A” is 30 KN/m2, determine
the value of q2.

2. For the shown figure below, calculate the increase in vertical stresses at point A.

131 GEE2601/1
5.8 SOLUTIONS TO SELF-ASSESSMENT ACTIVITIES

Activity 5.1: Prescribed book exercises

2. See prescribed book for solutions.


3. See prescribed book for solutions.
4. See prescribed book for solutions.
5. See prescribed book for solutions.

Activity 5.2: Multiple-choice questions

1. (b)
2. (c)
3. (b)
4. (a)
5. (c)
6. (d)
7. (b)
8. (c)
9. (b)
10. (d)
11. (a)
12. (c)
13. (a)
14. (c)
15. (d)
16. (b)
17. (c)

132 GEE2601/1
Activity 5.3: Typical exam questions
1. q2 = 603 kN/m.
2. Δσz(A) = q × (I3(1) + I3(2) + I3(3) + I3(4) − I3(5))

5.9 SUMMARY

Now that you have worked through this study unit on stresses on soil mass pressure,
you should be familiar with the all-important principle of effective stress and be able to
distinguish between total normal stress, effective normal stress and neutral stress.
You can now determine the effects of gravity, seepage and a variety of applied surface
loads on the vertical compressive normal stress at any position in a soil profile. We
must emphasise again that the stresses caused by loaded areas are in addition to
those from gravity and seepage. Excavations may be treated as negatively loaded
areas. The knowledge that you have acquired in this study unit is a huge step in the
study of this course. It will enable you to fully understand the study unit on the theory
of shear strength, which is based on effective stress, and which will be used in
calculating the stability of slopes, all kinds of foundations and retaining structures. The
ability to calculate stresses caused within the soil by surface loaded areas will be used
in later study units on consolidation and settlement analysis.

5.10 VIDEO CLIPS, SLIDES AND OTHER WEB-BASED


RESOURCES

For additional explanations on stresses in soil masses, you can watch some video
clips. Web addresses/links to the video clips are available on the module site under
Additional Resources in a folder named Video clips, slides and other web-based
resources:

• Stresses in a soil mass and Mohr's circle:


https://www.youtube.com/watch?v=jzxFGuD1UEE
• Short lecture on stress distribution in a soil mass:
https://www.youtube.com/watch?v=oFrvxsBItko

133 GEE2601/1
• Calculation of change in stress - problem 1:
https://www.youtube.com/watch?v=xppMZDWusTA
• Soil Ch 7: Stresses due to loads:
https://www.youtube.com/watch?v=jvyeBp7Fjf0

REFERENCE WORKS AND FURTHER READING

Al-Agha, AS. 2015. Basics of foundation engineering with solved problems. Based on
“Principles of foundation engineering, 7th edition”.
Boussinesq, J. 1883. Application des potentiels a l’étude de l’équilibre et du
mouvement des solides élastiques. Paris: Gauthier-Villars.
Das, BM. 2010. Principles of geotechnical engineering. 7th ed. Connecticut: Cengage
Learning.
Das, BM. 2013. Fundamentals of geotechnical engineering. 4th ed. Connecticut:
Cengage Learning.
Das, BM & Sivakugan, N. 2017. Fundamentals of geotechnical engineering. 5th ed.,
international edition. Connecticut: Cengage Learning.
Das, B.M. & Sobhan, K. (2014). Principles of Geotechnical Engineering, 8th Edition, SI
Edition. CENGAGE Learning. ISBN 13: 9788131521588 Connecticut, United
States of America.
Fadum, RE. 1948. Influence values for estimating stresses in elastic foundations.
Proceedings of the Second International Conference on Soil Mechanics and
Foundation Engineering. Vol. 3, pp. 77-84.
Newman, NM. 1942. Influence charts for computation of stresses in elastic foundation.
University of Illinois Bulletin 338.
Terzaghi, K. 1936. Relation between soil mechanics and foundation engineering;
presidential address. Proceedings of the International Conference on Soil
Mechanics and Foundation, Engineering, 1. Vol. 3, pp. 13-18.
Westergaard, HM. 1939. Bearing pressures and cracks. Journal of Applied Mechanics,
6:A49-A53.

* Page, figure, table and chapter numbers may differ on updated and revised versions of the
prescribed book. See online version of study guide for updates.

134 GEE2601/1
Study unit 1 6

CONSOLIDATION SETTLEMENT

6.1 INTRODUCTION

Settlement is the most common form of foundation failure, and the type and/or size of
a foundation are often determined by settlement rather than bearing capacity
considerations. Excessive settlement of the foundations of buildings, bridges, dams
and other structures may lead to intolerable deformations in the structural elements,
leading to cracking, bending, warping and other unwanted forms of distress. This may
result in anything from increased maintenance costs to severe structural damages,
rendering the structure dangerous or otherwise unsuitable for the function for which it
was designed.

The calculation of settlements of various types of foundations is therefore important to


the geotechnical engineer and technician. Carefully study chapter 9 (pp. 211-267*) in
the prescribed book for a brief description of the mechanisms of compression of soil
layers, which are particle deformation, particle relocation and expulsion of water and
air. It is important to note that settlement of a foundation on clay is considered to occur
in three phases. These are immediate settlement, primary consolidation settlement
and secondary consolidation settlement. Coarse soil behaves quite differently from
clay during compression, and there is no clear distinction between the phases.

In this study unit we will concentrate on primary and secondary consolidation


settlement on clay. The time rate of primary consolidation will be covered too, but
without the mathematical details and derivations of the governing equations.

135 GEE2601/1
6.2 DEFINITIONS

You will come across the following terms in this study unit:

• Consolidation: The process during which a saturated clay layer is compressed by


sustained increased static stress, with expulsion of water and soil particles
rearrangement.
• Elastic settlement (immediate settlement): A process caused by the elastic
deformation of dry soil and of moist and saturated soils without any change in the
moisture content. Elastic settlement calculations are generally based on equations
derived from the theory of elasticity.
• Primary consolidation settlement: A result of a volume change in saturated
cohesive soils because of expulsion of the water that occupies the void spaces.
• Secondary consolidation settlement: A process which is observed in saturated
cohesive soils and is the result of the plastic adjustment of soil fabrics. It is an
additional form of compression that occurs at constant effective stress.

6.3 LEARNING OUTCOMES

On completion of this study unit, you should be able to assess consolidation on clay
soils, interpret the results and determine the various compressibility coefficients of the
clay. More specifically, you should be able to

• explain the consolidation of clay


• calculate the primary and secondary consolidation settlement of a foundation, and
the time required for the process
• use the results of the consolidation test to draw the relationship between the void
ratio and effective stress
• use the results of the consolidation test to determine the pre-consolidation pressure
• compile and interpret the results of the consolidation test that you performed during
the practicum
• use the results of the consolidation test to construct the in situ consolidation curve
and determine the compression and swell indices of the clay

136 GEE2601/1
To achieve the learning outcomes in this unit, work through the following topics:

• one-dimensional consolidometer test


• void ratio pressure plots
• normally consolidated and overconsolidated clay
• pre-consolidation pressure
• construction of the field consolidation curve, compressibility coefficients, mv, Cc,
and swell coefficient, Cs
• calculation of one-dimensional primary consolidation settlement of a foundation
• settlement time plots, coefficient of consolidation, cv
• settlement rate calculations and the effects of radial drainage
• settlements from secondary consolidation
• practical: the consolidometer test must be done in the laboratory and is covered in
the practical modules

6.4 FUNDAMENTALS OF CONSOLIDATION

Chapter 9 (pp. 211-267*) in the prescribed book covers this topic. Consolidation is the
process during which a saturated clay layer is compressed by sustained increased
static stress. During consolidation, density increases and pores are reduced by
expelling some of the pore water. Consolidation must not be confused with
compaction, which is densification of partially saturated soil by dynamic loads that
drive out the pore air only.

Figure 9.2(a) (p. 213*) in chapter 9 in the prescribed book shows a situation where
total stress is increased by the same amount everywhere in a clay layer by a large-
area fill over the surface. In this situation there will be no scope for lateral deformation
of the clay. For the effective stress to increase, there must be some relocation of the
mineral particles into a closer packing, but this is prevented by the virtually
incompressible pore water and the zero lateral deformation. The result is that the pore
water pressure (neutral stress) increases by the same amount that the total stress

137 GEE2601/1
increases, leaving the effective stress unchanged, as illustrated by figure 9.2(b) (p.
214*) in the prescribed book.

Consolidation commences immediately after application of the stress increase. Pore


water slowly migrates into the sand above and below, reducing the pressure head in
the clay. Water leaving the soil pores allows particle relocation, which increases the
effective stress. The increase in effective stress at a point is equal to the pore water
pressure decrease, so the total stress is balanced at all times. The distribution of
excess pore water pressure and elevated effective stress in the clay layer, some time
after consolidation has started, is illustrated by figure 9.2(c) (p. 214*) in the prescribed
book.

After a very long time, enough water has migrated from the pores to allow for sufficient
particle relocation that the effective stress increase can balance the applied total stress
increase. The pore water pressure has now dissipated to the value it had before the
total stress increase, that is, the excess pore water pressure is zero. Figure 9.2(d) (p.
214*) in the prescribed book shows this situation. The clay is said to be in the fully
consolidated state.

Note:

• A total stress increase is initially carried entirely by an equal increase in pore water
pressure, called excess pore water pressure.
• Consolidation dissipates the excess pore water pressure by migration of pore
water. This is a very slow process because of the low hydraulic conductivity of
clay.
• After consolidation the total stress increase is carried entirely by an equal increase
in effective stress.
• It is this increase in effective stress, and the accompanying particle relocation into
a closer packing, that causes the vertical deformation, or consolidation settlement.
• From start to finish, the principle that effective stress plus pore water pressure is
equal to total stress is maintained everywhere in the clay.

138 GEE2601/1
• The clay layer remains saturated at all times. Only some water is squeezed from
the pores to allow for particle relocation.

6.5 LABORATORY CONSOLIDATION TEST

Section 9.3 (pp. 215-218*) in chapter 9 in the prescribed book covers this topic.
Consolidation properties of clay soils are studied in the laboratory in what is called the
consolidometer. Figure 9.3 (p. 216*) in the prescribed book shows a cross-section
through a standard consolidometer. You must know the details of the test procedure
as well as of the consolidometer. In the consolidometer the clay specimen is confined
by a metal ring and a pair of porous stones. The specimen is kept in a saturated
condition throughout the test. The vertical load is applied to the specimen, and the
situation is a perfect model of the loaded clay layer between two sand layers discussed
in the previous section. The load is applied in increments, and the deformation is
recorded against time for a few increments, as well as after 24 hours for each
increment, when there should be no more excess pore water pressure. At the end of
the test the load is decreased again in a few large steps, and then the sample is
removed, dried in the oven and the dry mass recorded. Figure 9.5 (p. 217*) in the
prescribed book as well as figure 6.1 below show a typical plot of deformation against
time for a load increment. The initial compression, primary consolidation and
secondary consolidation stages can clearly be distinguished.

139 GEE2601/1
FIGURE 6.1
Time-deformation plot during consolidation for a given load increment
(Das & Sobhan 2014)

6.6 VOID RATIO PRESSURE PLOTS


Section 9.4 (pp. 218-219*) in chapter 9 in the prescribed book covers this topic. The
height of solids, Hs, is calculated from the dry mass using equation 9.4 (p. 218*) in
the prescribed book. The void ratio at the end of each load increment or decrement
can be calculated from the height of the specimen at the end of the increment using
equation 9.6 (p. 218*) in the prescribed book. It is preferable to use this equation for
each void ratio and not the method of void ratio changes described in equations 9.4
to 9.8 (pp. 218-219*) in the prescribed book.

For each load increment, including the initial unloaded case during which ∆H = 0

• H = H0 - ∆H
• Hv = H – Hs
Hv H − Hs
• e= = (6.1)
Hs Hs

140 GEE2601/1
All the symbols are defined in section 9.4 (pp. 218-219*) in the prescribed book. Since
the load applied to the specimen usually ranges from low to very high, the void ratio is
plotted against the logarithm of the effective stress, using semi-logarithmic graph
paper. Figure 6.2 below and figure 9.7 (p. 219*) in the prescribed book show a typical
plot.

FIGURE 6.2
Typical plot of e against log 𝜎𝜎′ (Das & Sobhan 2014)

Example 6.1

A consolidometer test was performed on an undisturbed specimen of saturated clay.


The consolidometer ring had a diameter of 70 mm and a height of 19 mm. The specific
gravity of the soil was 2.70. The dry mass of the specimen was 101.5 g and the results
in table 6.1 below were obtained from the test. Plot the void ratio against the log of the
effective stress.

141 GEE2601/1
TABLE 6.1: Consolidometer results (example 6.1)

Effective stress (kN/m2) Deformation (mm)


0 0,00
25 0,08
50 0,26
100 0,53
200 0,92
400 1,47
800 2,20
1 600 3,07
200 2,71

Solution
Convert all dimensions to SI units and calculate the height of solids using equation 9.4
(p. 218*) in the prescribed book:

𝑊𝑊𝑆𝑆 0.1015 × 9.81


HS = = 0.009768𝑚𝑚 = 9.768𝑚𝑚𝑚𝑚
𝐴𝐴𝐺𝐺𝑠𝑠 𝛾𝛾𝑆𝑆 0.0702
𝜋𝜋 × × 2.7 × (1 000 × 9.81)
4

Calculate the void ratio for each load increment, using equation 6.1. The void ratio
values are shown in table 6.2 below:

TABLE 6.2: Void ratio calculations (example 6.1)


Effective stress
Deformation (mm) Height (mm) Void ratio
(kN/m2)
0 0.00 19.00 0.945
25 0.08 18.92 0.937
50 0.26 18.74 0.919
100 0.53 18.47 0.891
200 0.92 18.08 0.851
400 1.47 17.53 0.795
800 2.20 16.80 0.720
1 600 3.07 15.93 0.631

142 GEE2601/1
200 2.71 16.28 0.667
25 1.97 17.03 0.743

Plot the void ratio versus logarithm of the effective stress in figure 6.3 below. The plot
consists of two curves, one for loading and one for unloading. Upon unloading, any
clay swells a little, but the greater part of the deformation is permanent.

FIGURE 6.3
Consolidation curve for example 6.1

6.6.1 Normally consolidated and overconsolidated clay

Section 9.5 (p. 220-221*) in the prescribed book covers this topic. If a clay is
completely remoulded and tested in the consolidometer, the e-log σ' relationship will
be a straight line. The clay is said to be normally consolidated, since the present
effective stress is the maximum that it has experienced since remoulding. The void
ratio versus logarithm of the effective stress (e-log σ) relationship is called the virgin
consolidation curve. If the specimen is incrementally loaded up to a predetermined
value of effective stress and then unloaded, the clay will swell and rebound to a certain
extent due to elasto-plastic properties of soil, but most of the virgin deformation is
permanent. The mineral particles will not move back to their original positions. The
average slope of the rebound curve is much flatter than that of the virgin curve.

143 GEE2601/1
Reloading the specimen up to the previous maximum effective stress will result in
recompression to more or less the void ratio that it had at that stress before unloading,
although unloading and reloading will not follow exactly the same e-log σ' curve.
During unloading and reloading the clay is said to be overconsolidated since the
effective stress is less than the previous maximum value. This previous maximum
value is called pre-consolidation stress. If loading is increased above pre-consolidation
stress, σc', the clay is normally consolidated again and the e-log σ' relationship will
follow the extension of the virgin curve.

Undisturbed natural clay tested in the consolidometer will follow more or less the same
e-log σ' relationship as remoulded clay. The relationship may show some curvature at
low stresses because of unloading and disturbance during sampling. A typical
consolidation curve, including unloading and reloading, is shown in figure 6.4 below
and figure 9.8 (p. 220*) in the prescribed book.

FIGURE 6.4
Plot of e against log σ' showing loading, unloading, and reloading
branches (Das & Sobhan, 2014)

144 GEE2601/1
Some natural clay deposits have been overconsolidated by overburden soils that have
been removed over time by erosion or human activity. The typical e-log σ' plot for
overconsolidated clay consists of an almost flat section below the pre-consolidation
pressure, σc'. This section is parallel to the rebound curve. Above σc' is the steep and
straight virgin line. A curved section joins the two. This is typical of overconsolidated
clay as illustrated by the reloading section, dfg, in figure 6.4 above and figure 9.8 (p.
220*) in the prescribed book. On that curve c is at the pre-consolidation pressure, and
the need arises to determine the value of σc' for naturally overconsolidated clay. A
procedure to establish σc' was suggested by Casagrande (1936). It is clearly explained
in figure 9.9 (p. 220*) in the prescribed book, and the accompanying text of section
9.5 (p. 221*). You must be able to perform this procedure.

To establish whether natural clay is overconsolidated, you can compare the present
effective vertical stress calculated from the overburden and the position of the water
table with the pre-consolidation pressure obtained from Casagrande's procedure on
results from a consolidometer test performed on a specimen of the clay. If σc' is higher
than the present stress, the clay is overconsolidated.

Note:

• Clay is said to be normally consolidated if it is currently at the maximum effective


vertical pressure in its history.
• If the present effective overburden pressure is less than what the soil has
experienced in the past, the clay is said to be overconsolidated.
• The ratio of the maximum past effective vertical pressure to the present effective
vertical pressure is called the overconsolidation ratio (OCR).

6.6.2 Constructing the in situ consolidation curve

Section 9.6 (pp. 222-223*) in the prescribed book covers this topic. The consolidation
curve obtained from a standard consolidometer test does not exactly describe the in
situ behaviour of the clay because of sample disturbances and unloading effects. In

145 GEE2601/1
fact, the laboratory curve is somewhere between the in situ curve and the curve for
the completely remoulded clay, as illustrated in figure 6.9 of the prescribed book.

Terzaghi and Peck (1967) proposed a graphical correction to the laboratory


consolidation curve to approximate the e-log σ' behaviour of undisturbed in situ clay.
The procedure for normally consolidated clay is illustrated in figure 6.5 below and
figure 9.10 (p. 222*) in the prescribed book. The clay is at the current overburden
effective stress of σ0' and void ratio e0, and that is where the virgin consolidation curve
should start. You have to assume that the void ratio is not affected by the sampling
process, so e0 is the value calculated from the height of unloaded specimen in the
consolidometer, before water is added. The virgin curve is assumed to meet the
laboratory curve at a void ratio of 0.4e0, because that is more or less the value at which
the virgin and remoulded curves meet. If the clay is overconsolidated, the correction
procedure is slightly different, as shown in figure 6.5 below and figure 9.11 (p. 223*)
in the prescribed book. The in situ curve starts at the point (σ0', e0) and follows the
slope of the laboratory rebound curve to approximate the overconsolidated state, up
to the pre-consolidation pressure, σc'. The virgin compression curve starts at this point
and meets the laboratory curve at 0,4e0. You must be able to apply both these
correction procedures to laboratory curves.

146 GEE2601/1
FIGURE 6.5
Consolidation characteristics of normally consolidated clay of low to
Medium sensitivity (Das & Sobhan, 2014)

FIGURE 6.6
Consolidation characteristics of overconsolidated clay of low
to medium sensitivity (Das & Sobhan, 2014)

147 GEE2601/1
6.7 PRIMARY CONSOLIDATION SETTLEMENT CALCULATIONS

6.7.1 One-dimensional consolidation

Sections 9.7 to 9.8 (pp. 223-227*) in the prescribed book cover this topic. The results
from the corrected consolidation curves may be used to calculate one-dimensional
settlements of clay layers. The obvious way would be to read off the void ratio that the
clay would have at the final effective stress (overburden plus increment from loading),
subtract the initial void ratio, e0, and use the change in void ratio to determine the
volume change, using the phase relationships. Since the settlement is one-
dimensional, as was the compression of the specimen in the consolidometer, the
volume change can be converted to settlement.

∆e
• S=H (6.2)
1 + e0
Where:

• S = Settlement
• H = Thickness of the clay layer

If the stress increment is small, a linear relationship between void ratio and effective
stress may be assumed, and the coefficient of volume compressibility, mv, is defined
as

1  e0 − e1  1  ∆e 
mv =  =  
1 + e0  σ − σ  1+ e
' '
 ∆σ 
•  1 0  0
(6.3)

Combining equations 6.2 and 6.3 results in

• S = mv ∆σ' H (6.4)

The value of mv is determined from the corrected consolidation curve using equation
6.3 and the settlement is calculated using equation 6.4. Separate values for mv may
be determined for the normally consolidated and overconsolidated parts of the stress
increments. Care must be taken because mv may change appreciably over larger
stress increments and also with depth in the compressing layer. A better way would

148 GEE2601/1
be to use the constant slope, Cc, of the virgin consolidation line. Cc is defined as the
change in void ratio for a tenfold increase in effective pressure, or

e0 − e1 ∆e
• Cc = = (6.5)
σ' + ∆σ' σ'
log 0 ' log '1
σ0 σ0

The settlement may then be calculated by using equation 9.16 (p. 224*) in the
prescribed book. Even if the slope of the virgin curve is constant, and so is Cc, you
should not apply the method on a thick clay layer because the value of the overburden
pressure appears in equation 9.16 (p. 224*) in the prescribed book, and it increases
with depth. Rather divide the layer into a number of sub-layers, determine the values
of σ0, σ1 and e0 for each, and add the settlements. In fact, it is preferable to obtain and
test a specimen from the middle of each sub-layer in the consolidometer to allow for
vertical variation in the clay.

If the clay is overconsolidated, the final effective stress, σ1' may or may not exceed the
pre-consolidation value σc'. If σ1' < σc' the average slope, Cs, of the rebound curve
must be used for settlement calculation as in equation 6.16 of the prescribed book. Cs
is calculated in the same way as Cc. If σ1' > σc' the clay is going to compress according
to both the rebound curve and the virgin curve, and equation 9.17 (p. 225*) in the
prescribed book applies. The first term in the equation gives the settlement caused by
the stress increment from σo' to σc', whereas the second term gives the settlement
caused by the increment from σc' to the final stress, σ1' = σo' + ∆σ'.

A number of empirical equations for Cc and Cs have been proposed in research papers
over the years. A few of these can be found in sections 9.8 and 9.9 (pp. 225-235*) in
the prescribed book. Empirical relationships should always be used with caution, and
only for preliminary settlement calculations. Any design calculations should be based
on results from careful laboratory consolidometer testing on sound, undisturbed
specimens.

149 GEE2601/1
Example 6.2

Refer to example 9.3 (pp. 229-230*) in the prescribed book. The settlement of the
layer of clay is to be calculated.

Solution

Since the clay layer is relatively thin, it does not have to be subdivided. The initial
stress at the centre of the layer is calculated from the overburden as being 87.74
kN/m2. Note that the buoyant unit weight of the materials below the water table is used
because effective stress is required. The initial stress is less than the
overconsolidation pressure of 125 kN/m2, but when the stress increase of 50 kN/m2 is
added, the final stress will exceed 125 kN/m2. For part of the settlement the clay will
be in the overconsolidated state, and for the rest it will be normally consolidated.
Equation 9.19 (p. 225*) in the prescribed book will have to be used. The indices of
compressibility and rebound have to be estimated from the empirical relationship with
the liquid limit, rendering Cc = 0.27 and Cs = 0.045. The result is that the consolidation
settlement is equal to 62.3 mm.

6.7.2 Consolidation settlement of foundations

The preceding sections dealt with consolidation settlement of clay layers deforming
one-dimensionally. This would be an excellent model for a relatively thin layer of clay
close to a surface over which a large-area load is constructed. The consolidometer
test also models this perfectly. If the loaded area is small, like the foundation below a
column or wall, there are three-dimensional effects, like lateral deformation, and stress
increments that vary with depth and in the horizontal direction. The lateral deformation
allows particle relocation, with the result that there is an instantaneous increase in
effective stress. Since ∆σ' in equation 9.3 (p. 217*) in the prescribed book is not zero
like in the laterally confined situation, a smaller increase, ∆u, in pore water pressure
occurs. The increase in effective stress leads to immediate settlement, whereas the

150 GEE2601/1
dissipation of the lower excess pore water pressure results in smaller consolidation
settlement than predicted by the one-dimensional theory.

Skempton and Bjerrum (1957) proposed a correction factor, µ, by which the one-
dimensional settlement should be multiplied to account for the effects of reduced
excess pore water pressure. The value of µ is determined by many factors, the most
important of which are the overconsolidation ratio and the size and shape of the
foundation. µ may have the following ranges shown in table 6.3:

TABLE 6.3: Correction factor, µ

Type of soil Correction factor (µ)


Very soft clays and sensitive clays 1.0 to 1.2
Normally consolidated clays 0.7 to 10
Lightly overconsolidated clays 0.5 to 0.7
Heavily overconsolidated clays 0.2 to 0.5

The consolidation settlement of a footing, or other smallish foundation, is then


calculated by

• S = µSoed (6.6)
Where:

• Soed = One-dimensional settlement calculated from the consolidometer results


To correct this, consolidation settlement must be added to the immediate settlement.

Example 6.3

A footing 6 m × 6 m carrying a 160 kN/m2 pressure is located on top of a 15 m thick


clay layer that rests on solid rock. From consolidometer tests the average value of mv
was determined as 0.125 × 10-2 m2/kN. Since the clay is normally consolidated, a value
of 0.8 is assumed for µ. Determine the consolidation settlement of the centre of the
footing, assuming that it is flexible.

151 GEE2601/1
Solution

To take into account the variation of stress increase with depth, subdivide the clay into
5 layers of 3 m each. Then calculate the stress below the centre of the footing by
Fadum's method for the corner of a 3 × 3 m2 footing and multiply by 4. Calculate the
consolidation settlement by means of equation 6.4.

TABLE 6.4: Calculations (example 6.3)

Depth = ∆σ' Soed


Sublayer m and n I2
z (m) (kN/m2) (m)
1 1.5 2.00 0.233 149 0.058
2 4.5 0.67 0.121 78 0.030
3 7.5 0.40 0.060 38 0.015
4 10.5 0.29 0.033 21 0.008
5 13.5 0.22 0.021 13 0.005

The one-dimensional consolidation settlement below the centre of the footing is the
sum of the settlements of the individual layers and amounts to 0.116 m, or 116 mm.
To correct for the three-dimensional effect, multiply by µ = 0.8:

• S = 0.8 × 116 = 93 mm

6.8 SECONDARY CONSOLIDATION

The definition of primary consolidation states that the rate of compression is governed
by the rate of pore water dissipation. This, in turn, is governed by the hydraulic
conductivity of the clay. During secondary consolidation the rate is controlled by slow
changes in the clay fabric as mineral particles relocate themselves under the action of
constant effective stress. Although water is still being pushed from the diminishing
pores, the compression rate is independent of hydraulic conductivity.

152 GEE2601/1
Figures 9.5 (p. 217*) and 9.18 (p. 234*) in the prescribed book show typical curves of
deformation and void ratio of clay with time (note the log scale). The slope of the
secondary consolidation curve is Cα, as defined in equation 9.27 (p. 233*) in the
prescribed book. The settlement of a clay layer is calculated from

CαH t
• Ss = log 2 (6.7)
1 + ep t1

Where:

• Ss = Secondary consolidation
• H = Thickness of the clay layer
• ep = Void ratio at end of primary consolidation (figure 9.18, p. 234*) in the
prescribed book)
• t1 = Time at end of primary consolidation
• t2 = Time at end of secondary consolidation

Although performing the calculations is easy enough, there is considerable doubt


whether the results from consolidometer tests can be used to accurately determine
the secondary settlement of real clay layers, because many factors can influence the
clay's behaviour. There is experimental evidence that specimen thickness, magnitude
of load increment, cementation and overconsolidation ratio all have an effect on the
consolidometer results.

Example 6.4

Take example 9.6 (p. 235*) in the prescribed book. The following values are given:

• Thickness of layer = 3 m
• Void ratio (e0) = 0.8
• Compression index (Cc) = 0.28
• Average effective pressure on the clay layer 𝜎𝜎0′ = 130 kN/m2
• ∆𝜎𝜎 ′ = 50 kN/m2
• Secondary compression index (𝐶𝐶∞ ) = 0.02

153 GEE2601/1
What is the total consolidation settlement of the clay layer 5 years after the completion
of primary consolidation settlement? (Note: Time for completion of primary settlement
= 1.5 years.)

Solution

The primary consolidation settlement was calculated in example 6.2 (or example 9.3
(pp. 229-230*) in the prescribed book) as 62.3 mm. From equation 9.29 (p. 233*) in
the prescribed book:
𝐶𝐶
• 𝐶𝐶𝛼𝛼′ = 1+𝑒𝑒𝛼𝛼 ; The value of ep can be calculated as 𝑒𝑒𝑝𝑝 = 𝑒𝑒0 − ∆𝑒𝑒𝑝𝑝𝑝𝑝𝑝𝑝𝑝𝑝𝑝𝑝𝑝𝑝𝑝𝑝
𝑝𝑝

• Combining equations 9.14 (p. 224*) and 9.15 (p. 224*), we find the following:
𝜎𝜎𝑜𝑜′ +∆𝜎𝜎′ 130+50
• ∆𝑒𝑒 = 𝐶𝐶𝑐𝑐 𝑙𝑙𝑙𝑙𝑙𝑙 � �=0.28log� � = 0.04
𝜎𝜎𝑜𝑜′ 130
∆𝑒𝑒𝑒𝑒 0.04×3
• Primary consolidation settlement: 𝑆𝑆𝑝𝑝 = 1+𝑒𝑒 = 1+0.8
= 0.067𝑚𝑚
0

• 𝑒𝑒𝑝𝑝 = 𝑒𝑒0 − ∆𝑒𝑒𝑝𝑝𝑝𝑝𝑝𝑝𝑝𝑝𝑝𝑝𝑝𝑝𝑝𝑝 = 0.8 − 0.04 = 0.76


• Hp = 3.0 – 0.067 =2.933 m

Hence:
𝐶𝐶 0.02
• 𝐶𝐶𝛼𝛼′ = 1+𝑒𝑒𝛼𝛼 = 1+0.76
= 0.011
𝑝𝑝

From equation 9.28 (p. 233*):

𝑡𝑡 5
• 𝑆𝑆𝑆𝑆 = 𝐶𝐶𝛼𝛼′ 𝐻𝐻𝑝𝑝 𝑙𝑙𝑙𝑙𝑙𝑙 �𝑡𝑡2 � = 0.011 × 2.933 × 𝑙𝑙𝑙𝑙𝑙𝑙 �1.5� = 0.017𝑚𝑚
𝑝𝑝

Total consolidation settlement = Primary consolidation + Secondary consolidation

• Total consolidation settlement = 0.067 + 0.017 = 84 mm

154 GEE2601/1
6.9 TIME RATE OF PRIMARY CONSOLIDATION

Section 9.10 (pp. 236–241*) in the prescribed book covers this topic. When a total
stress increment over a wide area is applied to saturated clay, it is assumed that the
pore water pressure rises to take up the increase and that the effective stress will not
be affected. Over time the dissipation of pore water pressure gradually causes an
equal increase in effective stress, so that after a long time, the total stress increase is
carried entirely by the effective stress. As the effective stress increases, deformation
of the clay occurs. Figure 9.2(c) (p. 214*) shows that the percentages of the total
stress increments carried by the pore water pressure and effective stress vary over
the thickness of the layer. This means that the vertical strain will also vary over the
thickness of the layer. Terzaghi (1943) proposed a mathematical model, based on
several assumptions, of the rate of effective stress increase at any given depth in the
layer. Although it may be argued that some of Terzaghi's assumptions are debatable,
his theory is acceptable. Careful consolidometer testing is required to obtain the
required soil constants, though.

What is important for the geotechnical engineer and technician is to know how to
calculate the average degree of consolidation throughout the entire layer. This may be
defined as the ratio of the amount of settlement at a given time to the total primary
consolidation settlement. The symbol for degree of consolidation is U. Average degree
of consolidation, U, must not be confused with local degree of consolidation Uz that
varies with depth, as shown in figure 9.20 (p. 240*) in the prescribed book. Uz is
smallest for the level furthest from the drainage boundary. To calculate U may not be
an easy task, as may be clear from equation 9.41 (p. 239*) in the prescribed book,
and approximations in the form of equations 9.43 and 9.44 (p. 240*) may be used
instead. The definition of the time factor, Tv, in these equations is

cvt
• Tv = (6.8)
H2dr

155 GEE2601/1
Where:

• Hdr = Length of drainage path


• t = Time since start of primary consolidation
• cv = Coefficient of consolidation of the clay = k/(mvγw)

The definition of length of drainage path, Hdr, is the longest path through the soil that
the water has to migrate, and it depends on the drainage at the clay layer's boundaries.
The figures in table 6.2 of the prescribed book indicate that, for two-way drainage, Hdr
is half of the layer thickness, but it is equal to the layer thickness if drainage is only up
or down. The solutions in table 6.2 apply only to one-dimensional consolidation. Other
solutions for Hdr exist if the initial excess pore water pressure is not constant over the
entire depth of the clay layer, as would be caused by small loaded areas.

6.9.1 Determining the coefficient of consolidation

Section 9.11 (pp. 241-248*) in the prescribed book covers this topic. The value of cv
can be determined by running a consolidometer test in which the deformation of the
specimen is observed over time after an increase in the applied stress. A graphical
procedure, or curve-fitting method, is followed in which the deformation-time response
of the clay is compared with the mathematical relationship derived by Terzaghi (1947).
The graphical procedure can be performed either with time plotted on a logarithmic
scale, such as in figure 9.21 (p. 242*) in the prescribed book, or by plotting the square
root of the time. The latter is shown in figure 9.22 (p. 242*) in the prescribed book.
The procedures will not be repeated here.

Both procedures determine the time required for a certain average percentage of
consolidation of the specimen. From this time, and the corresponding value of the time
factor from table 9.3 (p. 241*) in the prescribed book, the value of cv can be calculated
for the clay. The consolidometer allows drainage to the top and bottom porous stones,
so Hdr is taken as half of the specimen height. In practice, the value of cv is determined
for more than one stress increment, because the compressibility and hydraulic
conductivity of the clay change with compression, and cv is a function of both.

156 GEE2601/1
Unfortunately, cv is very sensitive to sample disturbance, and to the magnitude of the
load increment, to name but a few factors, so settlement rates of clay layers sometimes
do not correspond to calculations.

Example 6.5

The consolidometer specimen's height after consolidation at 200 kN/m2 was 16.80
mm. The stress was increased to 400 kN/m2, and the additional deformation of the
specimen observed. The results are shown in the table below. Use the square-root-
of-time method to determine the coefficient of consolidation.

TABLE 6.5: Consolidometer readings (example 6.5)

Time (min) 0 0.25 0.50 1.00 2.50 4 9 16 25


Def. (mm) 0 0.31 0.39 0.48 0.60 0.74 0.99 1.25 1.51
Time (min) 36 49 64 81 100 200 400 1440
Def. (mm) 1.72 1.85 1.94 2.00 2.04 2.16 2.24 2.38

Solution

First determine the length of the drainage path. Use the average length over the stress
increment, which is half of the average height.

• Before increment: Hdr = 0.5 × 16.80 = 8.40 mm


• At the end of the increment: Hdr = 0.5 × (16.80 – 2.38) = 7.21 mm
• Average value of Hdr = 7.80 mm
• t90 = (7.4)2 = 54.76 min.

The value of T90, that is Tv at 90% average consolidation, can be read from table 9.22
(p. 241*) in the prescribed book as 0,848.
2
𝑇𝑇90 𝐻𝐻𝑑𝑑𝑑𝑑 7,802
𝑐𝑐𝑣𝑣 = = 0,848 = 0,942 𝑚𝑚𝑚𝑚2 / 𝑚𝑚𝑚𝑚𝑚𝑚
𝑡𝑡90 54,76

157 GEE2601/1
It is customary to express cv in units of m2/year, so

• cv = 0.496 m2/year

FIGURE 6.7
Square-root-of-time curve for example 6.5

6.9.2 Time for primary consolidation settlement

The time required for complete primary consolidation settlement is infinity, according
to the theory. This is clear from the value of T100 in table 9.3 (p. 241*) in the prescribed
book. To calculate the settlement after a given time, or the time for a certain settlement,
it is just a matter of applying the equation for Tv and the relationship between Tv and
U in table 9.3 (p. 241*) in the prescribed book, where U is the average degree of
consolidation. You will see several examples in the prescribed book, but they are
aimed mainly at illustrating the principles of Terzaghi's theory of consolidation. You
should concentrate on the calculation of settlement with time.

Example 6.6

A normally consolidated clay layer is 4 m thick, and is doubly drained. Above the clay
is a 5 m sand layer with unit weight of 16 kN/m3 above the water table and 20 kN/m3

158 GEE2601/1
below it. The water table is at 2 m below the surface. The properties of the clay are e0
= 0.80, Cc = 0.33, saturated unit weight = 21 kN/m3, cv = 0.5m2/yr. If a flexible 100
kN/m2 load is placed on the surface, calculate the following:

(a) Total primary consolidation settlement


(b) Primary consolidation settlement after 3 years
(c) Time required for 50 mm settlement
(d) The effect on the time required for 50 mm settlement if the drainage to the top is
prevented by the load

Solution

Calculate the total settlement:

Initial effective stress in middle of clay:

• = 2 × 16 + 3 × (20 – 10) + 2 × (21 – 10) = 84 kN/m2, if unit weight of water is taken


to be 10 kN/m3

Effective stress increase in middle of clay = 100 kN/m2

𝐶𝐶 𝐻𝐻 𝜎𝜎0′ +𝛥𝛥𝜎𝜎′ 0,33×4 84+100


• 𝑐𝑐
𝑆𝑆 = (1+𝑒𝑒 𝑙𝑙𝑙𝑙𝑙𝑙 = (1+0,8) 𝑙𝑙𝑙𝑙𝑙𝑙 = 0,250 𝑚𝑚 = 250 𝑚𝑚𝑚𝑚
)
0 𝜎𝜎0′ 84

Calculate the settlement after 3 years:

c v t 0,5 × 3
Tv = = = 0,375
• H2dr 22

Table 9.3 (p. 241*) in the prescribed book gives U = 87.2%, so the settlement after 3
years will be

• S3 = 0.872 x 250 = 218 mm

Calculate the time required for 50 mm settlement:

• U = 50/250 = 0.2 or 20% and figure 9.2 (p. 240*) in the prescribed book gives as
T20 = 0.0314

159 GEE2601/1
H2dr Tv 22 × 0,0314
t= = = 0,251yr.
• cv 0,5

Calculate the effect of single drainage:

• With single drainage, the length of the drainage path becomes Hdr = 4 m. The time
for 20% consolidation is changed to

H2dr Tv 42 × 0,0314
t= = = 1,005 yr
cv 0,5

6.9.3 Increasing the rate of precompression

Very thick layers of clay of low hydraulic conductivity will take very long to consolidate,
especially if only one-way drainage is allowed by the soil profile. As a result, preloading
may be uneconomical. By installing sand or wick drains in the clay in a regular grid
pattern, the drainage path may be shortened considerably, leading to increased rates
of consolidation.

6.10 SELF-ASSESSMENT ACTIVITIES

You now have the opportunity to apply what you have learnt in this unit. Answer the
following questions:

Activity 6.1: Prescribed book exercises

1. Read the summary in section 9.16 (p. 260*).


2. Carefully study examples 9.1 to 9.11.
3. Do problems 9.1 to 9.10.

Activity 6.2: Multiple-choice questions

1. The compression of soil layers as a result of foundation or other loadings is


caused by …
(a) deformation of soil particles.
(b) relocation of soil particles.

160 GEE2601/1
(c) expulsion of water and/air from the void spaces.
(d) All of the above.

2. The elastic settlement of the ground is caused by the deformation of soil …


(a) without a change in its water content.
(b) with a decrease in its water content.
(c) with an increase in its water content.
(d) in its dry condition.

3. When a saturated soil layer is subjected to a stress increase, the pore water
pressure …
(a) decreases.
(b) increases.
(c) remains constant.
(d) becomes zero.

4. The drainage caused by an increase in the pore water pressure in saturated


sandy soils …
(a) takes place slowly.
(b) takes place very slowly.
(c) takes place quickly.
(d) depends on the amount of water present in the soil.

5. Which of the following soils can have elastic and consolidation settlements
occurring simultaneously?
(a) clayey and silty soils
(b) sandy soils
(c) gravelly soils
(d) Both (b) and (c).

161 GEE2601/1
6. Which of the following settlements of ground surface as a result of foundation or
other loadings is an immediate settlement?
(a) elastic settlement
(b) consolidation settlement
(c) Both (a) and (b).
(d) None of the above.

7. Select the correct statement:


(a) Consolidation settlement in clay is generally equal to the elastic settlement.
(b) Consolidation settlement in clay is generally smaller than the elastic
settlement.
(c) Consolidation settlement in clay is generally several times greater than the
elastic settlement.
(d) Consolidation settlement in clay is generally negligible.

8. If a load is placed on the frictionless watertight piston of a model that consists of


cylinder with a spring at its centre and filled with water, and the drainage of water
is not allowed by opening the valve attached to the cylinder, the entire load will
be taken by …
(a) the spring.
(b) the water.
(c) both the spring and the water in a fixed proportion depending on their
individual compressibility values.
(d) None of the above.

9. Theoretically, the consolidation of clayey soil ends at time …


(a) t = 0.
(b) t = 6 months.
(c) t = 1year.
(d) t = ∞.

162 GEE2601/1
10. At the end of consolidation of clayey soil, which of the following is correct?
(a) total stress increase = effective stress increase
(b) excess pore water pressure = 0
(c) Both (a) and (b).
(d) total stress increase = excess pore water pressure

11. In the one-dimensional laboratory consolidation test, the soil specimen is kept
… in condition.
(a) air-dried
(b) oven-dried
(c) partially saturated
(d) fully saturated

12. During secondary consolidation of clayey soils, …


(a) expulsion of pore water pressure does not take place.
(b) some deformation takes place because of the plastic readjustment of soil
fabric.
(c) Both (a) and (b).
(d) None of the above.

13. For normally consolidated clays, the overconsolidation ratio (OCR) is …


(a) 0.
(b) 1.
(c) less than 1.
(d) greater than 1.

14. On a semi-logarithmic plot, the virgin compression curve is …


(a) a straight line.
(b) approximately a straight line.
(c) a parabolic curve.
(d) a circular curve.

163 GEE2601/1
15. The slope of the void ratio versus logarithm of effective pressure is called the …
(a) compression index.
(b) coefficient of compressibility.
(c) coefficient of consolidation.
(d) coefficient of volume compressibility.

16. If the liquid limit of undisturbed clay is 40%, its compression index will be
approximately …
(a) 0.21.
(b) 0.27.
(c) 0.3.
(d) None of the above.

17. Select the correct statement:


(a) swell index = compression index
(b) swell index > compression index
(c) swell index < compression index
(d) swell index << compression index

18. If the plasticity index of a clay is 20%, its compression index will be
approximately …
(a) 0.05.
(b) 0.07.
(c) 0.09.
(d) 0.27.

19. At the end of primary consolidation of soil, some settlement is observed


because of the plastic adjustment of soil fabrics, which is usually termed …
(a) creep.
(b) elastic settlement.

164 GEE2601/1
(c) consolidation settlement.
(d) immediate settlement.

20. Secondary consolidation settlement (usually termed creep) is more important


than primary consolidation settlement in … soils.
(a) sandy
(b) silty
(c) clayey
(d) organic and highly compressible inorganic

21. Which of the following is not an assumption of Terzaghi's one-dimensional


consolidation theory?
(a) The clay-water system is homogeneous.
(b) Saturation is incomplete.
(c) Compressibility of water is negligible.
(d) Darcy's law is valid.

22. Where symbols have their usual meaning, the time factor is defined as …
𝐶𝐶 𝑇𝑇
(a) 𝑇𝑇𝑣𝑣 = 𝐻𝐻𝑣𝑣2
𝑑𝑑𝑑𝑑

𝐶𝐶𝑣𝑣 𝑇𝑇
(b) 𝑇𝑇𝑣𝑣 = 𝐻𝐻
𝑑𝑑𝑑𝑑

𝐻𝐻 2
(c) 𝑇𝑇𝑣𝑣 = 𝐶𝐶 𝑑𝑑𝑑𝑑𝑇𝑇.
𝑉𝑉

(d) None of the above.

23. If the average degree of consolidation is 30%, the time factor will be …
(a) 0.
(b) 0.071.
(c) 0.236.
(d) 1.0.

165 GEE2601/1
24. The coefficient of consolidation of soil generally …
(a) does not vary with a change in liquid limit of soil.
(b) increases as the liquid limit of soil increases.
(c) decreases as the liquid limit of soil increases.
(d) does not have a regular trend of variation with changes in the liquid limit of
soil.

25. If a 3 m thick layer (double drainage) of saturated clay under a surcharge loading
underwent 90% primary consolidation in 75 days, the coefficient of consolidation
will be …
(a) 0.
(b) 0.00294 cm/s.
(c) 0.00294 cm2/s.
(d) 0.0294 cm2/s.

26. In several instances, foundation engineers use an approximate method to


determine the increase of stress with depth caused by the construction of a
foundation. This is referred to as …
(a) 1 vertical to 1 horizontal method.
(b) 2 vertical to 1 horizontal method.
(c) 2 vertical to 1.5 horizontal method.
(d) 1 vertical to 2 horizontal method.

27. Skempton-Bjerrum modification applies to …


(a) elastic settlement.
(b) primary consolidation settlement.
(c) secondary consolidation settlement.
(d) All of the above.

166 GEE2601/1
Activity 6.3: Typical exam questions

1. A 1.2 m height highway embankment in Soweto (Johannesburg) of large lateral


extent is to be placed over a surface clay deposit of 10 m thickness which is
underlain by sand as shown in the figure below.
(a) Compute the ultimate settlement expected under the embankment.
(b) If the maximum tolerable settlement of the embankment surface
pavement is 3 cm, when should the pavement be placed after completion
of embankment placement?
(c) What is the average excess pore water pressure at the completion of
embankment placement?
(d) What is the total effective stress (at mid of clay layer) at the time of
paving?

2. The soil profile shown below is for a foundation of an elevated water tower to be built
in Rathoke village near Marble Hall in Mpumalanga. Calculate the total primary
consolidation settlement due to 150 KPa net foundation loading. (Take γw = 10
KN/m3.)

167 GEE2601/1
6.11 SOLUTIONS TO SELF-ASSESSMENT ACTIVITIES

Activity 6.1: Prescribed book exercises

1. Consult the prescribed book for answers.


2. Consult the prescribed book for answers.
3. Consult the solution book for answers.

Activity 6.2: Multiple-choice questions

1. (d)
2. (a)
3. (b)
4. (c)
5. (d)
6. (a)
7. (c)
8. (b) because water is incompressible; (a) is correct when drainage of water gets
stopped when the valve is kept opened; and (c) is correct when the drainage of
water keeps taking place when the valve is opened.

168 GEE2601/1
9. (d)
10. (c)
11. (d)
12. (c)
13. (b)
14. (b)
15. (a)
16. (b)
17. (d)
18. (d)
19. (a)
20. (d)
21. (b)
22. (a)
23. (b)
24. (c)
25. (c)
26. (b)
27. (b)

Activity 6.3: Typical exam questions

1. (a) SC(t=∞) = 0.0878 m = 8.78 cm; (b) t = 0.7 yr = 255.5 days; (c) Δσ = 19.32
kN/m2; (d) σt ′= 45.9 KN/m2
2. SC(total) = 22.57 cm

169 GEE2601/1
6.12 SUMMARY

This study unit covered a vast amount of information on the consolidation of clay. The
following has been established:

• Consolidation settlement is not instantaneous but takes place over time.


• The size of the loaded area is important.
• Sample disturbance has a substantial influence on the laboratory results.
• Secondary consolidation settlement occurs after complete dissipation of excess
pore water pressure.

With this knowledge, you are now able to perform a consolidometer test on an
undisturbed clay specimen, and draw the relationship between the void ratio and the
effective stress from the results. Furthermore, you should also be able to interpret this
relationship to determine the preconsolidation pressure, construct the in situ
consolidation curve and determine the compression and swell indices of the clay. You
should also know how to calculate the coefficient of consolidation and the secondary
compression index from the time settlement curve. In addition to determining the
consolidation characteristics of clays, you should be able to use these to calculate the
settlement below large and small loaded areas, and the time required for
consolidation.

6.13 VIDEO CLIPS, SLIDES AND OTHER WEB-BASED


RESOURCES

For additional explanations on consolidation settlement, you can watch some video
clips. Web addresses/links to the video clips are available on the module site under
Additional Resources in a folder named Video clips, slides and other web-based
resources:

• Primary consolidation settlement solved examples:


https://www.youtube.com/watch?v=eYHKGbgsgjo

170 GEE2601/1
• 1-D consolidation and settlement part 2:
https://www.youtube.com/watch?v=Fw2sw3BzyT4
• Terzaghi's theory of consolidation settlement, soil mechanics:
https://www.youtube.com/watch?v=szI839XV758
• CEEN 341 - Lecture 15 - Elastic settlement and primary consolidation settlement:
https://www.youtube.com/watch?v=LxOyMLs503c

REFERENCE WORKS AND FURTHER READING

Al-Agha, AS. 2015. Basics of foundation engineering with solved problems. Based on
“Principles of foundation engineering, 7th edition”.

Casagrande, A. 1936. Determination of the preconsolidation load and its practical


significance. Proceedings of the 1st International Conference on Soil Mechanics
and Foundation Engineering, Cambridge, Mass., Vol. 3, pp. 60-64.

Das, BM. 2010. Principles of geotechnical engineering. 7th ed. Connecticut: Cengage
Learning.

Das, BM. 2013. Fundamentals of geotechnical engineering. 4th ed. Connecticut:


Cengage Learning.

Das, BM & Sivakugan, N. 2017. Fundamentals of geotechnical engineering. 5th ed.,


international edition. Connecticut: Cengage Learning.

Das, B.M. & Sobhan, K. (2014). Principles of Geotechnical Engineering, 8th Edition, SI
Edition. CENGAGE Learning. ISBN 13: 9788131521588 Connecticut, United
States of America.

Terzaghi, K. 1934. Large retaining wall tests. Engineering News Record, 1 Feb, 8
March, 19 April.

Terzaghi, K. 1943. Theoretical soil mechanics. New York: Wiley.

Terzaghi, K & Peck, RB. 1967. Soil mechanics in engineering practice. 2nd ed. New
York: John Wiley.

171 GEE2601/1
Skempton, AW & Bjerrum, L. 1957. A contribution to the settlement analysis of
foundations on clay. Geotechnique, 7:168.

* Page, figure, table and chapter numbers may differ on updated and revised versions of the prescribed
book. See online version of study guide for updates.

172 GEE2601/1
Study unit 1 7

SHEAR STRENGTH OF SOIL

7.1 INTRODUCTION

Soil is particulate in nature. This characteristic makes it impossible for soil to support
tensile stress for a prolonged period in the absence of cementation. On the other hand,
mineral grains are usually very strong and have low compressibility, so soil should be
very capable of supporting compressive stress. But, it is well known from studies in
strength of materials that if unequal compressive stresses are applied in the principal
directions, shear stresses are generated and they make soil weak in shear. From the
above, it seems that since compressive stresses are caused or influenced by gravity,
seepage and imposed surface loads, the resistance of the soil to the accompanying
shear stresses is something that should concern geotechnical engineers and
technicians. When you think of the various situations where shear stresses might exist,
like below foundations, around piles and ground anchors, in natural slopes,
embankments, earth dams, behind, below and in front of retaining walls and in road
pavements, it is clear that resistance to shear stress is what keeps everything from
shifting, sliding, sinking, toppling, or being pulled out of the ground.

This resistance to shear stress is what geotechnical practitioners call “shear strength”
and it is a resisting force per unit area, with the same units as stress. Before
proceeding, it is important to note that since there is never any significant tensile stress
in soil, it is preferable to change the standard structural notation and regard
compressive stress as positive. If this is not done, there will only be negative
(compressive) normal stress, and geotechnical engineers and technicians like to be
positive. The same applies to deformation, strain, volume change and volumetric strain

173 GEE2601/1
if a soil element becomes shorter or smaller, that quantity is positive. Chapter 10 (pp.
268-311*) in the prescribed book covers this study unit. As a general rule, revise the
theory on the Mohr circle on strength of materials.

7.2 DEFINITIONS

You will come across the following terms in this study unit:

• Deviator stress: The difference between the major and minor principal stresses
in a triaxial test, which is equal to the axial load applied to the specimen divided
by the cross-sectional area of the specimen.
• Mohr's circle: A graphical illustration of the mathematical equations of the
variation of normal and shear stress on a plane through an element of a material,
if the plane is rotated about the centre of the element.
• Principal stress: Normal stresses that are acting on the principal plane in which
the shear stresses are zero.
• Shear strength: The internal shear resistance per unit area that the soil mass can
offer to resist failure and sliding along any plane inside.

7.3 LEARNING OUTCOMES

On completion of this study unit, you should be able to apply the principles of shear
strength. More specifically, you should be able to

• explain how drainage conditions affect the shear behaviour of soil


• differentiate between the shear strength of coarse- and fine-grained soils
• explain the influence of stress changes on pore water pressure
• perform calculations that involve shear strength
• determine the shear strength parameters of soil by means of laboratory and on-
site tests

To achieve the learning outcomes in this unit, work through the following topics:

• Mohr-Coulomb failure criterion

174 GEE2601/1
• shear failure in saturated soil
• on-site shear strength testing
• effects of drainage conditions on shear behaviour
• pore pressure coefficients
• PRACTICAL: The direct shear test (shear box test), triaxial test and unconfined
compression test must be done in the laboratory and the results converted into
shear strength parameters.

7.4 MOHR-COULOMB FAILURE CRITERION

Section 10.2 (pp. 268-271*) in the prescribed book covers this topic. Mohr's circle is
a graphical illustration of the mathematical equations of the variation of normal and
shear stress on a plane through an element of a material, if the plane is rotated about
the centre of the element. The normal stress varies between the maximum and
minimum principal values in two orthogonal directions, and the shear stress varies
between plus and minus half of the difference between these principal stresses. A
negative shear stress simply means that the shear stress is in the opposite direction.

Mohr postulated that if the material fails in shear, it will be in the plane on which a
critical combination of normal and shear stresses act, and not on the plane of
maximum shear stress. The critical combination, if expressed in the form of a shear
stress vs normal stress plot, was experimentally found to be curved, as in figure 10.1
(p. 270*) in the prescribed book and figure 7.1 below. Coulomb (1776) expanded on
Mohr's postulation, and suggested that the curve could be approximated by a straight
line, over a small variation in normal stress. The result was the Mohr-Coulomb failure
criterion, equation 10.2 (p. 269*) in the prescribed book. The straight line has an
abscissa, called cohesion, on the shear stress axis and an inclination, tanφ. The angle
φ is called the angle of internal friction. The variables c and φ are called the shear
strength parameters.

175 GEE2601/1
FIGURE 7.1
Mohr's failure envelope and the Mohr-Coulomb failure criterion (Das & Sobhan 2014)

The physical meaning of the failure criterion is as follows:

• If an element of soil is acted upon by a stress system, and on any plane through
the element a combination of shear and normal stresses exists that meets the
failure criterion, the element will fail in shear.
• Graphically this may be interpreted as follows: If the stress coordinates on any
plane through the element, or the Mohr circle that represents the stress system on
the element, is plotted and it touches the failure criterion (or failure envelope as it
is also known), shear failure occurs.
• The orientation of the shear plane may be determined graphically by connecting
the point at which the Mohr circle touches the failure envelope with the centre of
the circle, as shown in figures 10.2 and 10.3 (p. 271*) in the prescribed book.
• A useful expression for the combination of principal stresses that will generate a
critical combination of shear and normal stresses is shown in the form of equation
10.8 (p. 272*) in the prescribed book.

176 GEE2601/1
7.5 FACTORS DETERMINING SHEAR STRENGTH

Sections 10.3 and 10.4 (pp. 271-280*) in the prescribed book cover this topic. The
shear strength of soil is provided mainly by frictional resistance between the mineral
grains. The Mohr-Coulomb failure criterion must therefore be expressed in terms of
effective normal stress, because effective stress is carried by the grains, and it is the
grain skeleton that resists the shear stress. The water carries the pore water pressure,
but makes no contribution to the resistance to shear. Equation 10.9 (p. 272*) in the
prescribed book is the failure criterion in terms of effective stress. All normal stresses
must be in terms of effective stress, including the Mohr circle.

Note that in most textbooks, some of which were written by the author of our prescribed
book, the shear strength parameters are written as c' and φ' to clearly state that
effective stresses were used in their determination. We will adopt this notation in this
study guide because it is international practice. When effective stresses cannot be
determined, total stresses are used and the parameters are denoted by cu and φu. The
meaning of the subscript, u, will become clear later. The shear strength parameters
for a given soil can only be determined by extensive testing in the laboratory. Their
values are governed by material properties and test conditions. Some of the material
properties involved are

• porosity
• particle-size distribution
• shape and orientation of soil particles
• water content
• presence of clay minerals
• stress history
• structure and possible cementation of the soil

The test conditions are concerned mainly with whether or not pore pressures that
accumulate during the test are allowed to dissipate and specimens are free to undergo
volume change. These effects will be discussed in detail in the following sections.

177 GEE2601/1
From the above you should realise how important it is to obtain and test representative
and undisturbed samples, and to keep the test conditions as close as possible to those
in the on-site soil mass.

In section 10.3 (p. 271-273*) in the prescribed book typical ranges of values of c' and
φ' are given in the text and in table 10.1 (p. 270*) for normally consolidated and
overconsolidated clays, silts and coarse-grained soils. You should be familiar with
these in order to judge whether test results are acceptable or not.

7.6 PORE WATER PRESSURE CHANGES DURING SHEAR

If the total stresses on a saturated soil element change, there is a tendency for the
element to change its volume. As we have discussed in the study unit on consolidation,
volume change is impossible if the pore water is not allowed to escape, so any change
in total stresses results in an excess pore water pressure. In the field, rapid water
movement and volume change may be prevented by low hydraulic conductivity of the
soil, whereas in laboratory shear tests the operator may do the same by sealing off a
specimen. Since any pore pressure has an effect on the shear strength of soil, it is
important that anybody working in the geotechnical field be fully aware of the way in
which pore pressure reacts to total stress changes and time.

7.6.1 General

In study unit 6 the process by which excess pore water pressure is slowly dissipated
and volume change allowed was called consolidation. In the discussion of shear
behaviour, it is convenient to name the pore pressure dissipation according to the
stress changes by which it was generated and two terms are used for the purpose,
namely consolidation and drainage.

If the excess pore water pressure resulted from a total stress change in the past, its
dissipation and the accompanying volume and effective stress changes are usually
called consolidation. We say, for example, that a clay layer has completely
consolidated under its own weight, or under the weight of an existing fill. In laboratory

178 GEE2601/1
testing, an undisturbed specimen is often subjected to a stress system that simulates
the conditions from which it was sampled, and time must be allowed for consolidation
before its shear strength can be determined.

In contrast to consolidation, the term “drainage” is usually used to describe dissipation


of excess pore water pressure caused by recent total stress changes. We often
describe the situation in soil of low hydraulic conductivity during or just after
construction as the undrained or short-term condition because there has been
insufficient time for dissipation of induced pore water pressures. The long-term or
drained condition would be at the end of the period required for drainage.

Because in situ principal stresses often do not differ much in the three orthogonal
directions and shear stresses are relatively small as a result, the term “consolidation”
is associated with low shear stress, especially in laboratory testing. Construction often
induces large shear stresses in the soil, and the same applies to the part of a
laboratory test in which a specimen is brought to shear failure, so drainage is generally
associated with dissipation in the presence of large shear stresses. In this sense, then,
the terms “consolidation” and “drainage” are used to distinguish between stages in a
laboratory experiment or a certain combination of stress changes that the experiment
is designed to simulate. To simplify the terminology, geotechnical engineers and
technicians distinguish between the following standard real-life and laboratory
situations:

• Consolidated-drained conditions exist when a soil has consolidated under one


set of total stresses, and then the stress system changes slowly so that no excess
pore water pressure can develop, or the new system of stresses remains in place
for sufficient time to allow dissipation. Coarse-grained soils, with high hydraulic
conductivity, will almost invariably behave in this way in the field, except during
earthquakes, explosions, etc.
• Consolidated-undrained conditions exist when a soil has consolidated under one
set of total stresses, and then the stress system changes rapidly so that excess
pore water pressure develops. The soil has not yet drained under the stress

179 GEE2601/1
system changes. This situation is often descriptive of the conditions in natural clay
soil during or soon after construction.
• Unconsolidated-undrained conditions may be described as the situation where
the soil has not yet consolidated under the previous stress system, and then the
system changes, generating additional excess pore water pressures.

We will discuss these three conditions again in sections to follow, and point out the
effects that these have on the behaviour and shear strength parameters. Any
uncertainties that may still exist about the different conditions and terms at this stage
will clear up eventually. One last remark: The term “drained” must not be confused
with dewatering, where most of the water is removed from a volume of soil by gravity
or pumping. “Drained” simply means that sufficient time is allowed for water movement
so that pore pressure can dissipate.

7.6.2 Pore pressure coefficients

It is assumed that any total principal stress change (∆σ1, ∆σ2, ∆σ3) in soil can be
regarded as a change in all-round pressure, equal to the change in the minor principal
total stress, ∆σ3, plus additional changes (∆σ2-∆σ3) and (∆σ1-∆σ3) in the other two
principal directions. To simplify calculations, it is assumed that ∆σ2 =∆σ3, so that (∆σ2-
∆σ3) falls away. The all-round pressure increase, ∆σ3, can cause no shear stresses
because it is the same in all three principal directions. When the excess pore pressure,
∆u3, resulting from it, dissipates, the process is usually called consolidation, as
discussed above. It is assumed that

• ∆u3 = B∆σ3 (7.1)

Since no volume change or relocation of particles is allowed by the isotropic stress


change in a saturated soil before onset of consolidation, the value of the coefficient,
B, should be very close to 1.0 except for the stiffest of soils (see table 10.2 (p. 283*)
in the prescribed book). B may be considerably less than 1.0 if the soil is not
completely saturated, because the smallest amount of air in the pores makes the soil
compressible. The additional total stress change in the major principal direction (∆σ1-

180 GEE2601/1
∆σ3) is called the deviator stress, σd, and it too will cause an increase, ∆u1, in pore
pressure, where

• ∆u1 = Ā(∆σ1-∆σ3) = Ā(σd) (7.2)

The value of coefficient, Ā, is a function of degree of saturation, overconsolidation ratio


and level of shear stress. Since the failure conditions are of special importance, the
value, Āf, at failure is usually stated. Typical values are shown in table 7.1 below for
completely saturated clay:

TABLE 7.1: Typical values of Āf

OCR 1 2 3 4 5 6 7 8 9 10
Āf 0,88 0,35 0,12 0,01 -0,05 -0,1 -0,11 -0,13 -0,14 -0,15

The dissipation of the excess pore water, ∆u1, is termed drainage, as discussed above.
We should emphasise that the two “types” of pore water pressure increase and
dissipation may occur simultaneously in field situations, and that we distinguish
between them for purposes of simplifying calculation, or for describing the steps in our
laboratory tests. In other cases, the changes do occur one at a time, even in the field.

Example 7.1

At a certain depth in a clay mass which was taken from a building site in Ventersdorp
(North West), the total stresses in an element are

• σ1 = 300 kN/m2
• σ3 = 100 kN/m2
• u = 50 kN/m2

The pore water pressure is caused by the depth below the water table. Construction
causes the following total stress changes: ∆σ1 = 120 kN/m2 and ∆σ3 = 40 kN/m2

Determine the effective stresses in the clay at that depth before construction started,
immediately after construction and a long time after construction. Assume that the clay
has a low hydraulic conductivity, B = 1.0 and Ā = 0.55.

181 GEE2601/1
Solution

Before construction, the clay has fully consolidated under its stresses:

• σ1' = σ1 – u = 300 – 50 = 250 kN/m2

• σ3' = σ3 – u = 100 – 50 = 50 kN/m2

Just after construction, also called the short-term situation, or undrained case

• All the stresses and pressures are equal to their previous values plus the
increments. Because the clay is assumed to have a low hydraulic conductivity, the
pore water pressure is independent of the water table in the short term.
• ∆u = B∆σ3 + Ā (∆σ1 – ∆σ3) = (1.0 x 40) + 0.55 x (120 – 40) = 84 kN/m2

• u = 50 + 84 = 134 kN/m2

• σ1 = 300 + 120 = 420 kN/m2

• σ1' = 420 – 134 = 286 kN/m2

• σ3 = 100 + 40 = 140 kN/m2

• σ3' = 140 – 134 = 16 kN/m2

A long time after construction, also called the long-term situation, or drained case

• The excess pore water pressure of 84 kN/m2 has dissipated and the pore water
pressure is governed by the water table again, so u = 50 kN/m2, while the total
stresses remain unchanged.

• σ1' = 420 – 50 = 370 kN/m2

• σ3' = 140 – 50 = 90 kN/m2

Note:

• From the solution of example 7.1 you can see that construction increased the
deviator stress, which is the diameter of the Mohr circle, from 200 to 280 kN/m2.
• At the same time the Mohr circle, in terms of effective stress, was moved to the left
by the elevated pore water pressure in the short-term situation.

182 GEE2601/1
• The combined effect could have caused the circle to touch the failure envelope!
The deviator stress remained at 280 kN/m2 after the end of construction, but
drainage moved the effective stress Mohr circle to the right, away from the failure
envelope.
• This illustrates that the undrained situation is usually a critical stage in the life of
foundations, embankments and many other structures on or in the ground.

7.7 DIRECT SHEAR TEST

Section 10.4 (pp. 273-280*) in the prescribed book covers this topic. In the direct
shear test, also called the shear box test, the specimen is confined within a metal box,
with porous plates at both ends, almost like the test setup of the consolidometer test.
The difference is that the shear box is split horizontally and the test equipment can
induce shear displacement between the two halves of the box, in addition to the normal
force, in order to shear the specimen. The normal and shear stresses at failure can be
calculated from the test results, and plotted to determine the shear strength
parameters of the soil. Figure 7.2 below and figure 10.3 (p. 273*) in the prescribed
book show the sample inside the shear box and figure 7.3 below and figure 10.4 (p.
274*) in the prescribed book show the equipment. You must know both the test setup
and test procedure for examination purposes.

The direct shear test, although it has certain disadvantages, is very useful for the shear
testing of sand, but clay specimens can be tested too. One big disadvantage is that
pore water pressure cannot be monitored, or dissipation controlled. As a result, the
test is only suitable for tests on dry sand or silt, or for consolidated-drained tests on
saturated sand, silt and clay. The test on saturated specimens must always be
performed extremely slowly to allow time for complete consolidation and drainage, so
that the pore water pressure remains at zero, otherwise the value of the normal stress
would be unknown.

183 GEE2601/1
FIGURE 7.2
Direct shear test arrangement (Das & Sobhan 2014)

FIGURE 7.3
Direct shear test apparatus (Das & Sobhan 2014)

184 GEE2601/1
7.7.1 Shear behaviour of dense and loose sand

Differences in the behaviour of various materials are clearly illustrated by means of a


direct shear test. Figure 10.5 (p. 275*) in the prescribed book shows the differences
between dense and loose specimens of the same sand sheared at the same normal
stress, σ'. Loose sand compresses during shear displacement, while the shear stress
gradually increases, until the shear strength, τf, is reached and the sand fails. In
contrast, dense sand initially compresses and then starts to expand with shear
displacement, and the shear stress increases rapidly to a maximum value. If shear
displacement is continued, the shear stress decreases again to a lower value. The
shear strength at failure of the dense sand, indicated by the peak in the curve, is called
the peak shear strength, and the final lower value is called the ultimate shear strength.
The latter is the shear strength after expansion. The ultimate strength of the dense
sand is approximately the same as the strength of the loose sand, because the void
ratio of the dense sand was increased by the expansion, whereas that of the loose
sand was decreased by the compression.

The stress coordinates (σ', τf) from a shear test will fall right on the Mohr-Coulomb
failure criterion for the soil. If a few more tests are performed on identical specimens
of the soil, each at a different normal stress, and all the coordinate pairs plotted, the
failure criterion can be drawn in by connecting the coordinate pairs, or fitting the best
straight line, as illustrated in figure 10.6 (p. 276*) in the prescribed book. The shear
strength parameters can then be determined. Clean sand and silt usually do not have
any cohesion intercept, so φ' completely describes the shear strength. The value of φ'
is sensitive to the density of the sand, and φ' at peak strength for the dense state may
be several degrees higher than φ' for the loose state. At high normal stresses, the
expansion of the dense sand is limited. This lowers the peak strength at high stresses,
resulting in a more curved failure envelope.

185 GEE2601/1
7.7.2 Shear behaviour of normally consolidated and overconsolidated clay

Section 10.6 (pp. 283-291*) in the prescribed book covers this topic. Saturated clay
specimens must be tested very slowly in the direct shear apparatus to guarantee
complete pore water pressure dissipation. The first step after the specimen has been
installed inside the shear box is to apply the normal stress to simulate the in situ
vertical stress on the soil. The applied normal stress will generate excess pore water
pressure within the soil, and time must be allowed for consolidation. Sampling will
probably allow some swelling in undisturbed specimens, and the consolidation in the
shear box is then called reconsolidation. After completion of consolidation, the sample
must be sheared at a very slow rate in order to allow complete drainage. Only then
can the operator be sure that there is no pore water pressure and that the applied
normal stress is indeed the effective stress.

The behaviour of normally consolidated and overconsolidated clay during shear is


similar to that of loose and dense sand. As illustrated in figure 10.15 (p. 286*) in the
prescribed book, overconsolidated clay has a peak shear strength, and then the shear
resistance is reduced until it finally reaches a minimum value called the residual
strength. Some authors also indicate ultimate shear strength at some displacement
well past the peak, as is done for dense sand. Residual strength is reached only after
a hundred millimetres or more for some types of clay, so it cannot be determined with
the direct shear apparatus. Normally consolidated clay has no peak and the shear
resistance increases with shear displacement until the shear strength is reached.

If the results from a number of direct shear tests on normally consolidated and
overconsolidated specimens of the same clay are plotted together on the normal
stress-shear stress axes, as was done in figure 10.14 (p. 285*) in the prescribed book,
the shear strength parameters can be determined from the two strength envelopes.
The data in the figure is for peak strengths in the case of the overconsolidated clay.
Normally consolidated clay has little or no cohesion, whereas overconsolidated clay
has cohesion, but its angle of internal friction is smaller than that of normally
consolidated clay.

186 GEE2601/1
The interpretation of shear strength parameters of overconsolidated clay and dense
sand must be done with care. For instance, should peak strength be used for
overconsolidated clay, or rather ultimate strength, or residual strength? For each of
these, shear strength parameters can be determined. The answer is not easy, but if
you are sure that there has never been any shear failure in the soil mass, peak values
may be used. If in doubt, rather use ultimate values. If evidence of existing shear
planes is found, residual strength should be used.

Example 7.2

The results shown in table 7.2 were obtained from direct shear tests on three
specimens of the same clay which were taken from a building site in Centurion
(Gauteng). A shear box with side lengths of 100 mm was used. Calculate the soil's
shear strength parameters.

TABLE 7.2: Direct shear tests for example 7.2

Shear Shear force Shear force Shear force


displacement on specimen 1 on specimen 2 on specimen 3
(mm) (N) (N) (N)
2 255 343 431
4 586 691 758
6 808 978 1 090
8 984 1 187 1 444
10 1 161 1 395 1 692
12 1 258 1 566 1 804
14 1 290 1 634 1 909
16 1 268 1 652 1 873
Normal force (N) 4 000 5 000 6 000

187 GEE2601/1
Solution

The displacement of the shear box during the test has the effect that the shear area
of the specimen changes and this has to be taken into account when shear stress is
calculated. For example, for 2 mm displacement, the shear area would be

• Area = 100 x (100 -2) = 9 800 mm2


• The shear stress can be calculated from
• Shear stress = (shear force)/(9 800)

The overhanging part of the specimen stays in contact with the rim of the shear box,
so normal stress can be taken as constant. Note that some authors keep the area
constant for shear stress calculation, but it is more correct to use the smaller area.

• Normal stress = (Normal force)/(100 x 100) N/mm2

The calculations are presented in table 7.3 below. Take note of the units.

TABLE 7.3: Direct shear calculations for example 7.2

Shear Shear Shear stress Shear stress Shear stress


displacement area on specimen 1 on specimen 2 on specimen 3
(mm) (mm2) (kN/m2) (kN/m2) (kN/m2)
2 9 800 26 35 44
4 9 600 61 72 79
6 9 400 86 102 117
8 9 200 107 129 157
10 9 000 129 152 183
12 8 800 143 178 205
14 8 600 150 190 221
16 8 400 151 186 223
Normal stress
400 500 600
(kN/m2)

The results should be plotted on a graph of shear stress against shear displacement
for evaluation purposes in figure 7.4 below. To determine the shear strength
parameters, plot the stress coordinates at failure. Since specimens do not fail at

188 GEE2601/1
exactly the same shear displacement, the shear stresses at the same displacement
are usually regarded as “failure”. In this example 15 mm will be used. The stress
coordinates are given in table 7.4 and are plotted in figure 7.5.

FIGURE 7.4
Shear stress against shear displacement for example 7.2

FIGURE 7.5
Stress coordinates for example 7.2

189 GEE2601/1
TABLE 7.4: Stress coordinates for example 7.2

Normal stress, Shear strength,


Specimen no. Specimen no.
σ' (kN/m2) τf (kN/m2)
1 400 150 1
2 500 188 2
3 600 222 3

From figure 7.5 the following may be determined:

• The cohesion is the intercept of the straight line with the vertical axis: c' = 6 kN/m2.
• The angle of internal friction is the angle of the straight line:
• Tan φ' = (223-6)/600 = 0.362
• φ' = 20o

Note:

• A protractor should not be used to measure φ' because the scales used for the two
axes in figure 7.5 are not identical.

7.8 TRIAXIAL SHEAR TESTS

Section 10.5 (pp. 280-282*) in the prescribed book covers this topic. Take note of the
basic principles of the triaxial test, and carefully study the details of the triaxial cell in
figure 10.11 (pp. 281-282*) in the prescribed book. You should learn the diagram for
examination purposes, as well as the procedures to perform the unconsolidated-
undrained, consolidated-undrained and consolidated-drained tests.

In the triaxial test, the specimen is in the shape of a cylinder. At the top and bottom it
is in contact with porous discs, a top cap and pedestal. The circumference of the
specimen is sealed off by a thin rubber membrane. It is connected to the outside only
through the porous discs via tubes running from the top cap and pedestal.
Instrumentation connected to the tubes monitors the volume change and/or the pore
water pressure, and the pore water pressure can be held at a constant value, called
the back pressure. The specimen should have a length of twice its diameter to

190 GEE2601/1
minimise the effects of the porous discs that prevent lateral deformation at the ends
during axial compression. The ideal is that a specimen must have a uniform
deformation over its length, but the end-effects cause it to deform rather like a wine
barrel.

The space between the triaxial chamber and the specimen sealed in its membrane is
filled with water. When the water is pressurised, it exerts pressure on all external
surfaces of the specimen, through the membrane and top cap. This pressure is
commonly known as the chamber pressure or confining pressure, and the stress that
it generates in the specimen is a principal stress since water pressure is always at
right angles to a surface. As you will see shortly, this is the minor principal stress, σ3.
The sample is loaded by application of a force in the loading ram. This generates an
additional principal stress on the horizontal plane. The magnitude of this stress is the
force divided by the cross-sectional area of the specimen, and it is called the deviator
stress, σd. Since all the applied stresses are compressive principal stresses, with the
vertical greater than the horizontal, the major principal stress is given by

• σ1 = σ3 + σd (7.3)

So that

• σd = σ1 - σ3 (7.4)

From equation 7.4 you can see that the deviator stress is the diameter of the Mohr
circle, which is a measure of the shear stress in the specimen, as we discussed before.
It is important to note that the area on which the force in the loading ram works is not
a constant. During compression the specimen becomes shorter and wider, thereby
increasing the cross-sectional area. At any stage of the test, when the axial
deformation is ∆L, and the volumetric deformation is ∆V, the average area of the
deformed specimen is taken as

1 − ∆V
V0
• A = A0 (7.5)
1 − ∆L
L0

191 GEE2601/1
Where:

• A0 = Initial cross-sectional area


• V0 = Initial specimen volume
• L0 = Initial specimen length

The deviator stress must be calculated taking this into account. From our discussion
in section 7.4 it should be clear that the chamber pressure simulates the confining
pressure from the surrounding soil in the field, while the deviator stress is increased
to bring the specimen to shear failure.

7.8.1 Consolidated-drained (CD) tests

Section 10.6 (pp. 282-291*) in the prescribed book covers this topic. The CD test is a
simulation of the consolidated-drained situation that we discussed in section 7.4. The
specimen is subjected to chamber pressure, which generates pore water pressure.
Time is allowed for consolidation until the pore water pressure reaches some
predetermined value, called the back pressure. The back pressure is used to keep the
specimen saturated and to simulate the static pore water pressure in the field situation.
In some tests a zero back pressure is used. After consolidation the effective confining
pressure can be calculated:

• σ3' = σ3 – u = Chamber pressure – Pore water pressure

The deviator stress is increased slowly, to allow time for drainage through the porous
discs and tubes, so that the specimen fails at a pure water pressure equal to the back
pressure, which is maintained by the instrumentation. Throughout the test the axial
deformation is measured by means of a dial gauge and the volumetric deformation by
recording the outflow of water through the connections to the porous stones. The
measured values of axial and volumetric deformations with increased axial force can
be plotted to illustrate the behaviour of different materials. This is illustrated in figure
10.13 (p. 284*) in the prescribed book.

The triaxial behaviour in figure 10.13 (p. 284*) agrees with the direct shear behaviour
in figures 10.6 and 10.8 (p. 276* & p. 278*, respectively) in the prescribed book. Loose

192 GEE2601/1
sand and normally consolidated clay gradually compress, whereas dense sand and
overconsolidated clay expand before shear failure. The peak shear strength in figure
10.13(c) is typical of dense sand and overconsolidated clay. By testing several
specimens of the soil, and plotting the Mohr circle at failure for each, a common
tangent may be drawn and the shear strength parameters determined from it, as
illustrated in figure 10.14 (p. 285*) in the prescribed book. Learn figures 10.13 to
10.15 (pp. 284-286*) in the prescribed book for examination purposes.

Example 7.3

A consolidated-drained triaxial test on a saturated specimen of soft clay taken from a


road construction site in Phalaborwa (Limpopo) was performed. The confining
pressure was 200 kN/m2 and the specimen was initially 38 mm in diameter and 76 mm
long. A zero back pressure was applied. The data recorded during the test appears in
the first three columns of the table. The remaining four columns show how the data
was interpreted to obtain the behaviour of the soil. The following were used in the
calculations:

• Initial area A0 = π/4 × 382 = 1 134.1 mm2


• Initial volume V0 = A0 × 76 mm3 = 1 134.1 mm2 × 76 mm = 86.19 cm3
TABLE 7.5: Consolidated-drained triaxial test data for example 7.3
∆L (mm) ∆V (cm3) Load (N) εa =∆L/L0 ∆V/V0 A (mm2)
0 0 0 0 0 1134,1
0,0304 0,05 46 0,004 0,00058 1138,0
0,593 0,15 74 0,0078 0,00174 1141,1
1,216 0,26 83 0,116 0,0030 1149,1
1,710 0,45 95 0,0154 0,0052 1145,9
1,459 0,65 103 0,0192 0,0075 1147,6
1,748 0,75 107 0,0230 0,0087 1150,7
2,28 1,05 124 0,030 0,0122 1154,9
2,83 1,35 136 0,0372 0,0157 1160,4
3,39 1,82 149 0,0445 0,0211 1162,0
3,88 2,07 161 0,051 0,0240 1166,5
4,39 2,34 173 0,0578 0,0272 1170,9

193 GEE2601/1
4,97 2,64 190 0,0654 0,0306 1176,3
5,50 2,89 202 0,0724 0,0335 1181,7
6,03 3,14 211 0,0794 0,0364 1186,9
6,58 3,35 219 0,0866 0,0389 1193,3
7,11 3,62 227 0,0936 0,0420 1198,7
7,65 3,82 235 0,1006 0,0443 1205,2
8,16 4,10 244 0,1074 0,0476 1210,1
8,69 4,33 252 0,1144 0,0502 1216,2
9,24 4,49 260 0,1216 0,0521 1223,8
9,76 4,65 258 0,1284 0,0540 1230,8

The maximum value of the deviator stress was 212.5 kN/m2, which is the assumed
failure of the specimen. With back pressure = 0, all stresses are effective and the Mohr
circle at failure may be drawn from

• σ3' = 200 kN/m2


• σ1' = 200 + 212.5 = 412.5 kN/m2

You should plot the data from the table, in the same way as in figure 10.13 (p. 284*)
in the prescribed book, and view the shear behaviour of the clay. Note that volume
change data during consolidation is usually ignored because it is relatively small.

Example 7.4

Three consolidated-drained triaxial tests were run on specimens of the same soil taken
from a construction site in Welkom (Free State). The results are shown in table 7.6
below. Only the results at the points of failure of the specimens are shown. All
specimens were initially 100 mm long and 50 mm in diameter. A 100 kN/m2 back
pressure was maintained artificially in all specimens with the purpose of keeping the
soil saturated and simulating the effect of the water table in the field situation. Calculate
the shear strength parameters of the soil.

194 GEE2601/1
TABLE 7.6: Consolidated-drained triaxial tests data for example 7.4

Specimen 1 Specimen 2 Specimen 3


Change in length (mm) 5 7 8
Confining pressure, σ3 (kN/m2) 200 400 600
Pore water pressure, u (kN/m2) 100 100 100
Change in volume (cm3) 5 7 10

Solution

From the dimensions of the specimens, the initial area and volume are calculated as

• A0 = 1 963.5 mm2

• V0 = 196.35 cm3 = 1 963.35 × 103 mm3

Calculate the conditions at failure. The calculations will be shown in detail for specimen
1 only, but all results are shown in the table below:

Area at failure:

1 − ∆V 1− 5
V0 196,35
A0 = 1963,5 × = 2014 mm 2
1 − ∆L 1− 5
• A= L0 100

Deviator stress at failure:

• σd = Force/Area = 469/2 014 N/mm2 = 0.2329 MN/m2 = 232.9 kN/m2

Minimum effective principal stress at failure:

• σ3' = σ3 – u = 200 – 100 = 100 kN/m2

Maximum effective principal stress at failure:

• σ1 = σ 3 + σ d = 200 + 232.9 ≈ 433 kN/m2


• σ1' = σ1 – u = 432.9 – 100 ≈ 333 kN/m2

195 GEE2601/1
TABLE 7.7: Calculations of consolidated-drained triaxial tests data for example 7.4

Specimen 1 Specimen 2 Specimen 3


Area (mm2) 2014 2036 2026
Cell pressure, σ3 (kN/m2) 200 400 600
Pore water pressure, u (kN/m2) 100 100 100
Deviation stress, σd (kN/m2) 232,9 316,8 405,2
σ3' (kN/m2) 100 300 500

The Mohr circle can now be drawn for each specimen at failure, using the effective
minor and major principal stresses σ3' and σ1', and the shear stress parameters
determined from the tangent common to the three circles. It is left to you to do this and
check if you agree with the following, somewhat unrealistic, values:

• c' = 85 kN/m2

• φ' = 9o

7.8.2 Consolidated-undrained (CU) tests

Section 10.7 (pp. 291-296*) in the prescribed book covers this topic. The
consolidated-undrained situation discussed in section 7.4 is simulated by the CU test.
Application of the chamber pressure will induce excess pore water pressure, ∆u3. The
specimen is allowed to consolidate under the influence of the confining pressure and
back pressure, until the pore water pressure is equal to the back pressure. The valves
in the connection lines are then closed, preventing any further drainage and volume
change. This also shuts the connection to the back pressure source so that it can no
longer influence the pore water pressure. When the deviator stress is subsequently
slowly increased to failure, excess pore water pressure, ∆u1, will develop. The pore
pressure starts at the value of the back pressure and then increases or decreases,
depending on ∆u1. During application of deviator stress, the axial deformation and pore

196 GEE2601/1
water pressure must be recorded. To calculate the cross-sectional area for deviator
stress, the value of ∆V in equation 7.5 is set to zero.

CU tests are ideal for determining the value of the pore pressure coefficients, B and
Ā, because the responses in the pore water pressure to changes in chamber and
deviator stresses are monitored. The behaviour of different types of soil in the CU test
is illustrated in figure 10.20 (p. 292*) in the prescribed book. You should learn the
figure for the examination.

Saturated loose sand or normally consolidated clay would try to compress and push
out some water during shear, like it did in the CD test. But with drainage prevented,
the pore water pressure increases. This pore water pressure reduces the effective
stresses within the specimen and stress at failure, i.e. the shear strength, is lower than
in the CD test. We can visualise Mohr's circle shifting to the left along the normal stress
axis as pore water pressure increases while the diameter of the circle is increasing to
reflect the increasing deviator stress. Mohr's circle is bound to touch the failure
envelope at a relatively low value of deviator stress. In contrast, during the CD test,
there is no excess pore water pressure, so Mohr's circle does not shift and deviator
stress can increase to a higher value before the circle will touch the failure envelope.

Dense sand and overconsolidated clay would like to expand, as it did in the CD test,
but the prevention of water movement will reduce the pore water pressure to low or
even negative values. Negative pore water pressure will increase effective stress and
the shear strength will be higher than that of the loose sand or normally consolidated
clay. Visualise Mohr's circle shifting to the right along the normal stress axis as the
pore water pressure decreases, allowing the deviator stress to increase to a higher
value before the failure envelope is touched.

Note:

• The shear strength parameters should be determined in terms of effective stresses,


because pore water pressures are monitored.
• Beware of Mohr-Coulomb failure envelopes in terms of total stress, such as the
solid lines in figures 10.20 to 10.22 (pp. 292-294*) in the prescribed book,

197 GEE2601/1
because their positions on the graph depend on the value of the back pressure,
so ccu and φcu do not have unique values for a given soil!
• If undrained shear strength values have to be used, as may be the case in short-
term analyses, rather draw up a table of deviator stress at failure against effective
consolidation pressure, σ3'(cons).
• Figures like these are useful to illustrate soil behaviour in textbooks, but do not use
them for design.
• However, do take note of the effect of overconsolidation on the shape of the failure
envelope illustrated in figure 10.21 (p. 293*) in the prescribed book.
• It is clear that all the tests should be performed at stress levels either above or
below the preconsolidation pressure, or both, in order to have common tangents
to the resulting Mohr circles.

Example 7.5

Consolidated-undrained triaxial tests on saturated overconsolidated clay specimens


taken from a site in Witbank (Mpumalanga) produced the values at failure shown in
table 7.8 below. Zero back pressure was used. Determine the shear strength
parameters for the clay, as well as the value of the pore pressure coefficient Āf,
assuming that B = 1.0.

TABLE 7.8: Consolidated-undrained triaxial tests on saturated overconsolidated clay


specimens for example 7.5

Chamber Deviator Pore water


pressure (kN/m2) stress (kN/m2) pressure (kN/m2)
Specimen 1 270 347 37
Specimen 2 540 613 65
Specimen 3 810 907 94

Solution

The solution will be presented in tabular form, since you should be familiar with doing
calculations from triaxial data by now.

198 GEE2601/1
TABLE 7.9: Consolidated-undrained triaxial tests on saturated overconsolidated clay
specimens for example 7.5
σ3 u σd σ3' σ1' Āf =
(kN/m2) (kN/m2) (kN/m2) (kN/m2) (kN/m2) ∆u / σd
Specimen 1 270 37 347 233 580 0,106
Specimen 2 540 65 613 475 1 088 0,106
Specimen 3 810 94 907 716 1 623 0,104

You should draw the Mohr circles, find the common tangent and determine the
effective stress shear strength parameters for the clay. These should be
• c' = 25 kN/m2
• φ' = 21.5o

The pore pressure coefficient is calculated as the average of the value from the three
tests.
• Āf = 0.105

7.8.3 Unconsolidated-undrained (UU) tests


Section 10.8 (pp. 296-299*) in the prescribed book covers this topic. The UU test is
usually performed on saturated undisturbed clay specimens to determine the short-
term or undrained shear strength of the clay. This is the existing shear strength without
the void ratio of the clay being changed by additional consolidation in laboratory tests.
The terminology may be confusing here, because the soil has actually been
consolidated before sampling by the weight of the overburden, so the “unconsolidated”
refers to the laboratory procedure only.

The specimen is confined by the chamber pressure but no consolidation is allowed,


so the pore water pressure absorbs virtually all of the chamber pressure since
coefficient B is very close to 1.0. The magnitude of the chamber pressure is not
important since, whatever it is, the effective stress, σ3', will not be affected. The
specimen is then sheared by increasing deviator stress without permitting drainage
and all specimens, being identical and initially at exactly the same σ3', will have the
same shear strength. This is illustrated in figure 10.24 (p. 297*) in the prescribed book

199 GEE2601/1
by several Mohr's circles of the same diameter. The saturated UU failure envelope in
terms of total stress is the common tangent to all the circles and it may be used without
a problem, because its position is not influenced by the initial pore water pressure.
Since the failure envelope is a horizontal line, the shear strength parameters are φu =
0 and cu. It is clear that cu is nothing but half the deviator stress at failure.

If pore water pressures should be monitored and effective stresses calculated, the
same effective stress Mohr circle would be obtained for all the specimens at failure.
This circle would touch the effective stress failure envelope as shown by circle Q in
figure 10.25 (p. 298*) in the prescribed book, but UU tests cannot be used to
determine c' and φ' because the failure envelope cannot be drawn from only one
effective stress circle. Usually we would retrieve undisturbed specimens from various
depths and determine the values of cu. These values, plotted against the depth of
retrieval, could then be very useful in the design of foundations and so on, because it
gives the variation with depth of the short-term shear strength. The parameter φ(cu)
indicated in figure 10.24 (p. 297*) in the prescribed book should not be used since it
has a different value for every circle.

7.8.4 Unconfined compression tests


Section 10.9 (pp. 299-301*) in the prescribed book covers this topic. The unconfined
compression test is a special type of UU test in which the confining pressure is zero.
Specimens need not be inside a triaxial chamber for the test and figure 10.27 (p. 300*)
shows an unconfined test setup. Sand cannot be tested in this way because a sand
specimen needs confinement to retain its cylindrical shape, and the test is used on
clay only. The unconfined shear strength, cu, which is half the unconfined compressive
strength, qu, is expected to be the same as that obtained from UU tests in the case of
saturated undisturbed specimens. In practice the clay swells and softens to some
extent after sampling, or fissures may open up, and in the UU test the chamber
pressure restores the original condition of the soil to some extent. In the absence of
the confining pressure, slightly lower values of cu are measured as shown in figure
10.26 (p. 299*) in the prescribed book.

200 GEE2601/1
7.8.5 Effect of sample disturbance

Section 10.11 (p. 302-303*) in the prescribed book covers this topic. Figure 10.30 (p.
303*) in the prescribed book shows the effect that disturbance of clay has on its shear
strength. The figure only mentions unconfined shear strength, but the effect will be
observed in all other types of shear strength tests too. The topmost curve in the figure
is the behaviour of the truly undisturbed condition, and that is what we intend to
measure in tests on representative specimens. The second curve shows the behaviour
for the same clay, consolidated to the undisturbed void ratio, after it had been
completely remoulded. Remoulding destroys the soil's structure and stress history and
these two factors contribute to shear strength. The difference in shear strength in the
two conditions may not be much different for some types of clay, whereas others may
lose a large percentage of their strength during remoulding, but all clay materials will
behave differently in the remoulded state.

We can expect that any disturbance of any clay will have an effect on its shear
behaviour, so the behaviour that we determine in our shear tests will be between those
of the completely undisturbed and totally remoulded conditions. The worse the
disturbance, the further away the performance will be from what we intend to
determine. Careful sampling, transportation and storage practices should be a very
high priority if characteristic results are to be obtained from laboratory shear testing
and virtually any other type of soil test.

7.9 SELF-ASSESSMENT ACTIVITIES

You now have the opportunity to apply what you have learnt in this unit. Answer the
following questions:

Activity 7.1: Prescribed book exercises

1. Read the summary in section 10.13 (pp. 305-306*).


2. Do examples 10.1 to 10.5 (pp. 279-296*).
3. Do problems 10.1 to 10.5 (p. 306*).

201 GEE2601/1
4. Do problem 10.9 (p. 307*).
5. Do problem 10.18 (p. 308*).

Activity 7.2: Multiple-choice questions

1. The internal resistance per unit area that the soil mass can offer to resist failure
and sliding along any plane inside it is called the … of the soil.
(a) strength
(b) shear strength
(c) compressive strength
(d) bearing capacity

2. The shear strength of soil is, in general, a function of …


(a) cohesion between the soil particles.
(b) frictional resistance between the soil particles.
(c) moisture content and pore water pressure in the soil mass.
(d) All of the above.

3. The concept of shear strength is not required directly to analyse the problems
related to …
(a) the bearing capacity of foundations.
(b) the stability of earth slopes.
(c) flow through the soil mass.
(d) lateral earth pressure from soils on retaining structures.

4. The soil mass fails because of the presence of …


(a) a critical combination of normal stress and shear stress.
(b) the maximum normal stress.
(c) the maximum shear stress.
(d) either the maximum normal stress or the maximum shear stress.

202 GEE2601/1
5. The Mohr-Coulomb failure criterion for soils is expressed in terms of …
(a) effective stress cohesion.
(b) effective angle of internal friction.
(c) Both (a) and (b).
(d) None of the above.

6. For sandy soils, the shear strength is …


(a) directly proportional to the effective stress.
(b) directly proportional to the square of the effective stress.
(c) inversely proportional to the effective stress.
(d) inversely proportional to the square of the effective stress.

7. If the effective stress within a sandy soil mass is zero, its shear strength will be
equal to …
(a) zero.
(b) half of the effective stress cohesion.
(c) effective stress cohesion.
(d) tangent of the effective angle of internal friction.

8. The value of effective stress cohesion c′ for inorganic silts is …


(a) zero.
(b) the same as c′ for sands.
(c) the same as c′ for clays.
(d) Both (a) and (b).

9. For normally consolidated clays, effective stress cohesion …


(a) c' = 0.
(b) c' ≈ 0.
(c) c' > 0.
(d) c' cannot be approximated to zero.

203 GEE2601/1
10. For dense cohesionless soils, the drained friction angle …
(a) φ′ < 35°.
(b) 35° < φ′ < 40°.
(c) 40° < φ′ < 45°.
(d) φ′ > 45°.

11. For normally consolidated clays, the drained friction angle …


(a) φ′ = 0.
(b) φ′ ≈ 0.
(c) φ′ < 20°.
(d) 20° < φ′ < 30°.

12. For cohesionless soils, the ratio of the effective major principal stress to the
effective minor principal stress is …
(a) tan(45° + φ'/2).
(b) tan(45° - φ'/2).
(c) tan2(45° + φ'/2).
(d) tan2(45° - φ'/2).

13. Which one of the following is the oldest and simplest form of shear test
arrangement?
(a) direct shear test
(b) triaxial shear test
(c) vane shear test
(d) Both (b) and (c).

14. In direct shear test, the failure within the soil takes place along …
(a) the bottom of the shear box.
(b) the plane of split of the shear box.

204 GEE2601/1
(c) the weakest soil layer.
(d) any plane depending on the soil density.

15. Select the correct statement based on the direct shear test results for sands.
(a) Peak shear strength is equal to ultimate shear strength.
(b) Peak shear strength is greater than ultimate shear strength.
(c) Peak shear strength is less than ultimate shear strength.
(d) None of the above.

16. On shear load application, which one of the following can increase in size?
(a) loose dry sand
(b) medium dry dense sand
(c) dense dry sand
(d) None of the above.

17. Which one of the following is the most reliable test method for determining shear
strength parameters?
(a) direct shear test
(b) triaxial shear test
(c) unconfined compression test
(d) Both (b) and (c).

18. In the standard triaxial test, a cylindrical soil specimen has …


(a) length = 38 mm, diameter = 38 mm.
(b) length = 76 mm, diameter = 76 mm.
(c) length = 76 mm, diameter = 38 mm.
(d) length = 38 mm, diameter = 76 mm.

19. Which of the following is not a standard type of the triaxial test?
(a) unconsolidated-undrained (UU) test

205 GEE2601/1
(b) unconsolidated-drained (UD) test
(c) consolidated-drained (CD) test
(d) consolidated-undrained (CU) test

20. For saturated soft soils, the Skemptons' pore pressure parameter …
(a) B = 1.
(b) B ≈ 1.
(c) B > 1.
(d) B < 1.

21. In a triaxial test for normally consolidated clay, chamber confining pressure =
100 kN/m2 and deviator stress = 130 kN/m2. What is the effective major principal
stress?
(a) 30 kN/m2
(b) 100 kN/m2
(c) 130 kN/m2
(d) 230 kN/m2

22. For the test results given in Q. 21, what will drained friction angle φ′ be?
(a) 0°
(b) 23.2°
(c) 45°
(d) 90°

23. For the test results given in Q. 21, what will the angle that the failure plane
makes with the major principal plane be?
(a) 0°
(b) 23.2°
(c) 45°
(d) 56.6°

206 GEE2601/1
24. Which of the following triaxial tests is completed quickly?
(a) UU test
(b) CU test
(c) CD test
(d) All of the above.

25. φ = 0 condition is observed in the …


(a) UU test on clays.
(b) UU test on saturated clays.
(c) CU test on saturated clays.
(d) CD test on saturated clays.

26. The unconfined compression test is a special type of … triaxial test.


(a) UU
(b) CU
(c) CD
(d) None of the above.

27. If the unconfined compression strength of clay is 40 kN/m2, its consistency can
be described as …
(a) soft.
(b) stiff.
(c) very stiff.
(d) hard.

28. If the unconfined compression strength of a clay sample is 100 kN/m2, its
undrained shear strength will be …
(a) 0.
(b) 50 kN/m2.

207 GEE2601/1
(c) 100 kN/m2.
(d) 200 kN/m2.

29. The sensitivity ratio of most clays ranges from about …


(a) 1 to 8.
(b) 10 to 80.
(c) 100 to 800.
(d) None of the above.

30. Select the incorrect statement.


(a) Most soils are partially thixotropic.
(b) Thixotropic is a time-dependent reversible process in which materials
soften when remoulded, but this loss of strength is gradually regained with
time when the materials are allowed to rest.
(c) The sensitivity ratio of quick clays is greater than 8.
(d) None of the above.

Activity 7.3: Typical exam questions

1. The following data was obtained from direct shear tests conducted on a soil
sample taken from a mine in Thabazimbi (Limpopo). Determine the value for c
and φ.
Normal Force Shear Force Area of Sample
Test
(kg) (kg) (mm)
1 4 5.80 5.5 x 5.5
2 8 6.94 5.5 x 5.5
3 12 8.1 5.5 x 5.5
4 16 9.6 5.5 x 5.5

2. The results from the triaxial test from the same mine in Thabazimbi (Limpopo)
are shown below. Determine the value for c and φ.

208 GEE2601/1
Confining Stress 𝛔𝛔3
Test Deviator Stress ∆𝝈𝝈𝟏𝟏 (kN/m2)
(kN/m2)
1 30 57
2 60 79
3 90 92

7.10 SOLUTIONS TO SELF-ASSESSMENT ACTIVITIES

Activity 7.1: Prescribed book exercises


1. Consult the prescribed book for answers.
2. Consult the prescribed book for answers.
3. Consult the prescribed book for answers.
4. Consult the prescribed book for answers.
5. Consult the prescribed book for answers.

Activity 7.2: Multiple-choice questions


1. (b)
2. (d)
3. (c)
4. (d)
5. (c)
6. (a)
7. (a)
8. (d)
9. (b)
10. (c)
11. (d)
12. (c)
13. (a)
14. (b)
15. (b)
16. (c)

209 GEE2601/1
17. (b)
18. (c)
19. (b)
20. (b)
21. (d)
22. (b)
23. (d)
24. (a)
25. (a)
26. (a)
27. (a)
28. (b)
29. (a)
30. (d)

Activity 7.3: Typical exam questions


1. c = 13 kN/m2 and φ = 19o
2. c = 12 kN/m2 and φ = 15o

7.11 SUMMARY
After carefully studying this study unit, you should be able to perform the various shear
strength tests on soils, as well as interpret the results to determine the shear strength
parameters and/or pore water pressure coefficients. You should also know the
differences in the behaviour of loose sand, dense sand, normally consolidated clay
and overconsolidated clay in the direct shear test and the different types of triaxial
tests.

Note:
• The direct shear test is used to determine the effective shear strength parameters
of dry or saturated sand and saturated clay. Unsaturated soil specimens have pore
water pressure of unknown magnitude, so effective stresses cannot be calculated.

210 GEE2601/1
• Peak and ultimate strength can be measured in the direct and triaxial shear tests,
but shear displacement is usually too small for the clay to reach residual shear
strength.
• Triaxial tests have the benefit over direct shear tests in that pore water pressure
can be monitored and dissipation controlled.
• Direct shear gives values of φ' about two or three degrees higher than triaxial shear,
because of different boundary conditions.
• Consolidated-drained and consolidated-undrained triaxial tests are used to
determine effective shear strength parameters of all saturated soils. The same
Mohr-Coulomb shear strength criterion, in terms of effective stress, applies to both
types of test, and identical values of c' and φ' are obtained, but be careful not to
compare peak and ultimate shear strength parameters. In CU tests the pore water
pressure is changed by deviator stress. This, in turn, changes the effective stress,
which determines the shear strength. In the CD test, the pore water pressure is
dissipated all the time, without any influence on effective stress. The shear
strengths obtained for the same soil from the two types of tests will therefore be
different. Yet, when effective stresses are calculated, the same shear strength
parameters apply.
• The unconfined compression test, vane shear test and UU triaxial test give more
or less the same undrained shear strength, cu, for any particular saturated clay,
with small differences caused by sample disturbance, softening, rate of shear
failure and other factors. Correction coefficients may be applied to allow for these
factors.

7.12 VIDEO CLIPS, SLIDES AND OTHER WEB-BASED


RESOURCES

For additional explanations on shear strength of soil, you can watch some video clips.
Web addresses/links to the video clips are available on the module site under

211 GEE2601/1
Additional Resources in a folder named Video clips, slides and other web-based
resources:

• Shear strength of soils:


https://www.youtube.com/watch?v=s5BX0mJFAus
• Module 38: Shear strength of soil:
https://www.youtube.com/watch?v=LViTxWFa7tQ
• Drained and undrained soil shear strength:
https://www.youtube.com/watch?v=ImHQLxuC_IU&spfreload=10
• Shear strength of soil: Soil mechanics:
https://www.youtube.com/watch?v=GN8V7UGQwb8&spfreload=10

REFERENCE WORKS AND FURTHER READING

Coulomb, CA. (1776). Essai sur une application des règles des maximis et minimis à
quelques problèmes de statique, relatifs à I'architecture. Paris: l'Imprimerie
Royale.

Das, BM. 2010. Principles of geotechnical engineering. 7th ed. Connecticut: Cengage
Learning.

Das, BM. 2013. Fundamentals of geotechnical engineering. 4th ed. Connecticut:


Cengage Learning.

Das, BM & Sivakugan, N. 2017. Fundamentals of geotechnical engineering. 5th ed.,


international edition. Connecticut: Cengage Learning.

Das, B.M. & Sobhan, K. (2014). Principles of Geotechnical Engineering, 8th Edition, SI
Edition. CENGAGE Learning. ISBN 13: 9788131521588 Connecticut, United
States of America.

* Page, figure, table and chapter numbers may differ on updated and revised versions in the prescribed
book. See online version of study guide for updates.

212 GEE2601/1
Study unit 1 8

SUBSURFACE EXPLORATION

8.1 INTRODUCTION

The earth’s crust, which supports buildings and other civil engineering structures,
consists of a very complex and variable combination of materials. These include soils,
rock, water, air and organic material. Since the structure and composition of the crust
are so variable and sometimes unpredictable, the geotechnical engineer and
technician must do a proper site and material investigation for every project to become
familiar with the conditions and materials. The material investigation must include a
test programme that includes in situ and laboratory tests to determine the properties
of the materials encountered. This unit is covered in detail in chapter 12 (pp. 336-
389*) in the prescribed book.

The purpose of subsurface exploration is discussed thoroughly in section 12.1 (pp.


336-337*). A detailed approach to how the geotechnical engineer and technician can
undertake such an exercise is discussed in detail in section 12.2 (pp. 337-340*).
Subsurface exploration comprises several steps which include the following:

• collection of preliminary information


• reconnaissance
• site investigation

The geotechnical design matches the stability and other requirements of the
foundation, embankment, earth dam, or excavation with the geological details and with
the materials and their properties, and can only be done with a reasonable knowledge
of everything involved. That is why most geotechnical designs are unique. It is

213 GEE2601/1
important to note that the geotechnical investigation and material testing is not finished
by the time the design is completed. As construction proceeds, new information
becomes available, and the design should be evaluated, adjusted, or even redone if
necessary. This is especially true if extensive excavation that allows access to the
geological formations and materials is involved.

You will come across the following term in this study unit:

• Subsurface exploration: The process of identifying the layers of deposits that


underlie a proposed structure and their physical characteristics.

8.2 LEARNING OUTCOMES

On completion of this study unit, you should be able to plan and execute a site
investigation and subsurface investigation programme. More specifically, you should
be able to

• correctly describe the sampling of soils and coring of rocks


• identify and explain a variety of in situ tests and derive approximate soil properties
from the results
• sketch the vital parts of sampling and testing equipment

The knowledge gained from studying this unit should be seen as an extension of the
knowledge you gained on soil profiling and geology in Geology and Soil Mechanics
(GSM1501) module.

To achieve the learning outcomes, work through chapter 12 (pp. 336-389*) in the
prescribed book. This part of the syllabus is purely theoretical. Study the whole of
chapter 12 for assignment and examination purposes, including the examples. Be
prepared to answer questions on any of the topics listed below:

• subsurface exploration programme


• exploratory borings in the field
• procedures for sampling soils
• observation of water levels

214 GEE2601/1
• vane shear test
• cone penetration test
• pressure meter test
• dilatometer test
• coring of rocks
• preparation of boring logs
• soil exploration report

Note:

The purpose of subsoil exploration includes the following:

• determining the nature of soil at the site and its stratification


• obtaining disturbed and undisturbed soil samples for visual identification and
appropriate laboratory tests
• determining the depth and nature of bedrock, if and when encountered
• performing some in situ field tests such as permeability tests, vane shear tests and
standard penetration tests
• observing drainage conditions from and into the site
• assessing any special construction problems in respect of the existing structure(s)
nearby
• determining the position of the water table

Read the summary of chapter 12 (pp. 336-389*) in the prescribed book. Attempt the
problems at the end of chapter 12 for self-assessment purposes. Note that you must
be able to sketch the vital parts of the sampling and testing equipment, but exact
dimensions of equipment, like sampling tubes, etc. are not required for assignment
and examination purposes.

215 GEE2601/1
8.3 SELF-ASSESSMENT ACTIVITIES

You now have the opportunity to apply what you have learnt in this unit. Answer the
following questions:

Activity 8.1: Prescribed book exercises


1. Do problems 12.1 to 12.18 (pp. 383-387*) and consult the solution book for
answers.
2. Do the critical thinking problem 12.19 (p. 387*) and consult the solution book
for answers.

Activity 8.2: Prescribed book exercises


1. The process of identifying the layers of deposits that underlie a proposed
structure and their physical characteristics is generally referred to as …
(a) subsurface determination.
(b) subsurface exploration.
(c) in situ testing.
(d) site investigation.

2. Subsurface exploration is necessary for …


(a) selecting the type and depth of foundation for a given structure.
(b) evaluating the load-bearing capacity of the foundation.
(c) estimating the probable settlement of the foundation.
(d) All of the above.

3. Which of the following phases of the subsurface exploration programme


consists of planning, making test boreholes and collecting soil samples at
desired intervals for subsequent observation and laboratory tests?
(a) collection of preliminary information
(b) reconnaissance
(c) site investigation
(d) None of the above.

216 GEE2601/1
4. The depth of borehole for a 3-storey building with a width of 30 m will be
approximately …
(a) 3.5 m.
(b) 6 m.
(c) 10 m.
(d) 16 m.

5. The minimum depth of core boring into bedrock is about …


(a) 1 m.
(b) 1.5 m.
(c) 3 m.
(d) 9 m.

6. The approximate spacing of boreholes for highway projects is …


(a) 10 – 30 m.
(b) 20 – 60 m.
(c) 40 – 80 m.
(d) 250 – 500 m.

7. The exploration cost should generally be … of the cost of the structure.


(a) 0.1 to 0.5%
(b) 1 to 2 %
(c) 1 to 5%
(d) 2 to 5%

8. Which of the following is the simplest method of making exploratory boreholes?


(a) auger boring
(b) wash boring
(c) percussion drilling
(d) rotary drilling

217 GEE2601/1
9. Drilling mud is a slurry of water and …
(a) china clay.
(b) bentonite.
(c) silty soil.
(d) silty-clayey soil.

10. Disturbed soil samples cannot be used for …


(a) grain-size analysis.
(b) specific gravity of soil solids.
(c) classification.
(d) hydraulic conductivity.

11. The soil sample collected by a sampler is generally considered to be undisturbed


when the area ratio is …
(a) less than 10%.
(b) equal to or less than 10%.
(c) greater than 10%.
(d) equal to or greater than 100%.

12. The split-spoon samples are generally taken at intervals of about …


(a) 0.5 m.
(b) 1.0 m.
(c) 1.5 m.
(d) 3.0 m.

13. If the numbers of blows required for the penetration of a split-spoon sampler
into the soil deposit for three 152.4 mm consecutive intervals are 8, 12 and 10,
the standard penetration number (N) will be …
(a) 18.
(b) 20.

218 GEE2601/1
(c) 22.
(d) 30.

14. The standard penetration number measured in the field is required to be


corrected for …
(a) hammer efficiency.
(b) borehole diameter and rod length.
(c) sampling method.
(d) All of the above.

15. If the corrected standard penetration number (N60) for a clayey soil deposit is
greater than 30, the soil consistency will be described as …
(a) soft.
(b) stiff.
(c) very stiff.
(d) hard.

16. If the corrected standard penetration number (N60) for a clayey soil deposit lies
between 0 and 5, its unconfined compressive strength will be in the range of …
(a) 0 – 50 kN/m2.
(b) 50 – 200 kN/m2.
(c) 200 – 400 kN/m2.
(d) > 400 kN/m2.

17. The standard penetration number measured in the field is corrected for the
effective overburden pressure for …
(a) clayey soils.
(b) silty soils.
(c) sandy soil.
(d) All of the above.

219 GEE2601/1
18. For a dense sandy soil deposit, the standard penetration number is generally …
(a) less than 5.
(b) between 5 and 10.
(c) between 10 and 30.
(d) between 30 and 50.

19. A Shelby tube is …


(a) a thin wall steel tube.
(b) used to obtain undisturbed clayey soil samples.
(c) sharpened at its bottom end.
(d) All of the above

20. After completion of the boring in a highly permeable soil deposit, the level of
water in the borehole generally stabilises in …
(a) 1 h.
(b) 24 h.
(c) 1 week.
(d) more than 1 week.

21. Which of the following soil parameters is obtained directly from the field vane
shear test?
(a) drained cohesion
(b) undrained cohesion
(c) unconfined compression strength
(d) angle of shearing resistance

22. The undrained cohesion obtained from the field vane shear test is generally …
(a) safe for foundation design.
(b) unsafe for foundation design.

220 GEE2601/1
(c) multiplied by a factor greater than 1 for foundation design.
(d) Both (b) and (c).

23. Which of the following field tests does not require a borehole?
(a) cone penetration test (CPT)
(b) standard penetration test (SPT)
(c) pressure meter test (PMT)
(d) All of the above.

24. When the rock core samples are recovered, a recovery ratio of 1 indicates the
presence of …
(a) highly fractured rock.
(b) highly jointed rock.
(c) intact rock.
(d) None of the above.

25. The NX size rock core sample has an approximate diameter of …


(a) 22 mm.
(b) 28 mm.
(c) 41 mm.
(d) 54 mm.

26. If the rock quality designation (RQD) is in the range of 0.25 to 0.5, the rock
quality can be described as …
(a) very poor.
(b) poor.
(c) good.
(d) excellent.

221 GEE2601/1
27. Select the incorrect statement.
(a) Geophysical exploration techniques allow rapid coverage of large
areas.
(b) Geophysical exploration techniques are more expensive than
conventional exploration by drilling.
(c) Geophysical exploration techniques are generally used for
preliminary work only.
(d) None of the above.

28. Which of the following is not a geophysical exploration technique?


(a) drilling
(b) seismic refraction survey
(c) cross-hole seismic refraction survey
(d) resistivity survey

29. Which of the following earth materials has a low electrical resistivity?
(a) sand
(b) gravel
(c) saturated clay
(d) rock

30. The ratio of velocity of P waves to that of S waves is …


(a) 0.
(b) 1.
(c) less than 1.
(d) greater than 1.

222 GEE2601/1
Activity 8.3: Typical exam questions

1. Briefly explain the purpose of subsurface exploration.


2. Explain in detail how you would collect preliminary information for a road
construction site in Limpopo.
3. You have been appointed as a boring contractor in a local mine in Burgersfort.
As a specialist boring engineer, you have to explain in writing to the client the
different methods that are available for use in the industry.
(a) Explain to the client the difference between auger boring and post hole
auger.
(b) Under what conditions of continuous flight augers be ideal for use?
(c) Describe wash boring to the client.
(d) Explain to the client the difference between rotary drilling and drilling mud.

8.4 SOLUTIONS TO SELF-ASSESSMENT ACTIVITIES

Activity 8.1: Prescribed book exercises

1. Consult the solution book for answers.


2. Consult the solution book for answers.

Activity 8.2: Multiple-choice questions

1. (b)
2. (d)
3. (c)
4. (c)
5. (c)
6. (d)
7. (a)
8. (a)
9. (b)
10. (d)

223 GEE2601/1
11. (b)
12. (c)
13. (c)
14. (d)
15. (d)
16. (a)
17. (c)
18. (d)
19. (d)
20. (b)
21. (b)
22. (d)
23. (a)
24. (c)
25. (d)
26. (b)
27. (b)
28. (a)
29. (c)
30. (d)

Activity 8.3: Typical exam questions

1. p. 336* in prescribed book.


2. p. 337* in the prescribed book.
3. pp. 341-344* in the prescribed book.

8.5 SUMMARY

You should now be able to plan and execute a site investigation, obtain undisturbed
samples, do a variety of in situ soil tests and interpret the results to determine
approximate soil properties.

224 GEE2601/1
8.6 VIDEO CLIPS, SLIDES AND OTHER WEB-BASED
RESOURCES

For additional explanations on subsurface exploration, you can watch some video
clips. Web addresses/links to the video clips are available on the module site under
Additional Resources in a folder named Video clips, slides and other web-based
resources.

• Soil sampling techniques: Hand auger & direct push probe:


https://www.youtube.com/watch?v=YXWp79UKZf4
• How to take a soil sample:
https://www.youtube.com/watch?v=3_U9Z3fy0Ig
• Drilling core samples in the field:
https://www.youtube.com/watch?v=EHzYXNtvAHA
• An introduction to drilling and sampling in geotechnical practice:
https://www.youtube.com/watch?v=IGk1HI1NGug.

REFERENCE WORKS AND FURTHER READING

Das, BM. 2010. Principles of geotechnical engineering. 7th ed. Connecticut: Cengage
Learning.

Das, BM. 2013. Fundamentals of geotechnical engineering. 4th ed. Connecticut:


Cengage Learning.

Das, BM & Sivakugan, N. 2017. Fundamentals of geotechnical engineering. 5th ed.,


international edition. Connecticut: Cengage Learning.

* Page, figure, table and chapter numbers may differ on updated and revised versions of the prescribed
book. See online version of study guide for updates.

225 GEE2601/1
Notes

....................................................................................................................................

....................................................................................................................................

....................................................................................................................................

....................................................................................................................................

....................................................................................................................................

....................................................................................................................................

....................................................................................................................................

....................................................................................................................................

....................................................................................................................................

....................................................................................................................................

....................................................................................................................................

....................................................................................................................................

....................................................................................................................................

....................................................................................................................................

....................................................................................................................................

....................................................................................................................................

....................................................................................................................................

....................................................................................................................................

....................................................................................................................................

....................................................................................................................................

....................................................................................................................................

226 GEE2601/1
Study unit 1 9

LATERAL EARTH PRESSURE

9.1 PURPOSE OF STUDY UNIT

If an element of material is compressed by a stress in any direction, it will expand in


the orthogonal directions. If expansion is totally or partially prevented, a compressive
stress will develop in the material in that orthogonal direction, and the ratio between
this stress and the applied stress is determined by the properties of the material as
well as by the extent to which the expansion is prevented. The situation described
above is the source of lateral earth pressure. It exists within the soil mass, and at the
interface between soil and any physical near-vertical boundary. The soil is usually
stressed in the vertical direction by its weight, and the lateral pressure will act against
retaining walls, basement walls, bulkheads, bridge abutments, buried conduits and
other structures with which the soil is in contact.

The magnitude of the lateral stress depends on the shear characteristics of the soil,
lateral strain conditions, pore water pressure and the state of equilibrium of the soil.
These, in turn, depend on the drainage conditions, the interaction between the
structure and the soil, and the displacement of the interface. To design structures with
which soils are in contact, geotechnical technicians must be able to evaluate the
stresses acting on the interfaces to the best of their ability, taking into account the
factors that have a bearing on their magnitude. The lateral interaction between soil
and structure is extremely complicated, and you must be very careful not to overlook
any aspect that could influence the calculated pressures.

227 GEE2601/1
The lateral pressure theories by Rankine and Coulomb, which are in general use, are
imperfect because they are based on several simplifying assumptions, and both may
render inaccurate lateral stresses if applied to a problem for which they not suitable.
In this course we will highlight the positive and negative aspects of the theories. Note
that chapter 9 in the prescribed book only covers the basic earth pressure theories
and shows how these are used to calculate earth pressure distributions for different
conditions. Stability calculations for different types of retaining structures are
discussed in chapter 12. For that reason, the notes on chapter 9 contain only a few
examples. Several more will be included in the unit on chapter 12.

9.2 DEFINITIONS

You will come across the following terms in this study unit:

• At-rest lateral earth pressure: A wall may be restrained from movement. For
example, a basement wall is restrained from movement due to a slab of the
basement, and the lateral earth forces in this case can be termed Po (see figure
9.1).
• Active lateral earth pressure: In cases where the wall is free from its upper edge
(retaining wall), the wall may move away from the soil that is retained with distance
"+ΔH" (i.e. the soil pushes the wall away). This means the soil is active and the
force of this pushing is called active force and termed Pa (see figure 9.1).
• Passive lateral earth pressure: For the wall shown below (retaining wall) on the
left side there is soil with a height less than the soil on the right and, as we
mentioned above, the soil on the right will push the wall away, so the wall will be
pushed into the soil on the left (i.e. soil compresses the soil on the left). This means
the soil has a passive effect and the force in this case is called passive force and
termed PP (see figure 9.1).

228 GEE2601/1
FIGURE 9.1
Three types of lateral earth pressure (LEP) (Al-Agha 2015)

9.3 LEARNING OUTCOMES


On completion of this study unit, you should be able to calculate lateral stresses and
forces against retaining structures for a variety of geometries and soil properties. More
specifically, you should be able to

• calculate lateral earth pressure at rest for dry and submerged soils using the
theories of Rankine and Coulomb
• calculate lateral earth pressure at active and passive pressures using the theories
of Rankine and Coulomb
• correctly include the effects of self-weight, submergence, simple surcharge loads
and interface friction in calculations
• select the correct theory for a specific problem

229 GEE2601/1
To achieve the learning outcomes in this unit, work through the following topics:

• earth pressure at rest


• Rankine's and Coulomb's theories of active and passive pressure for cohesionless
and cohesive soils
• the effects of own weight, partial submergence, surcharge and interface friction

9.4 EARTH PRESSURE AT REST

Chapter 14 (pp. 446-489*) in the prescribed book covers this topic. The ideal at-rest
condition can only be found within a perfect half-space in which gravity is the only
force. This means that the material extends to infinity in both horizontal directions and
downwards below the horizontal surface and it is completely homogeneous. No
surcharge pressure acts at the surface. At-rest earth pressure would then be defined
as the horizontal pressure inside the half-space.

Examples of earth-retaining structures are shown in figures 9.2 and 9.3.

FIGURE 9.2 FIGURE 9.3


Retaining wall (Reinforced Earth 2018) Retaining wall (Shutterstock 2018)

230 GEE2601/1
9.4.1 Dry homogeneous soil

The at-rest lateral pressure is assumed to also act on any vertical section near a
frictionless, infinitely high, completely rigid, static, vertical wall, as well as on the wall
surface itself, as shown in figure 14.2 (p. 448*) in the prescribed book. The soil surface
must be horizontal. In addition, geotechnical engineers assume that the lateral stress
on a wall of finite length can be approximated by the at-rest value, as long as all other
conditions mentioned above are met. Equation 14.1 (p. 447*) in the prescribed book
defines the ratio of the horizontal effective stress (the at-rest lateral pressure) to the
vertical effective stress as the coefficient of earth pressure at rest, K0. Vertical stress
and at-rest pressure both increase linearly with depth, like in figure 14.3 (p. 449*) in
the prescribed book. The force per unit length that acts against the wall can be
calculated from the area of the pressure diagram or from integration of equation 14.3
(p. 443*) over the height H of the wall. The result is equation 14.9 (p. 449*) in the
prescribed book, and the force acts at H/3 from the bottom. The value of the coefficient
K0 cannot be derived from basic principles, and is governed by many factors such as
grain size distribution, stress history, ageing and mineralogy. Various researchers
have come up with empirical relationships between K0 and other soil properties. The
most widely accepted ones are given by equations 14.4, 14.5 and 14.6 (p. 448*) in
the prescribed book. Note that the effective friction angle, φ', must be used in equation
14.3 (p. 448*).

Example 9.1

A 6 m high vertical retaining wall supports coarse-grained soil with zero cohesion with
the following properties:

• φ' = 350
• γ = 16 kN/m3
• The soil is dry and the surface is horizontal

Determine the distribution of at-rest lateral pressure as well as the force per unit length
on the wall.

231 GEE2601/1
Solution

First determine the value of the coefficient K0:

• Equation 14.3 (p. 448*) in the prescribed book should be used for granular
material.
• K0 = 1 – sin φ' = 1 – sin35° = 0.426

Next, calculate the pressure at the bottom of the wall:

• From equation 14.2 (p. 447*) in the prescribed book


• σh' = 0.426 × 16 × 6 = 40.9 kN/m2

The pressure distribution is like the one in figure 14.3 (p. 449) in the prescribed book,
starting at σh' = 0 at the top of the wall to σh' = 40.9 kN/m2 at the bottom. Finally, the
force may be calculated using equation 14.9 (p. 449*) in the prescribed book:

• P0 = 0.5K0γH2 = 0.5 x 0.426 x 16 x 62 = 122,7 kN/m

Calculating the triangle of the pressure diagram gives the same result. The point of
application of the force is H/3 = 2 m above the bottom of the wall.

9.4.2 Partially submerged soil

It is assumed that at rest, conditions will prevail in non-homogeneous soils, if the


condition of zero lateral strain is met and the surface is horizontal. One form of non-
homogeneity is partial submergence. Since K0 is defined in terms of effective stresses,
the buoyant unit weight of the soil must be used in the submerged part. The result is
that the effective lateral pressure increase with depth will be different below the water
table, and the diagram consists of two linear parts, as can be seen in figure 14.4(a)
(p. 450*) in the prescribed book. To this effective lateral stress must be added the
hydrostatic pore water pressure, as illustrated by figure 14.4(b) (p. 450*) in the
prescribed book. If the distribution of total lateral stress is desired, the effective at-rest
lateral pressure and the pore water pressure may be combined into a single diagram,
as in figure 14.4(c) (p. 450*) in the prescribed book, but it is not necessary to do this.

232 GEE2601/1
All calculations of forces and their points of application can conveniently be done from
the areas and shapes of the separate diagrams.

Example 9.2

Redo example 9.1 but with the groundwater water table at 4 m above the bottom of
the wall. The saturated unit weight of the soil below the water table is 21 kN/m3.

Solution

• The value of K0 remains at 0.426 as in example 9.1.

At the level of the water table the following can be calculated:

• σ0' = 16 × 2 = 32 kN/m2
• σh' = K0σ0' = 0.462 × 32 = 14.8 kN/m2

At the bottom of the wall:

• σ0' = 16 × 2 + (21 - 10) × 4 = 76 kN/m2 (assuming that γw = 10 kN/m3)


• σh' = K0σ0' = 0.462 × 76 = 35.1 kN/m2

The pressure diagram is similar to figure 14.4(a) (p. 450*) in the prescribed book. At
the bottom of the wall the pore water pressure is

• u = 10 x 4 = 40 kN/m2

The pore water pressure diagram is triangular, like in figure 14.4(b) (p. 450*) in the
prescribed book. Areas and sub-areas of the pressure diagrams can be used to
calculate forces and their points of application, and the resultant force can be
determined from these in the usual way.

9.5 RANKINE'S THEORY OF ACTIVE AND PASSIVE EARTH


PRESSURE

Section 14.3 (pp. 451-458*) in the prescribed book covers this topic. Any element in
a dry soil mass that is in the at-rest condition is acted upon by the vertical stress from
the overburden and the at-rest lateral pressure. These are principal stresses. If the

233 GEE2601/1
soil is compressed or allowed to expand in one of the horizontal directions, without
inducing shear stress on the principal planes, the vertical stress will remain the same.
At the same time the horizontal stress in the direction of the strain will increase if the
horizontal strain is compressive and decrease if the soil expands. The ratio of
horizontal to vertical stress will therefore deviate from K0. The increase or decrease in
the horizontal stress in a soil element cannot continue indefinitely, because the change
in the principal stress will increase shear stresses within the element until shear failure
is reached. The condition of plastic equilibrium is reached when soil is on the verge of
failure everywhere.

Rankine investigated the stresses within a soil mass in the state of plastic equilibrium
to formulate his theories of earth pressure. The magnitude of strain required to render
the soil in the state of plastic equilibrium varies according to the type of material
involved. Much higher compressive strain is needed to reach plastic equilibrium than
would be necessary to achieve the same result if the strain were tensile. The
prescribed book gives an excellent overview of the effects of wall yielding on the lateral
earth pressure.

9.5.1 Active case

If soil at rest is allowed to expand laterally by movement of a wall away from the soil
mass, settlement will occur in the soil, but the wall is assumed to be frictionless, so
shear stress is not generated on the interface. Because the soil surface is horizontal,
the vertical stress σ0' remains the major principal stress. It is caused by the
overburden, and its value should remain the same within a given element, whereas
the horizontal stress in the direction of lateral strain, or minor principal stress, is
decreased from its at-rest value. The result is that Mohr's circle will become larger,
expanding towards the left as can be seen in figure 14.5 (pp. 452-453*) in the
prescribed book, until it touches the failure envelope. The horizontal stress is now at
its minimum value and Rankine called it the active pressure.

234 GEE2601/1
A free-standing retaining wall supporting the soil must be designed to withstand the
active lateral pressure with a certain margin of safety. However, if the wall is prevented
from yielding, like when it is part of a structure, it will have to resist the at-rest pressure.
The relationship between the effective horizontal and vertical stresses can easily be
derived from Mohr's circle at failure for a general soil with cohesion and angle of
internal friction.

1 − sin φ ' cos φ '  φ'   φ' 


σ 'a = σ '0 − 2c ' = σ '
0 tan 2
 45 −  − 2c ' tan 45 −
 


1 + sin φ ' 1 + sin φ '  2   2  (9.1)
Where:

• σa’ = Active lateral pressure

• ɸ’ = Frictional angle of soil

• c’ = Cohesion of soil

• σ0’ = Pressure due to unit weight of soil

If Ka is defined as

 φ' 
K a = tan 2  45 − 

 2 
• (9.2)
Then

σ 'a = σ '0K a − 2c ' K a


• (9.3)
Ka is the coefficient of Rankine's active earth pressure and it is always less than or
equal to 1. The effective shear strength parameter φ' is used to calculate the value of
K0, and therefore equations 9.1 and 9.3 are in terms of effective stress. But the
derivation can be made in terms of total stress as well, if φu is used to calculate K0,
and cu is used in the equations instead of c'. σa and σ0 will then be total stresses and
pore water pressure is not taken into account separately.

In a dry soil mass without any surcharge, σ0'= γz, where z is the depth from the surface,
and γ is the unit weight of the soil. Equation 9.3 suggests that the active earth pressure,

235 GEE2601/1
σa', increases linearly with depth, just as the at-rest pressure, but the second term
indicates that active pressure will be negative, or tensile, close to the surface, unless
the cohesion is zero. The typical distribution is shown in figure 14.5(c) (p. 453*) in the
prescribed book. The depth z0 below which the active pressure is compressive can be
derived by inserting zero for the value of σa' into equation 9.3.

2c '
z0 =
γ Ka
• (9.4)
Soil cannot withstand tensile stresses for a prolonged period and the negative portion
of the active pressure diagram is ignored in stability analyses. The soil will probably
develop tensile cracks or lose contact with the wall over the depth z0. So, excavation
can only be done in soil to depth z0 without bracing.

Note:

• The value of z0 will be modified by the presence of surcharge pressure, q,


because the vertical effective stress then becomes σ0'= γz + q.

Example 9.3

An 8 m high vertical wall supports a dry soil with a horizontal surface. The soil
properties are as follows:

• c' = 5 kN/m2
• φ' = 300
• γ = 15 kN/m3

If the wall moves away from the soil, inducing plastic equilibrium in the soil, determine
the active earth pressure distribution on the wall.

Solution

The pressure diagram will have the shape of figure 14.5(c) (p. 453*) in the prescribed
book. All that is required is to calculate the active pressure at the top and bottom of

236 GEE2601/1
the wall. Equation 9.3 will be used for this purpose. In addition, the depth to zero
pressure will be calculated as well as the resultant force against the wall. Determine
the value of Rankine's Ka from equation 14.19 (p. 455*) in the prescribed book:

 φ'  30 
K a = tan 2  45 −  = tan 2  45 −  = 0,33333
2   2 
 

At the top:

• σ0' = 0
σ 'a = σ '0 K a − 2c ' K a = 0 − 2 × 5 × 0,33333 =
• - 5.77 kN/m2

At the bottom:

• σ0' = 15 × 8 = 120kN/m2
• σ 'a = σ '0 K a − 2c ' K a = 120 × 0,33333 − 2 × 5 × 0,33333 = 34.23 kN/m2

The level to zero lateral pressure is calculated from equation 9.4:

2c ' 2×5
z0 = = = 1,155 m
γ Ka 15 0,33333

In the calculation of the force the negative part of the diagram will be ignored, because
soil cannot withstand tensile pressure. The effective height of the wall is therefore

• 8 – 1.155 = 6.845 m. From the triangular distribution:


• Pa = 0.5 x 34.23 x 6.845 = 117.2 kN/m
• The point of application is 6.845/3 = 2.28 m above the bottom

9.5.2 Passive case

If a frictionless vertical wall is pushed into the soil mass laterally, the soil will
experience compressive strain and the horizontal stress will increase from the at-rest
value. The vertical stress remains constant at σ0' = γz. As a result the horizontal stress
will become the major principal stress and the vertical stress the minor principal stress
by the time plastic equilibrium is reached. This is illustrated by Mohr's circles in figure
14.6(b) (p. 455*) in the prescribed book. The soil is now in the passive state and the

237 GEE2601/1
lateral stress reaches its maximum value, called the passive Rankine earth pressure,
σp'. The relationship between the principal stresses is again derived from the geometry
of Mohr's circle at failure, and is given by

2 φ'   φ' 
σ p' = σ '0 
tan  45 +  + 2c ' tan 45 + 
 
 2   2 
(9.5)

If Kp is defined as

 φ' 
• K p = tan 2  45 + 

(9.6)
 2 

Then

σ p' = σ '0K p + 2c ' K p


• (9.7)

Where:

• Kp = Rankine's coefficient of passive earth pressure

Clearly Kp is the inverse of Ka and its value is greater than or equal to 1.

Contrary to the active pressure, the passive pressure is positive everywhere but it also
increases linearly with depth. This is illustrated in figure 14.6(c) (p. 455*) in the
prescribed book. Passive pressure is often relied upon to support a retaining structure
against horizontal movement and overturning.

Example 9.4

Repeat example 9.3 but this time the wall is pushed into the soil to induce plastic
equilibrium. Determine Rankine's passive earth pressure distribution.

Solution

Calculated Rankine's passive coefficient Kp from equation 9.19 in the prescribed book:

 φ'   30 
K p = tan 2  45 +  = tan 2  45 +  = 3,0
 2   2 

238 GEE2601/1
At the surface:

• σ0' = 0
• σ p' = σ '0K p + 2c ' K p = 0 + 2 × 5 × 3,0 = 17.32 kN/m2

At the bottom of the wall:

• σ0' = 15 × 8 = 120 kN/m2


σ p' = σ '0K p + 2c ' K p = 120 × 3,0 + 2 × 5 × 3,0
• = 360.00 + 17.32 = 377.32 kN/m2

The pressure diagram will be similar to the one in figure 14.6(c) (p. 455*) in the
prescribed book and forces may be calculated from the areas of the sub-areas:

For the rectangular sub-area:

• Pp1 = 17.32 × 8 = 138.56 kN/m and the point of application is at H/2 = 4 m


For the triangular area:

• Pp2 = 0.5 × 360 × 8 = 1 440 kN/m and the point of application is H/3 = 2.667 m
• Resultant force = 138.56 + 1 440 = 1 578.56 kN/m
• Point of application i=s (138.56 × 4 + 1 440 × 2.667)/1 578.56 = 2.784 m

9.5.3 Effects of surcharge, submergence and a layered soil profile

Section 14.4 (pp. 458-466*) in the prescribed book covers this topic. Up to this point
we have only considered active and passive pressures against a retaining wall in
contact with homogeneous dry soil with a horizontal surface. Section 14.4 in the
prescribed book expands Rankine's theories to include surcharge, submergence and
soil layers with different shear strength parameters. The text is very comprehensive
and no additional explanations will be provided here. Instead, an extensive example
will be used to illustrate the calculations, but you need to work through examples 14.1
to 14.4 (pp. 466-471*) in the prescribed book first in order to become familiar with the
calculations and especially the shapes of the lateral earth pressure diagrams.

239 GEE2601/1
Note:

• The presence of the surcharge increases the vertical stress and this decreases the
depth of the tensile crack (or zone of tensile active pressure) in example 14.4 (p.
469-471*). A different formula may be derived for z0 by putting σ0' = γz + q in
equation 9.3 to allow for the surcharge, but the best way would be to calculate the
values of σa' at the top and at some convenient depth within the layer and
interpolate to find the depth at which σa' becomes zero.
• The active pressure diagram is discontinuous and changes slope at the contact
between soils because Ka is a function of φ', and is different for each soil.
• Pore water pressure is considered in addition to the active pressure, which is in
terms of effective stress. From the numbers you can see that pore water pressure
may exceed active pressure, and many retaining structures fail as a result of water
pressure when the water table rises to unforeseen levels.

Example 9.5

A retaining wall is embedded into the ground to a depth of 3 m. On one side of the wall
a granular backfill is placed to 5 m above natural ground level, and on the opposite
side the same granular material is placed to 1 m above natural ground level. On both
sides a 10 kN/m2 surcharge is anticipated. The 5 m backfill will become submerged to
2 m below the backfilled surface, and a water table may be found at natural ground
level on the opposite side. The in situ clay will prevent seepage below the wall. Above
the water tables the soil may be assumed to be dry. The geometry of the wall with the
backfills and the positions of the water tables are shown in figure 9.4. The following
properties have been determined for the two soils involved:

• For the in situ soil φ' = 300


• c' = 10 kN/m2
• γsat = 20 kN/m3
• For the backfill φ' = 370
• c' = 0 kN/m2

240 GEE2601/1
• γsat = 20 kN/m3 and γdry = 16 kN/m3
• Assume that the unit weight of water is 10 kN/m2

Calculate and draw the distributions of the lateral earth pressures on both sides of the
wall.

FIGURE 9.4
Geometry of retaining wall for example 9.5

Solution

The wall will tend to deflect to the left. If we assume lateral movement and/or rotation
about the lower end of the wall, the soil on the right-hand side will expand to follow the
movement of the wall while the soil on the left-hand side will be compressed. The soil
surface is horizontal and the wall is vertical, and Rankine's theories may therefore be
used to calculate the lateral pressure distributions if sufficient strain can develop on
both sides for the soil to reach plastic equilibrium. Rankine's active state will exist on
the right-hand side and his passive state on the left-hand side.

241 GEE2601/1
We will calculate the lateral pressure on the active side first. It is very useful to think
about the probable pressure distribution before attempting any calculations because
this may prevent mistakes. From equation 9.3 the lateral pressure is determined by
vertical stress, the value of Ka and the cohesion. In a soil for which the unit weight is
constant, the vertical pressure will increase linearly with depth. If Ka and c' are constant
too, the lateral pressure within the layer will also increase linearly with depth, resulting
in a straight-line diagram of lateral pressure like in the examples in the prescribed
book. Therefore, lateral pressure has only to be calculated at the tops and bottoms of
layers with constant properties. These values are plotted and connected by straight
lines to complete the diagram.

In this example we will have to calculate the lateral pressure at the following depths:
the surface of the granular fill, the water table where the unit weight changes from dry
to submerged, the natural ground level (NGL) where the values of Ka and c' change,
and the bottom of the wall. The calculations can now be done. First calculate
coefficients of active earth pressure for all materials involved from equation 9.2.

For the granular fill:

• Ka = tan2 (45 – φ'/2) = tan2 (45 –37/2) = 0.2486


• c' = 0 kN/m2
For the in situ soil:

• Ka = tan2 (45 – φ'/2) = tan2 (45 – 30/2) = 0.3333

• c' = 10 kN/m2

Next, calculate the values of the vertical effective stress and the active lateral stress
at the four positions mentioned above, using equation 9.3.

At the surface:

• σo' = 10 kN/m2 caused by the surcharge

The material involved is the granular fill:

• σa' = σo'Ka = 10 × 0.2486 = 2.5 kN/m2

242 GEE2601/1
At the water table:

• σo' = 10 + (16 × 2) = 42 kN/m2

The material involved is the granular fill:

• σa' = 42 × 0.2486 = 10.4 kN/m2

At the NGL:

• σo' = 42 + (20 - 10) × 3 = 72 kN/m2

Both the granular fill and the in situ material are involved:

• σa' = 72 × 0.2486 = 17.9 kN/m2 in the fill

• σa' = 72 × 0.3333 – 2 × 10 × (0.3333)1/2 = 12.5 kN/m2 in the in situ soil

At the bottom:

• σo' = 72 + (20 – 10) × 3 = 102 kN/m2

The material involved is the in situ soil:

• σa' = 102 × 0.3333 – 2 × 10 × (0.3333)1/2 = 22.5 kN/m2

The calculated active lateral pressure values are plotted on the right-hand side of
figure 9.5 and connected by straight lines. The final step is to add the pore water
pressure diagram to the active side. Pore water pressure is assumed to be hydrostatic
below the water table, increasing from zero at the water table to 60 kN/m2 at the bottom
of the wall. The diagram is shown in figure 9.5. The passive lateral pressure will be
calculated next. Again, take some time to think about the possible shape of the
distribution. The water table is at the interface between the in situ soil and the fill. In
effect that leaves us with two homogeneous layers, and it will only be necessary to
calculate the pressure at the surface, the interface and the bottom.

Calculate the coefficients of passive earth pressure, using equation 9.6.

For the granular fill:

• Kp = tan2 (45 + φ'/2) = tan2 (45 + 37/2) = 4.0228


• c' = 0kN/m2

243 GEE2601/1
For the in situ soil:

• Kp = tan2 (45 + φ'/2) = tan2 (45 + 30/2) = 3.0000


• c' = 10 kN/m2

Next, calculate the values of the vertical effective stress and the passive lateral stress
at the three positions mentioned above, using equation 9.7. At the surface:

• σo' = 10 kN/m2 caused by the surcharge

The material involved is the granular fill:

• σa' = σo'Ka = 10 × 4.0228 = 40.2 kN/m2

At the NGL/WT:

• σo' = 10 + 16 × 1 = 26 kN/m2

Both the granular fill and the in situ material are involved:

• σa' = 26 × 4.0228 = 104.6 kN/m2 in the fill


• σa' = 26 × 3.0000 + 2 × 10 × (3,0000)1/2 = 112.6 kN/m2 in the in situ soil

At the bottom:

• σo' = 26 + (20 – 10) × 3 = 56 kN/m2

The material involved is the in situ soil:

• σa' = 56 × 3,0000 – 2 × 10 × (3.0000)1/2 = 204.6 kN/m2

The calculated passive lateral pressure values are plotted on the left-hand side of
figure 9.5 and connected by straight lines. The final step is to add the pore water
pressure diagram to the passive side. Pore water pressure is assumed to be
hydrostatic below the water table, increasing from zero at the water table to 30 kN/m2
at the bottom of the wall. The diagram is shown in figure 9.5. This concludes the
calculations for the example.

244 GEE2601/1
FIGURE 9.5
Rankine's lateral pressures on retaining wall for example 9.5

Note:

• To give a step-by-step explanation, all the calculations were done on separate


lines. But, once you get to know the procedures, the calculations may be done in
tabular form. This saves time and writing space, and all numbers are kept together
on about half a page. We will illustrate this in the stability analyses of retaining
structures in study unit 12.

9.5.4 Effects of wall friction, an inclined soil surface and a non-vertical wall

Section 14.5 (pp. 471-472*) in the prescribed book covers this. When the surface of
the soil is not horizontal, vertical planes and the interface between soil and wall are no
longer principal planes and different expressions must be derived for Rankine's Ka and
Kp. Equation 14.45 (p. 471*) in the prescribed book can be used to calculate Ka. Note
that this expression is only valid for cohesionless soil. Kp is the inverse of Ka as before.
If the soil has cohesion, the equation is invalid. The same applies when α approaches
φ'. During derivation of equation 14.45 (p. 471*) in the prescribed book, it is assumed
that the lateral pressure, as well as the resultant active force per unit length, is parallel
to the inclined soil surface, which implies that the wall can no longer be frictionless.

245 GEE2601/1
The angle of friction between the soil and the wall must at least be equal to α for the
equation to be valid.

9.5.5 Accuracy of Rankine's theory

Since the assumption of a frictionless retaining wall is an idealisation, the effects of


wall friction on lateral pressure should be examined. We have already discussed the
need for friction on the interface in the case of an inclined soil surface. If the interface
friction angle (denoted by δ) is lower than α, the active pressure calculated by means
of Ka from equation 14.45 (p. 471*) in the prescribed book will be an underestimation
of the actual value. At the same time, the passive pressure calculated from Kp = 1/Ka
will be an overestimation. These errors may lead to instability of a retaining wall, so
rather not use Rankine's theory when the soil surface is steeper than φ'/3. On the other
hand, if the soil surface is level, or near to it, Rankine's theory ignores the contributions
from any interface friction towards supporting the soil mass on the active side and
increasing the passive pressure. This means that Rankine's theory usually
overestimates active pressure and underestimates passive pressure. The theory is
therefore on the conservative side, and it can be used with confidence.

9.6 COULOMB'S EARTH PRESSURE THEORY

Section 14.6 (pp. 472-481*) in the prescribed book covers this topic.

9.6.1 Active case

Coulomb considered the force equilibrium of a wedge of soil in contact with the soil.
Figure 14.17 (p. 474*) in the prescribed book shows the vector diagram for a wedge
ABC of cohesionless soil on the active side of the wall. If the wall deflects away from
the soil by shifting or tilting, this will induce movement of the soil behind the wall,
causing shear failure to occur on the wall surface AB and the surface BC within the
soil. The movement of the wedge will include a downward component so that forces F
and Pa are oriented at an angle φ' below the normal to surface BC and at an angle δ

246 GEE2601/1
below the normal to surface AB, respectively. The angle δ is known as the angle of
interface friction. It has the same physical meaning as φ', and can be measured in a
direct shear test in which the lower half of the shear box is replaced with a similarly
sized piece of the wall material, like concrete or steel. As you can see from the figure,
Coulomb's theory provides for non-vertical walls and sloping soil surfaces, and it
makes no assumption about a frictionless wall, since the correct value of δ is
incorporated.

In Coulomb's original theory several trial failure wedges were considered by varying
the inclination of surface BC and vector diagrams were drawn for all of them. The
wedge that gave the largest value of the force Pa was regarded as the critical one, and
that value of Pa was taken as the active force on the wall, but in the opposite direction.
This graphical solution was very powerful and could include irregular soil surfaces,
non-uniform surcharge, cohesion, cracks to any specified depth, as well as adhesion
between soil and wall. Different soil layers and partial submergence could be included
as well, although the process was rather time-consuming. If you are interested in the
graphical method, you may find it in many older geotechnical textbooks. Note that the
procedure only gave the magnitude of the force Pa, but with no indication of its point
of application. In the absence of surcharge and cohesion, the assumption was that the
point of application would be at H/3 from the bottom of the wall.

With the availability of calculators and computers, it is much quicker to use a


mathematical solution. For cohesionless soil the vectors are expressed in terms of
inclination β of surface BC and of θ, α, φ'and δ, and the expression for Pa is equation
14.50 (p. 474*) in the prescribed book. The critical wedge is found by differentiation in
respect of β, and Coulomb's active force per unit length is given by equations 14.52
and 14.53 (p. 475*) in the prescribed book. Unfortunately, the mathematical solution
is somewhat less powerful than the graphical method. It can no longer handle irregular
soil surfaces or non-uniform surcharge. The solution in the presence of partial
submergence or other forms of non-homogeneity is only approximate, more so when
the soil surface is not horizontal. For a vertical wall and horizontal soil surface, the

247 GEE2601/1
mathematical expression for Coulomb's active earth pressure can be extended to
include cohesion and adhesion.

cw
σ 'a = σ '0 K a − 2c ' K a (1 + )
• c' (9.8)

Where:

• cw = Adhesion between soil and wall material, in terms of effective stress

The depth of the zone of tension may be determined from this equation in the same
way as was done to derive equation 9.4.

9.6.2 Passive case

The passive case can be analysed in the same way as the active one. In figure 9.21
the wall is moved into cohesionless soil, and a passive wedge ABC is formed by shear
failure on the planes AB and BC. The wall movement will force the wedge upward and
therefore the forces F and Pp are inclined at angles φ' and δ above the normal to the
planes. The inclination of plane BC is again varied and vector diagrams drawn until Pp
reaches its critical value, which is the minimum value in the passive case. The
mathematical solution is expressed in terms of Coulomb's coefficient of passive earth
pressure, Kp as in equations 14.54 and 14.55 (p. 479*) in the prescribed book. If the
wall is vertical and the soil surface horizontal, the expression for Coulomb's passive
pressure can be extended to include cohesion and adhesion.

cw
σ p' = σ '0K p + 2c ' K p (1 + )
• c' (9.9)

9.6.3 Accuracy of Coulomb's theory

Coulomb's theory does not make any assumptions regarding interface shear strength
parameters, δ and cw. They can be measured in laboratory tests and used in the
analyses. This removes some of the uncertainty inherent in Rankine's theory, which
should make Coulomb's the better option. However, Coulomb did not allow for the
influence of interface friction on the shapes of the failure planes and made the

248 GEE2601/1
assumption that the sliding surfaces BC in figures 14.18 and 14.19 (pp. 480-481*) in
the prescribed book are straight lines. The true sliding surfaces are in fact curved, and
the incorrect assumption leads to underestimation of the active pressure and
overestimation of the passive pressure. Both errors are on the unsafe side, and
Coulomb's theory should be used with caution. Active pressure is usually not affected
much by the curvature of the failure surface, but Coulomb's passive pressure may be
wrong by a wide margin if δ > φ'/2. The above applies to the graphical solution. We
have already discussed inaccuracies and drawbacks of the mathematical solution
relative to the graphical.

Example 9.6

Figure 9.6 below shows a concrete retaining wall that supports a 5 m high granular fill.
The unit weight and shear strength parameters of the fill material are given on the
figure. The fill is homogeneous and dry. Calculate and draw Coulomb's active pressure
distribution against the wall, and comment on the point of application and orientation
of the active force.

FIGURE 9.6
Geometry of retaining wall for example 9.6

249 GEE2601/1
Solution

The first step is to determine all the angles to be used in the calculation of Ka:

• α = 15°
• φ' = 40°
• δ = 25°
• θ = atan (1/5) = 11.3°

Next, calculate Ka from equation 9.53 in the prescribed book:

• Ka = 0.3476
Now calculate the active pressure at the top and bottom of the wall:

• At the top σ0' = 0 so that σa' = 0


• At the bottom σ0' = 18 × 5 = 90 kN/m2 so that σa' = 90 × 0.3476 = 31.3 kN/m2

The active pressure diagram is shown below in figure 9.7.

FIGURE 9.7
Coulomb's active pressure on retaining wall for example 9

250 GEE2601/1
• Pa = 0,5KaγH2 = 78.25 kN/m2

Note:

• The active pressure is inclined at δ = 25° above the normal to the wall surface, or
δ + θ = 36.3° above the horizontal.
• The active force per unit width of the wall is calculated from the area of the pressure
diagram, or from equation 14.52 (p. 475*) in the prescribed book.
• Pa is inclined at 36.3° above the horizontal like σa', and from the triangular shape
of the pressure diagram its point of application is 5/3 m above the bottom of the
wall.

9.7 SELF-ASSESSMENT ACTIVITIES

You now have the opportunity to apply what you have learnt in this unit. Answer the
following questions:

Activity 9.1: Prescribed book exercises

1. Read the summary of the chapter in section 14.8 (pp. 483-484*).


2. Carefully study examples 14.1 to 14.4 (pp. 466-471*).
3. Do exercises 14.1 to 14.10 (pp. 494-487*).
4. Carefully study example 14.5 (pp. 482-483*).
5. Do exercise 14.19 (p. 489*).

Activity 9.2: Multiple-choice questions

1. Which of the following is not a retaining structure?


(a) retaining wall
(b) basement wall
(c) raft
(d) bulkhead

251 GEE2601/1
2. When a retaining structure does not move either to the right or to the left of its initial
position, the ratio of the effective horizontal stress to the effective vertical stress is
generally represented by …
(a) K.
(b) K0.
(c) Ka.
(d) Kp.

3. For coarse-grained soils, the coefficient of earth pressure at rest can be estimated
by using Jaky's equation, which is given as … where all the symbols have their
usual meanings.
(a) K0 = 1 + sin φ'
(b) K0 = 1 - sin φ'
(c) K0 = (1 - sin φ') (OCR) sin φ'
(d) K0 = 0.44 + 0.42[PI (%)/100]

4. The total force per unit length of the retaining wall of height H when it does not
move either to the right or to the left of its initial position is given as … where all
the symbols have their usual meanings.
(a) P0 = K0γH
(b) P0 = 0.5K0γH
(c) P0 = 0.33K0γH
(d) P0 = 0.5K0γH2

5. The magnitude of coefficient of earth pressure at rest in most soils ranges


between …
(a) 0.0 and 0.5.
(b) 0.0 and 1.0.
(c) 0.5 and 1.0.
(d) 0.5 and 2.0.

252 GEE2601/1
6. The condition in which every point in a soil mass is on the verge of failure refers
to …
(a) elastic equilibrium.
(b) plastic equilibrium.
(c) Both (a) and (b).
(d) None of the above.

7. When a retaining structure moves towards the soil backfill, the stress condition
within the soil backfill is called the …
(a) at-rest state.
(b) active state.
(c) passive state.
(d) Both (b) and (c).

8. Rankine's theory of earth pressure assumes that …


(a) the back face of the wall in contact with the soil backfill is smooth.
(b) the wall extends to an infinite depth.
(c) Both (a) and (b).
(d) the soil is massless.

9. The coefficient of Rankine's active earth pressure is …


(a) Ka = tan (45° - φ/2).
(b) Ka = tan (45° + φ/2).
(c) Ka = tan2 (45° - φ/2).
(d) Ka = tan2 (45° + φ/2).

10. The coefficient of Rankine's passive earth pressure is …


(a) Kp = tan (45° - φ/2).
(b) Kp = tan (45° + φ/2).
(c) Kp = tan2 (45° - φ/2).
(d) Kp = tan2 (45° + φ/2).

253 GEE2601/1
11. In Rankine's active state, the failure plane within the soil backfill makes an angle
with the horizontal given as …
(a) 45°.
(b) φ'.
(c) 45° - φ/2.
(d) 45° + φ/2.

12. In Rankine's passive state, the failure plane within the soil backfill makes an
angle with the horizontal given as …
(a) 45°.
(b) φ'.
(c) 45° - φ/2.
(d) 45° + φ/2.

13. For Rankine's active state, the active earth pressure from the cohesionless soil
backfill at the bottom of a retaining wall of height H is … where all the symbols
have their usual meanings.
(a) Pa = KaγH
(b) Pa = 0.5KaγH
(c) Pa = 0.33KaγH
(d) Pa = 0.5KaγH2

14. For Rankine's passive state, the passive earth pressure from the cohesionless
soil backfill at the bottom of a retaining wall of height H is … where all the
symbols have their usual meaning.
(a) Pp = KpγH
(b) Pp = 0.5KpγH
(c) Pp = 0.33KpγH
(d) Pp = 0.5KpγH2

254 GEE2601/1
15. The total active force per unit length of the retaining wall of height H from the
cohesionless soil backfill is given as … where all the symbols have their
meanings.
(a) Pa = KaγH.
(b) Pa = 0.5KaγH.
(c) Pa = 0.33KaγH.
(d) Pa = 0.5KaγH2.

16. The total passive force per unit length of the retaining wall of height H from the
cohesionless soil backfill is given as …
(a) Pp = KpγH.
(b) Pp = 0.5KpγH.
(c) Pp = 0.33KpγH.
(d) Pp = 0.5KpγH2.

17. The total active force on the retaining wall of height H from the cohesionless soil
backfill acts above the base of the wall at a height of …
(a) H/4.
(b) H/3.
(c) H/2.
(d) 3H/4.

18. The total passive force on the retaining wall of height H from the cohesionless
soil backfill acts above the base of the wall at a height of …
(a) H/4.
(b) H/3.
(c) H/2.
(d) 3H/4.

255 GEE2601/1
19. For Rankine's active state, the active earth pressure from the cohesive soil
backfill at the bottom of a retaining wall of height H is … where all the symbols
have their usual meanings.
(a) Pa = KaγH.
(b) Pa = KaγH – 2c'(Ka)0.5
(c) Pa = KaγH + 2c'(Ka)0.5
(d) Pa =0.5KaγH2 - 2c'(Ka)0.5

20. The typical value of wall tilt (the ratio of horizontal displacement of the wall top
to its height when the wall rotates about its bottom) required for achieving
Rankine's passive state in dense sand is …
(a) 0.005.
(b) 0.01.
(c) 0.02.
(d) 0.04.

21. An application of surcharge at the top of the soil backfill …


(a) causes no change in the earth pressure along the depth of the wall.
(b) decreases the earth pressure along the depth of the wall.
(c) increases the earth pressure along the depth of the wall.
(d) increases the earth pressure near the top of the wall only.

22. The presence of cohesion in the soil backfill …


(a) causes no effect on the earth pressure along the depth of the wall.
(b) decreases the active earth pressure along the depth of the wall.
(c) increases the passive earth pressure along the depth of the wall.
(d) Both (b) and (c).

256 GEE2601/1
23. The depth of tension cracks in the cohesive soil backfill under undrained
condition is …
𝐶𝐶𝑢𝑢
(a) 𝑍𝑍0 = 𝛾𝛾
2𝐶𝐶𝑢𝑢
(b) 𝑍𝑍0 = 𝛾𝛾
𝛾𝛾
(c) 𝑍𝑍0 = 𝐶𝐶
𝑢𝑢

2𝛾𝛾
(d) 𝑍𝑍0 = 𝐶𝐶
𝑢𝑢

24. The development of tensile cracks in the upper part of the cohesive soil backfills …
(a) causes no effect on the earth pressure along the depth of the wall.
(b) decreases the active earth pressure along the depth of the wall.
(c) increases the active earth pressure along the depth of the wall.
(d) Both (b) and (c).

25. For a retaining wall with a rough vertical back, the total active earth pressure
acts …
(a) horizontally.
(b) in a direction making an angle greater than 90° with the vertically upward
direction.
(c) in a direction making an angle less than 90° with the vertically upward
direction.
(d) in any direction.

26. For a retaining wall with a rough vertical back, the total passive earth pressure
acts …
(a) horizontally.
(b) in a direction making an angle greater than 90° with the vertically upward
direction.
(c) in a direction making an angle less than 90° with the vertically upward
direction.
(d) in any direction.

257 GEE2601/1
27. Which of the following earth pressure theories considers the roughness of the
back of the wall?
(a) Rankine's active earth pressure theory
(b) Rankine's passive earth pressure theory
(c) Coulomb's earth pressure theory
(d) All of the above.

28. Coulomb's active earth pressure coefficient becomes equal to Rankine's active
earth pressure where α is the angle made by the top surface of the soil backfill
with the horizontal, θ is the inclination of the back face of the wall to the vertical
and δ′ is the angle of friction between the soil backfill and the wall.
(a) α = 0 and θ = 0
(b) α = 0 and δ′ = 0
(c) θ = 0 and δ′ = 0
(d) α = 0, θ = 0 and δ′ = 0

29. The wall friction results in …


(a) reduction in the total active earth pressure.
(b) an increase in the total passive earth pressure.
(c) Both (a) and (b).
(d) an increase in the total earth pressure.

30. When the soil-wall interface friction angle becomes greater than about half of
the soil backfill frictional angle, Coulomb’s earth pressure theory overestimates
the passive force, which is on …
(a) the unsafe side of the design.
(b) the safe side of the design.
(c) Both (a) and (b) governed by the site conditions.
(d) None of the above.

258 GEE2601/1
Activity 9.3: Typical exam questions

The structure shown below in the sketch is a retaining wall that has to be
constructed in Tzaneen, Limpopo. The wall weighs 24 kN/m3 and retains on its
vertical face soil with an equivalent density of 18 kN/m3 and an angle of internal
friction (Ǿ) of 30. The retained soil also carries a surcharge of 3.5 kN/m2. The
safety factor for overturning is 2.5. Draw the ground pressure diagram for this
retaining wall.

9.8 SOLUTIONS TO SELF-ASSESSMENT ACTIVITIES

Activity 9.1: Prescribed book exercises

1. See the prescribed book for solutions.


2. Consult the prescribed book for answers.
3. See the prescribed book for solutions.
4. Consult the prescribed book for answers.

259 GEE2601/1
Activity 9.2: Multiple-choice questions

1. (c)
2. (b)
Discussion: K0, Ka and Kp are called coefficient of earth pressure at rest,
coefficient of active earth pressure and coefficient of passive earth pressure,
respectively.
3. (b)
4. (d)
5. (c)
6. (b)
7. (c)
8. (c)
9. (c)
10. (d)
11. (d)
12. (c)
13. (b)
14. (b)
15. (d)
16. (d)
17. (b)
18. (b)
19. (c)
20. (a)
21. (c)
22. (d)
23. (b)
24. (c)
26. (c)
27. (c)

260 GEE2601/1
28. (d)
29. (c)
30. (a)

Activity 9.3: Typical exam questions 58.15 kPa & 103.25 kPa

9.9 SUMMARY

During the course of this study unit on lateral earth pressure, you should have learnt
how to calculate earth pressure at rest for dry and submerged soils, or active and
passive pressures using the theories of Rankine and Coulomb. What may be even
more important is that either Rankine's or Coulomb's method may be more suitable
for a given retaining problem, or they may give erroneous or even dangerous results
under certain conditions. In some cases solutions can only be approximate because
of parameters not allowed for in the theories.

With the knowledge acquired, you should be able to calculate lateral stresses and
forces against retaining structures for a variety of geometries and soil properties. This
study unit gives an excellent background to study unit 12 in which the lateral earth
pressures will be used in stability calculations for retaining structures. You need to
ensure that you are familiar with the contents of study unit 9 before attempting to read
study unit 12.

9.10 VIDEO CLIPS, SLIDES AND OTHER WEB-BASED


RESOURCES

For additional explanations on lateral earth pressure, you can watch some video clips.
Web addresses/links to the video clips are available on the module site under
Additional Resources in a folder named Video clips, slides and other web-based
resources:

• At-rest, active and passive earth pressure:


https://www.youtube.com/watch?v=RC6-LJphzW4

261 GEE2601/1
• Mod-2 Lec-1 Lateral earth pressure theories & retaining walls-1:
https://www.youtube.com/watch?v=ucbinKVZvF8
• Lateral earth pressure basics: https://www.youtube.com/watch?v=zIyOFeJyY-4

REFERENCE WORKS AND FURTHER READING

Al-Agha, AS. 2015. Basics of foundation engineering with solved problems. Based on
“Principles of foundation engineering, 7th edition”.

Coulomb, CA. 1776. Essai sur une application des règles des maximis et minimis à
quelques problèmes de statique, relatifs à I'architecture. Paris: l'Imprimerie
Royale.

Das, BM. 2010. Principles of geotechnical engineering. 7th ed. Connecticut: Cengage
Learning.

Das, BM. 2013. Fundamentals of geotechnical engineering. 4th ed. Connecticut:


Cengage Learning.

Das, BM & Sivakugan, N. 2017. Fundamentals of geotechnical engineering. 5th ed.,


international edition. Connecticut: Cengage Learning.

Rankine, WMJ. 18-57. On stability on loose earth. Part I, 9-27. London: Philosophic
Transactions of Royal Society

* Page, figure, table and chapter numbers may differ on updated and revised versions of the
prescribed book. See online version of study guide for updates.

262 GEE2601/1
Study unit 1 10

SLOPE STABILITY

10.1 INTRODUCTION

In study unit 9 we discussed the lateral pressure of soil against retaining structures in
detail. From that study unit it became clear that soil cannot support itself to an
appreciable height in a near-vertical position. Its shear strength is too low and it needs
the support from a retaining structure. If the face is at a flatter angle, the shear strength
may be sufficient for the soil to stand unsupported to a certain height. Natural slopes,
cuts, fills and earth dams are examples of unsupported soil faces.

In this study unit you will study various factors that play a role in the stability of slopes.
We will discuss a variety of possible slope failure mechanisms together with models
or methods of analysing each. You will realise that failure mechanisms are very
complicated and cannot be modelled perfectly, so simplifying assumptions have to be
incorporated into the models. One of the factors that cannot be taken into account
exactly is the variability of the soil, and some factor of safety will be incorporated into
the analyses to allow for this and other uncertainties. Figures 10.1 and 10.2 below
show infinite and finite slopes, respectively.

263 GEE2601/1
FIGURE 10.1
Infinite slope (the long slope on the face of a mountain) (Shutterstock 2018)

FIGURE 10.2
Finite slope (earth dam)

10.2 DEFINITIONS

You will come across the following terms in this study unit:
• Finite slope: A slope that is limited in extent (i.e. slopes of embankments, earth
dams).
• Infinite slopes: A constant slope of infinite extent (i.e. the long slope on the face
of a mountain).

264 GEE2601/1
• Safety factor: The ratio between the force/moment/stress that stabilises a
structure, or element of a structure, and the force/moment/stress that acts to
disturb the same structure or element.
• Slope stability: The potential of soil covered slopes to withstand movement.

10.3 LEARNING OUTCOMES


On completion of this study unit, you should be able to analyse a variety of slopes to
determine the factor of safety against sliding along a trial failure surface.

More specifically, you should be able to

• determine the safety factor of earth slopes


• evaluate the factors that have to be taken into account in the analysis of a particular
slope
• select the appropriate method of analysis
• conduct the analysis to assess the stability of a particular slope

To achieve the learning outcomes in this unit, work through the following topics:
• the definition of a factor of safety
• stability of infinite slopes
• stability of homogeneous finite slopes with cylindrical failure surfaces
• stability of general slopes with cylindrical failure surfaces

10.4 FACTOR OF SAFETY


Chapter 13 (pp. 390-445*) in the prescribed book deals with the contents of this study
unit. The engineering definition of safety factor, as shown in section 10.2 above and
section 13.2 (pp. 391-392*) in the prescribed book, is the ratio between the
force/moment/stress that stabilises a structure, or element of a structure, and the
force/moment/stress that acts to disturb the same structure or element. In soil,
instability usually occurs in the form of shear failure, and the factor of safety is therefore
defined as

• FSs = τf / τd (10.1)

265 GEE2601/1
Where:
• FSs = Factor of safety in respect of shear strength
• τf = Average shear strength on the failure surface
• τd = Average shear stress developed on the failure surface

Since shear strength of soil consists of two components, cohesion and friction, the
definition of FSs can be extended to include these, as shown in equations 13.2 to
13.4 (p. 391*) in the prescribed book. Note that the disturbing shear stress is
expressed in terms of developed cohesion and developed internal friction angle. This
may seem strange at this stage, but will prove to be very useful. Separate factors of
safety may even be introduced for cohesion and friction, as shown in equations 13.5
and 13.6 (pp. 391-392*) in the prescribed book. Usually the two factors are taken to
be equal, but dissimilar values may be used to allow for different uncertainties
regarding the two strength components. A factor of safety equal to 1 means that shear
strength is fully mobilised and failure is imminent.

10.5 INFINITE SLOPES WITHOUT SEEPAGE

Section 13.3 (pp. 392-396*) in the prescribed book covers this section. An infinite
slope may also be defined as a slope in which the depth H at which shear failure is
analysed is much smaller than the slope height. The extent of the sliding volume,
measured along the slope face, is usually very large and end effects may be ignored.

Figure 13.2 (p. 392*) in the prescribed book shows part of a section through a dry
infinite slope without seepage. Because of the infinite extent, the side forces F on the
element are equal and opposite and they completely cancel each other. The weight W
of the element is in equilibrium with the normal force Nr and shear force Tr on its base.
These reaction forces may be calculated from the components of R = W in the two
directions. Because the definition of the safety factor is in terms of stresses, the forces
Nr and Tr will be converted to stresses by dividing them by the base area of the
element, as done in the prescribed book.

266 GEE2601/1
The result is

• σ' = γH cos2 β (10.2)


• τ = γH cos β sin β (10.3)
τ is also the developed shear stress, so that

• τ = τd = cd' + σ' tan φd' (10.4)

Where:

• cd' = Effective cohesion


• φd' = Angle of friction that developed along potential failure surface

Combining equations 10.2, 10.3 and 10.4 above and doing a little mathematical
manipulation lead to equation 13.14 (p. 391*) in the prescribed book. Substituting the
equations’ defining safety factors in respect of cohesion and friction angle, and
assuming that these factors are both equal to FSs, result in

c' tan φ'


• FS S = 2
+ (10.5)
γH cos β tan β tan β

From the equation you can see that FSs will decrease with increasing depth H. If the
soil profile is fairly deep, shear failure will occur at a critical depth, Hcr where FSs = 1.
The value of Hcr may be determined from the equation by setting the factor of safety
equal to unity. The result is equation 13.17 (p. 392*) in the prescribed book.

10.6 INFINITE SLOPES WITH SEEPAGE

Section 13.3 (pp. 392-396*) in the prescribed book deals with this. The presence of
water will influence the effective stresses in the soil and change the value of the factor
of safety. Water cannot be stationary because of the slope, and in the majority of cases
seepage will occur parallel to the slope face. Figure 13.3 (p. 394*) in the prescribed
book shows parallel seepage in a completely submerged slope. The analysis may be
performed by considering saturated unit weight and boundary water pressures, or by
considering submerged unit weight and seepage pressure, as we discussed in study
unit 5. The first option is usually adopted for slope stability analysis simply because

267 GEE2601/1
calculations are less involved, especially if the direction of flow is not the same
everywhere.

As in the case of dry slopes, the stability of an element is analysed. Side forces F now
include boundary water pressure, but the two forces will still be equal and opposite
and may be omitted from force equilibrium analyses. The weight of the element is
calculated using the saturated unit weight, and it is in equilibrium with the normal force
Nr and shear force Tr components of the reaction force R on its base. The forces are
converted to normal and shear stresses as before. Pore water pressure must be
subtracted from this normal stress before the resistive shear stress that develops at
the base can be calculated, as in equation 13.25 (p. 398*) of the prescribed book.

The pore water pressure at the base is determined from the equipotential lines, which
are at right angles to the seepage direction, and applying Bernoulli's equation. The
result is shown in figure 10.3 below:

• u = H cos2 β (10.6)

FIGURE 10.3
Analysis of infinite slope (with seepage) (Das & Sobhan 2014)

268 GEE2601/1
As in the dry case, the developed shear stress τd in equation 13.26 (p. 398*) in the
prescribed book must be equal to the applied shear stress τ in equation 13.24 (p.
398*) in the prescribed book and assuming identical safety factors for c' and tan φ',
leads to

c' γ' tan φ'


• FS S = 2
+ (10.7)
γ sat H cos β tan β γ sat tan β

A critical depth may be determined for this case too, by setting the factor of safety
equal to 1.

Example 10.1

A natural 1:4 slope (1 vertical:4 horizontal) must be analysed for stability. The soil
profile is weathered to a depth of 3 m, below which solid rock is encountered. Direct
shear tests were done to determine the shear strength parameters of the soil with the
following results:

• c' = 10 kN/m2 and φ' = 30°.


• The unit weight of dry soil is 16 kN/m3 and it increases to 21 kN/m3 when it
becomes saturated.
• Assume that the unit weight of water is 10 kN/m3.

a) Determine the factor of safety against shear failure on the soil-rock interface
when the soil is dry.
b) Determine the factor of safety against shear failure on the soil-rock interface
when it is completely submerged with seepage parallel to the slope.

Solution

a) For dry soil:


Calculate the slope angle:
β = tan-1 (¼) = 14.0°

269 GEE2601/1
Calculate the factor of safety using equation 10.5:

c' tan φ' 10 tan 30


FS S = + = + = 3,20
2
γH cos β tan β tan β 16 × 3 cos 14 tan 14 tan 14
2

b) For submerged soil:


• Calculate the submerged unit weight: γ' = γsat – γw = 21 – 10 = 11 kN/m3
• Calculate the factor of safety using equation 10.7:

c' γ' tan φ' 10 11tan 30
FS S = + = + = 1,89
2
γ sat H cos β tan β γ sat tan β 21× 3 cos 14 tan 14 21tan 14
2

Submergence with steady seepage substantially reduces the factor of safety


compared with the dry case.

10.7 FINITE SLOPES WITH CIRCULARLY CYLINDRICAL FAILURE


SURFACES

Section 13.5 (pp. 400-401*) in the prescribed book covers this topic. If the sliding
depth is approximately equal to the slope height, the slope is regarded as finite. The
sliding surface is usually assumed to be in the form of a circular cylinder, although
slope geometry and a layered soil profile may force the sliding surface to assume other
forms. Apart from the shape, the position of the sliding surface is not fixed and the
person doing the stability analysis should investigate a variety of positions in order to
come up with the most critical surface. A few possibilities are shown in figure 13.6 (p.
401*) in the prescribed book, and you should know the differences between toe circles,
slope circles and midpoint circles. The mass procedure of stability analysis is applied
when the soil is more or less homogeneous. The analysis is merely a matter of moment
equilibrium about the axis of the cylinder.

10.7.1 Slopes in soil with φ = 0 (clay soils)

The only source of shear strength in the undrained condition of a saturated soil is
cohesion. The moment equilibrium of the soil above the trial sliding surface in figure

270 GEE2601/1
13.7 (p. 402*) in the prescribed book leads to an expression for the developed
cohesion shown in equation 13.44 (p. 410*) in the prescribed book. This expression
can be combined with the definition of the factor of safety, equation 13.45 (p. 411*) in
the prescribed book, to obtain

• FSs = cu r2 θ / (W1l1 – W2l2) (10.8)

Note that this factor of safety is for the specific trial surface. The position of the centre
and the value of the radius should be varied systematically to find the surface with the
minimum factor of safety. This minimum value is the factor of safety for the slope, and
the circle for which it was calculated is termed the critical circle. Tension cracks may
exist to a depth z0 in cohesive soil as a result of shrinkage and stress changes. We
discussed this in connection with lateral pressure. If cracks near the top of the slope
fill with water during rainfall, an additional moment will develop that must be included
in the moment equilibrium analysis. The presence of the crack may also have an effect
on the value of θ. Equation 10.8 can be modified to

c ur 2 θ
• FS s = (10.9)
(W1l1 − W2 l 2 ) + Pw y c
Where:

• Pw = Horizontal water force = 0,5 γwz02


• yw = Position of water force below centre of circle

Fellenius (1927) and Taylor (1937) independently analysed many slopes to find
similarities in the positions of the critical circles and the values of the factors of safety.
A dimensionless coefficient, m = cd/γH, called the stability number, was used to
compare slopes. According to their findings, which are shown graphically in figures
13.8 to 13.11 (pp. 404-406*) in the prescribed book, the slope angle β and the depth
to a firm layer are very important factors determining the type and position of the critical
failure surface. The latter is expressed in terms of the depth (D). With the aid of these
figures, called Taylor's charts, the search for the critical slope is greatly reduced or
eliminated. The charts do not provide for tension cracks.

271 GEE2601/1
Figure 13.8(b) (p. 404*) is always used first. It gives the stability number, m, from
which cd and the factor of safety can be calculated. The figure also gives the type of
slope failure surface. If the slope angle exceeds 53°, the failure surface is a toe circle
and more detail about the position of the circle can be found from figure 13.9 (p. 405*).
For slope angles less than 53°, the depth (D) determines whether the critical surface
would be a toe, midpoint, or slope circle. Slope circles are again located from the detail
in figure 13.9 (p. 405*), whereas figure 13.10 (p. 406*) gives information about the
location of midpoint circles. A golden rule for determining the type of circle is that, for
φ = 0, the critical circle will always be as deep as possible, and figure 13.8 (p. 404*)
in the prescribed book reflects this. If D = 1, the geometry dictates only slope circles.
For D = 1.2 critical toe circles occur down to β of about 200, after which the surface will
be a slope circle. For D = 1.5 and 2 midpoint circles are the most critical down to a
slope angle of about 10°, with slope circles at flatter angles, while D > 2 means a
midpoint critical circle for all slope angles. Carefully interpolate on figure 13.8 (p. 404*)
for other values of D, taking the boundaries between the circle types into account.

Example 10.2

A slope is to be excavated at an angle of 30° to a height of 10 m. The soil is saturated


clay for which cu = 35 kN/m2 and γ = 18 kN/m3. A hard stratum is situated at a depth
of 15 m.

a) Determine the factor of safety against slope failure and the position of the most
critical sliding surface.
b) What is the maximum slope angle that the clay will be able to sustain?

Solution

a) D = 15/10 = 1.5 and β = 30°: m ≈ 0.165 midpoint circle.

From equations 13.46 and 13.45 in the prescribed book:

• cd = 0.165 × 18 × 10 = 29.7 kN/m2


• FSs = 35/29.7 = 1.18

272 GEE2601/1
From figure 13.10 (p. 406*) of the prescribed book, with D = 1.5 and b = 30°:

• n ≈ 0.5 distance = 0.5 × 10 = 5 m from toe.

b) At maximum slope angle, FSs = 1 and cd = cu = 35 kN/m2

From equation 10.46 (p. 412*) in the prescribed book:

• m ≈ 35/(18 × 10) = 0.194

From figure 13.8 (p. 408*) in the prescribed book, with m = 0.194:
• β = 60° toe circle

From figure 13.9 (p. 405*) in the prescribed book:


α = 35° and θ = 72°, from which the circle geometry can be determined

Example 10.3

A slope is to be cut at 45° in saturated clay for which cu = 30 kN/m2 and γ = 16 kN/m3.
Determine the maximum height of the slope if a factor of safety of 1.5 is to be
maintained. A firm layer is at 12 m below the surface.

Solution

Calculate the developed cohesion:


cd = 30/1.5 = 20 kN/m2

Because of the presence of the firm layer, the solution cannot be obtained directly,
and a process of iteration is used. Figure 13.8 and equation 13.46 in the prescribed
book are used in the iterations. Initially a slope height of 6 m is guessed.

If

• H = 6 m, then D = 12/6 = 2 m; m ≈ 0.179


• H ≈ 30/(16 × 0.179) = 10.5 m

273 GEE2601/1
If:
• H = 10.5 m, then D = 12/10.5 = 1.14; m ≈ 0.175
• H ≈ 30/(16 × 0.175) = 10.7 m

If:
• H = 10.7 m, then D = 12/10.7 = 1.12; m ≈ 0.174
• H ≈ 30/(16 x 0.174) = 10.8 m

It seems that the iteration process will converge at a height of about 10.8 m. Accuracy
is not all that good, because of interpolation in the rather small and busy figure.

10.7.2 Slopes in soil with φ > 0

In effective stress analysis the shear strength of soil includes both cohesion and
friction. The analysis again involves moment equilibrium about the centre of the trial
circular arc. The driving moment is caused by the weight of the soil above the arc, and
the two components of developed shear strength, cd and tan φd', provide the resisting
moment. As a matter of convenience, these components were considered separately
in the stability analysis.

The stability analysis of the soil mass above the trial failure surface involves a
graphical procedure using a vector diagram like the one in figure 13.16 (p. 413*) in
the prescribed book. Several trial surfaces are analysed to find the surface with the
lowest factor of safety, and the process may be very time-consuming.

From his analysis of many different slopes, Taylor was able to simplify the process by
presenting a chart, figure 13.17 (p. 415*) in the prescribed book, which gives a stability
number for the critical circle. These types of calculations are numerically demanding
and Taylor's chart is useful in this regard. However, there are some simple, yet useful,
software tools available that can be used for these calculations (Slope Stability
Analysis Software (STABL), GEO5 Slope Stability, Visual Slope and SVSlope). Figure
13.18 (p. 416*) in the prescribed book further simplifies the calculations by eliminating
the iterative procedure that is involved in determining the factor of safety. Note that the
figures apply only to homogeneous slopes and do not allow for pore water pressures.

274 GEE2601/1
Example 10.4

A homogeneous dry embankment, 8 m high, makes an angle of 60° with the horizontal.
Soil properties are γ = 20 kN/m3, c' = 25 kN/m2, φ' = 30°. Use Taylor's chart to determine
the safety factor for the embankment.

Solution

The first option is to use an iterative procedure.

• First guess FSφ = 2.0:


• tan φd' = (tan 30°)/2.0 = 0.28867
• φd' = 16.1°

From figure 13.17 (p. 414*) in the prescribed book, for β = 60° and φd' = 16.1°:
• m = 0.112

From equation 10.5 in the prescribed book:

• cd = 0,112 × 20 × 8 = 17.92 kN/m2


• FSc = 25/17.92 = 1.40

Clearly FSc ≠ FSφ, so use FSφ = 1.4:


• φd' = 22.40
• m ≈ 0.089
• cd = 0.089 × 20 × 8 = 14.24 kN/m2
• FSc = 1.76

FSc ≠ FSφ, so use FSφ = 1.76:


• φd' = 18.2°
• m ≈ 0.10
• cd = 0.10 × 20 × 8 = 16.0 kN/m2
• FSc = 1.56

The iterative process continues until convergence, which is at FSs ≈ 1.6 for this slope.

275 GEE2601/1
A second option would be to guess different values for FSφ, determine the
corresponding value of FSc by means of Taylor's chart and use a plot to determine the
correct FSs. This was done in example 13.6 (p. 411*) in the prescribed book.

The third option is to use the charts by Singh to eliminate the iterations. The slope
angle of 60° gives an inclination of 1 vertical to 0.577 horizontal, so we will have to
interpolate between figures 13.18(a) and (b) of the prescribed book, with φ' = 30° and
c'/γH = 0.156. The two figures give FSs ≈ 1.45 and FSs ≈ 1.7, respectively, which leads
to the conclusion that the factor of safety should be about 1.6.

Example 10.5

An embankment is to be constructed with a slope angle of 50°. The soil has γ = 20


kN/m3, c' = 20 kN/m2, φ' = 40°. The factor of safety must be 2.0. What is the maximum
height of the embankment?

Solution

For FSφ = FSc = FSs = 2.5, calculate the developed shear parameters:
• φd' = tan-1 [(tan 40) / 2,0] = 22,76°
• cd = 20/2.0 = 10 kN/m2

From figure 10.15 in the prescribed book, with φd' = 22.76° and β = 50°:
• m ≈ 0.065

This gives Hcr ≈ 10/(20 × 0,065) = 7.69 m

10.8 METHOD OF SLICES

Sections 13.7 and 13.8 (pp. 423-429*) in the prescribed book cover this topic. Taylor's
charts are useful tools to use in stability analyses, but they can only be used for
homogeneous slopes without pore water pressure. A more general and powerful
method of analysis is required to analyse slopes that include one or more of the
following features: more than one type of soil, shear strength that varies with depth,

276 GEE2601/1
partial submergence with static water tables or seepage conditions. The method of
slices is a very powerful instrument. It involves subdivision of the volume of soil above
a trial slip surface into vertical slices. The stability of each slice is analysed, as well as
the overall stability of all the slices together. The method is illustrated in figure 13.24(a)
in the prescribed book, and the forces that act on a typical slice, slice n, are shown in
figure 13.24(b).

Mathematically the method of slices poses a problem. For any number of slices greater
than 1, the number of unknowns (interslice forces P and T, base forces N and T)
exceeds the number of equations (vertical, horizontal and moment equilibrium) and
the analysis cannot be done. In other words, the number of unknowns is greater than
the number of equilibrium equations. To overcome the problem, a number of “methods
of slices” evolved, and each makes an assumption about some of the unknowns in
order to reduce their number. Two of these are generally included in textbooks
because they can be solved by means of manual calculations. Both methods assume
circularly cylindrical failure surfaces.

Note that several trial surfaces have to be analysed in order to find the minimum value
of the factor of safety for the slope. A systematic procedure must be adopted in which
the centre of the trial surface is shifted in a grid pattern, and for every centre a number
of circles of different radii are analysed.

10.8.1 Ordinary method of slices

The method was devised by Fellenius (1927). His method overcomes the problem of
too many unknowns by assuming that, for every slice, the interslice forces on the left
exactly balance those on the right.

Resolving forces perpendicular to the base of slice n leads to


• Nr = Wn cos αn (10.10)

Division by ∆Ln gives the normal stress on the base


• σ n' = Nr / ∆Ln = Wn cos α n / ∆Ln (10.11)

277 GEE2601/1
Available shear strength on the base is
• τ = c n' + σ n' tan φ n' = c n' + (Wn cos α n / ∆Ln) tan φ n'

Note:

• c n' and φ n' are the shear strength parameters of the soil in which the base of slice
n is located.
• Soil layers above the base only contribute to the weight.

Developed shear strength is

1  W cos α n 
τd =  c n '+ n tan φ n ' 
FS s  ∆L n 

Multiplication by ∆Ln gives the developed shear force:


1
• Td = (c n ' ∆L n + Wn cos α n tan φ n ') (10.12)
FS s

Moment equilibrium of the sliding mass about the centre of the circle involves
summation of the individual slice driving moments, r Wn sin αn, to give the overall
driving moment, as well as summation of the individual slice shear resistance
moments, r Td, to give the overall resisting moment. The result is

n =p
∑ (c n ' ∆L n + Wn cos α n tan φ n ')
n =1
• FS s = n =p
(10.13)
∑ Wn sin α n
n =1

Equation 10.13 above and equation 13.51 (p. 424*) in the prescribed book are valid
for dry slopes and they are in terms of effective stresses. The equations are also valid
for undrained conditions, but then total stresses cu and φu are used.

If pore water pressures exist, there will be a boundary water force Un perpendicular to
the base of slice n, with Un = un ∆Ln. This force is part of Nr in equation 10.10. The
effective stress now becomes

• σ n' = (Nr – Un) / ∆Ln = (Wn cos αn – un ∆Ln) / ∆Ln = Wn cos αn / ∆Ln – u

278 GEE2601/1
The other equations are adjusted accordingly, with the result that
n =p
∑ [c n ' ∆L n + ( Wn cos α n − u∆L n ) tan φ n ']
n =1
• FS s = n =p
(10.14)
∑ Wn sin α n
n =1

Since boundary water pressure was used in the derivation of equation 10.14, the
weights of the slices must be calculated using saturated unit weight below the phreatic
surface.

Note:

The second square bracket is in the wrong position in equation 13.52 (p. 425*) in the
prescribed book. It should be after the tan φ', and not before it.

• Water pressure is not considered separately in total stress analyses.

10.8.2 Bishop's simplified method of slices

Bishop (1955) overcame the problem of too many unknowns by considering slice
equilibrium in the vertical direction only. This step eliminates the horizontal interslice
forces from the analysis. He also assumed that the two vertical interslice forces were
equal. The derivation follows the same steps as for the ordinary method of slices, but
resolution of forces in the vertical direction causes the factor of safety to appear in the
expressions for both Tr and Nr. The result is that FSs cannot be solved directly from
the final equation of overall moment equilibrium and an iterative procedure is required.
Fortunately, convergence is rapid and three iterations are usually sufficient.

Bishop's equation for the steady-state seepage case is

 
n= p 1 
∑  (c '
n nb + (Wn − u b
n n ) tan φ n ' ) tan φn ' sin α n 

n =1  cos α n +
 FS s 
FS s = n= p

∑Wn sin α n
n =1 (10.15)

279 GEE2601/1
The equation is slightly simplified by putting

tan φ n ' sin α n


m α(n ) = cos α n + (10.16)
FS s

so that

n =p 
1 
∑ n n (c ' b + ( W n − u b
n n ) tan φ n ' ) 

n =1  m α(n ) 
• FS s = n =p
(10.17)
∑ Wn sin α n
n =1

If there is no pore water pressure, equation 10.17 is used with u = 0 for all slices, and
when total stress analyses are performed cu and φu are used instead of c' and φ'. In
total stress analyses pore water pressure is not considered separately, so un = 0.

Example 10.6

Figure 10.4 below shows a trial failure surface in a slope. The soil profile consists of
two layers. The upper layer has the following properties: c' = 10 kN/m2, φ' = 30°, γ =
16 kN/m3 above the phreatic surface, and γsat = 21 kN/m3 below the phreatic surface.
The lower layer has the following properties: c' = 15 kN/m2, φ' = 20°, γ = 17 kN/m3
above the phreatic surface, and γsat = 21 kN/m3 below the phreatic surface. Determine
the factor of safety against shear failure along the trial sliding surface by the ordinary
method of slices as well as by Bishop's simplified method of slices.

280 GEE2601/1
FIGURE 10.4
Geometry of slope for example 10.6

Solution: ordinary method of slices

The sliding mass has been subdivided into eight slices, as shown in figure 10.4. Slices
1 to 7 are 1.5 m wide and slice 8 is 1 m wide. Take slice 5 as a typical slice to illustrate
how the slice constants are being calculated. Figure 10.5 below shows the typical
slice.

281 GEE2601/1
FIGURE 10.5
Dimensions of typical slice

First calculate the weight of the slice:


Slice 5 intersects the upper as well as the lower soil layers. Taking vertical dimensions
along the centreline, h1 is that part of the slice that is in the upper layer above the
phreatic surface, h2 is the part of the slice in the upper layer below the phreatic surface
and h4 is the part of the slice in the lower layer below the phreatic surface. The
dimensions h1 to h4 should be taken from figure 10.4 since figure 10.5 is not to scale.

The weight of the slice can therefore be calculated from


W = b × [h1 × γdrg(1) + h2 × γsat(1) + h4 × γsat(2)]
= 1.5 × [0.95 × 16 + 0.80 × 21 + 1.90 × 21]
=107.9 kN

282 GEE2601/1
The second step is to calculate the value of the pore water pressure at the bottom of
the slice (at the failure surface):

An imaginary equipotential curve is drawn from the centre of the bottom of the slice to
the phreatic surface. At the phreatic surface the pore water pressure is zero and the
total head is constant along the equipotential. This means that, at the bottom of the
slice, the pore water pressure can be calculated from

• u = hw x γw

Where:
hw = Vertical distance between the top of the equipotential and the bottom of the slice,
as indicated in figure 10.5

For slice 5, the value of hw is 2.55 m (from figure 10.4).


• u = 2.55 x 10 = 25.5 kN/m2

Note that the method of determining u that is shown in figure 13.25 (p. 425*) in the
prescribed book is simpler, but it may render values of the pore water pressure that
are too high when the phreatic surface has an appreciable inclination.

The angle α is determined from the inclination of the radius or from the inclination of
the base of the slice. The length of slice base is measured from a scaled drawing, of
from ∆L = b/cos α.

The cohesion and friction angle of the slice are taken as those of the soil in which the
greater part of the slice base appears. For example, c' = 10 kN/m2 for slices 7 and 8
and c' = 15 kN/m2 for the remaining slices. Now that all variables are known, the
stability calculations can be performed. These are best done in the form of a table as
shown in table 10.1 below:

283 GEE2601/1
TABLE 10.1: Stability calculations

c'∆L +
n c' φ' ∆L W α u W sin α
(Wcosα – u∆L) tanφ'
1 15 20 1.55 25.2 -14.5 7.0 -6.31 28.18
2 15 20 1.51 57.7 -6.6 15.5 -6.63 34.99
3 15 20 1.55 84.8 14.0 21.5 21.23 41.00
4 15 20 1.60 97.1 20.4 24.0 33.85 43.15
5 15 20 1.70 107.9 28.1 25.5 50.82 44.37
6 15 20 1.95 106.0 39.8 21.0 67.85 43.99
7 10 30 2.35 73.8 50.3 11.0 56.78 35.79
8 10 30 2.15 17.6 62.3 0.0 15.58 26.22
233.18 297.69

Equation 10.14 can now be used to calculate the factor of safety against sliding along
the trial surface:

n =8
∑ [c n ' ∆L n + ( Wn cos α n − u∆L n ) tan φn '] 297,69
n=1
FS s = = = 1,277
n =8
233,18
∑ Wn sin α n
n=1

Solution: Bishop's modified method of slices

Slice weights and other properties have already been determined for the ordinary
method of slices and will not be repeated here. Do the calculations in the form of a
table again. An initial value of FSs = 2.00 will be used in the calculation of the mα
values. After the first iteration the value of FSs is changed to 1.464 and the mα values
are adjusted accordingly. After the second iteration FSs becomes 1.408, and a third
iteration, not shown in the table, results in FSs = 1.401, so that 1.40 may be taken as
the final value of the factor of safety for the trial sliding surface.

284 GEE2601/1
TABLE 10.2: Stability calculations
FSs = 2,00 FSs = 1,464

[c'b + (W– ub) tanɸn']m α(n)

[c'b + (W– ub) tanɸn']m α(n)


c'b + (W– ub) tanɸn'
W sinαn
mα(n)
n m α(n)

1 -6.31 27.85 0.9226 30.19 0.9059 30.74


2 -6.63 35.04 0.9725 36.03 0.9648 36.32
3 21.23 41.63 1.0139 41.06 1.0304 40.40
4 33.85 44.74 1.0007 44.71 1.0239 43.69
5 50.82 47.85 0.9678 49.44 0.9992 47.89
6 67.85 49.62 0.8848 56.08 0.9274 53.50
7 56.78 48.08 0.8609 55.85 0.9422 51.03
8 15.58 20.16 0.7204 29.77 0.8140 24.77
233.18 341.34 328.33
FSs = 1.464 1.408

** The value of FSs is calculated using equation 10.17. The initial value chosen for FSs
is not important. As an illustration, several values ranging from 1.0 to 4.0 were used
for this example and after three iterations, FSs = 1.40 was produced.

When you compare the factor of safety that resulted from the ordinary method of slices
with the one from Bishop's modified method of slices, it is clear that the former method
is conservative. This is especially true in the case of flat slopes with high pore water
pressures, and the ordinary method of slices is seldom used in practice.

285 GEE2601/1
10.9 SELF-ASSESSMENT ACTIVITIES

You now have the opportunity to apply what you have learnt in this unit. Answer the
following questions:

Activity 10.1: Prescribed book exercises


1. Read through the summary (13.11) on page 439.
2. Work through examples 13.1 and 13.2 (pp. 395-396*).
3. Do problems 13.3 to 13.5. Make sure that you learn how to use Taylor's curves.
4. Carefully study example 13.6. Familiarise yourself with Taylor's chart for φ > 0.
5. Do problem 13.7.
6. Carefully study example 13.7 to become familiar with the calculations.
7. Redo examples 13.7 and 13.8.
8. Do problems 13.10-13.20.

Activity 10.2: Multiple-choice questions


1. The factor of safety of a slope is defined as the ratio of …
(a) the average shear strength of the soil to the average shear stress
developed along the potential failure surface.
(b) the average shear stress developed along the potential failure surface to
the average shear strength of the soil.
(c) the average shear strength of the soil to the maximum shear stress
developed along the potential failure surface.
(d) the maximum shear stress developed along the potential failure surface to
the average shear strength of the soil.

2. For the design of stable slopes, the factor of safety in respect of strength is
generally considered to be …
(a) less than 1.
(b) equal to 1.
(c) greater than 1.
(d) greater than 1.5.

286 GEE2601/1
3. If the failure plane is parallel to the top ground surface of the slope, the slope is
called … slope.
(a) a finite
(b) an infinite
(c) a natural
(d) a human-made

4. For an infinite sandy slope inclined at an angle of 20° to the horizontal, the
drained angle of friction is 30°. The factor of safety of this slope will be …
(a) 0.63.
(b) 1.0.
(c) 1.59.
(d) None of the above.

5. For an infinite sandy slope inclined at an angle of 20° to the horizontal, the
drained angle of friction is 30°. If there is seepage through the soil slope and the
groundwater level coincides with top surface of the slope, its factor of safety will
be approximately …
(a) 0.63.
(b) 0.8.
(c) 1.0.
(d) 1.59.

6. Slope failures usually occur on …


(a) curved failure surfaces.
(b) plane failure surfaces.
(c) cylindrical failure surfaces.
(d) All of the above.

287 GEE2601/1
7. When the slope failure occurs in such a way that the surface of sliding passes at
some distance below the toe of the slope, it is called …
(a) slope failure.
(b) shallow slope failure.
(c) base failure.
(d) None of the above.

8. In which of the following methods of slope stability analysis can the non-
homogeneity of the soil and pore water pressure be taken into consideration?
(a) mass procedure
(b) method of slices
(c) Both (a) and (b).
(d) None of the above.

9. The slope stability number m is given as … where the symbols have their usual
meanings.
𝐶𝐶
(a) 𝑚𝑚 = 𝛾𝛾 𝑑𝑑
𝐻𝐻
𝛾𝛾𝐻𝐻
(b) 𝑚𝑚 = 𝐶𝐶𝑑𝑑

(c) 𝑚𝑚 = 𝐶𝐶𝑑𝑑 × 𝛾𝛾𝐻𝐻


(d) None of the above.

10. For slopes in homogeneous clayey soils in undrained conditions, the critical
failure circle is always a toe circle when the slope angle …
(a) β < 35°.
(b) β > 35°.
(c) β < 53°.
(d) β > 53°.

288 GEE2601/1
11. The maximum possible value of the stability number for the failure of slope in
homogeneous clayey soils under undrained conditions at the midpoint circle is

(a) 0.1.
(b) 0.181.
(c) 0.281.
(d) 0.3.

12. In the slope stability analysis method of slices, the widths of slices …
(a) need not be the same.
(b) should be the same.
(c) should differ significantly.
(d) should be greater than 1 m.

13. Which of the following methods of slope stability analysis satisfies the equations
of equilibrium in respect of both forces and moment?
(a) ordinary method of slices
(b) Bishop's simplified method
(c) Spencer's method
(d) All of the above.

14. Spencer's slope stability charts use a non-dimensional pore pressure


parameter. ru. For most practical cases, the value of ru may range from …
(a) 0 to 0.1.
(b) 0 to 0.5.
(c) 0 to 1.
(d) 0.5 to 1.

15. Which of the following methods of slope stability analysis provides the factor
safety of the finite slopes with the assumption that the potential failure surface
is a plane?
(a) ordinary method of slices

289 GEE2601/1
(b) Bishop's simplified method
(c) Spencer's method
(d) Culman's method

Activity 10.3: Typical exam questions

1. Evaluate the short-term stability of a dam that has to be constructed in a project


in Seshego, Polokwane. A section of the dam is shown in figure 10.6 below. The
embankment consists of saturated soil for which the angle of shearing
resistance φu = 0 and the undrained cohesion cu = 70 kN/m2. The calculation is
to be carried out for the reservoir depth of 18 m and for the case where the
reservoir has been completely emptied. In the calculations, the forces Ww and
U will have moments about the centre of the circle and therefore must be
evaluated.

FIGURE 10.6
Section of a dam to be constructed in Seshego

290 GEE2601/1
2. A section of an embankment in a road construction project in Calvinia, Northern
Cape, is shown in figure 10.7 below. Using the ordinary method of slices,
determine the factor of safety for the slope undergoing seepage and for the
failure surface. The soil properties are as follows:
• total density = 2 Mg/m3
• effective cohesion c' = 30 kN/m2
• effective friction angle φ' = 30°

FIGURE 10.7
An embankment in a road construction project in Calvinia, Northern Cape

3. Using the simplified Bishop method, determine the factor of safety for the
previous problem. This is the same problem that you solved by means of the
ordinary method of slices.

291 GEE2601/1
10.10 SOLUTIONS TO SELF-ASSESSMENT ACTIVITIES

Activity 10.1: Prescribed book exercises

1. Consult the solution book for answers.


2. Consult the solution book for answers.
3. Consult the solution book for answers.

Activity 10.2: Multiple-choice questions

1. (a)
2. (d)
3. (b)
4. (c)
5. (b)
6. (a)
7. (c)
8. (b)
9. (a)
10. (d)
11. (b)
12. (a)
13. (c)
14. (b)
15. (d)

Activity 10.3: Typical exam questions

1. Factor of safety F = 2.75


2. Factor of safety F = 1.73
3. Factor of safety F = 1.86

292 GEE2601/1
10.11 SUMMARY

In this study unit we discussed slope stability analysis in detail. You should be able to
analyse a variety of slopes to determine the factor of safety against sliding along a trial
failure surface. In this study unit you have learnt that slope angles and shear strength
parameters govern the position of the probable failure surface and that a carefully
planned search for the critical failure surface has to be performed in many cases. You
studied methods of analysis for infinite slopes with and without seepage, and for finite
slopes with circularly cylindrical failure surfaces, using mass procedures and the
methods of slices for undrained and drained conditions, including seepage.

10.12 VIDEO CLIPS, SLIDES AND OTHER WEB-BASED


RESOURCES

For additional explanations on slope stability, you can watch some video clips. Web
addresses/links to the video clips are available on the module site under Additional
Resources in a folder named Video clips, slides and other web-based resources:

• Slope stability: methods of slices:


https://www.youtube.com/watch?v=2wT2he6Numk
• Mod-05 Lec-40 Lecture 1 on stability of slopes:
https://www.youtube.com/watch?v=s87MHDA5evM
• Introduction to slope stability soil mechanics:
https://www.youtube.com/watch?v=rlCmaYi2rYk
• Slope stability:
https://www.youtube.com/watch?v=xg-Gw1NrkX8
• Bishop's simplified method:
https://www.youtube.com/watch?v=QVeSQOB1eBQ

293 GEE2601/1
REFERENCE WORKS AND FURTHER READING

Bishop, AW. 1955. The use of the slip circle in the stability analysis of slopes.
Geotechnique, 5:7 - 17.

Das, BM. 2010. Principles of geotechnical engineering. 7th ed. Connecticut: Cengage
Learning.

Das, BM. 2013. Fundamentals of geotechnical engineering. 4th ed. Connecticut:


Cengage Learning.

Das, BM & Sivakugan, N. 2017. Fundamentals of geotechnical engineering. 5th ed.,


international edition. Connecticut: Cengage Learning.

Das, B.M. & Sobhan, K. (2014). Principles of Geotechnical Engineering, 8th Edition,
SI Edition. CENGAGE Learning. ISBN 13: 9788131521588 Connecticut, United
States of America.

Fellenius, W. 1927. Erdstatische Berechnungen. Revised ed. Berlin: W Ernst u. Sons.

Taylor, DW. 1937. Stability of earth slopes. Journal of the Boston Society of Civil
Engineers, 24:197-246.

* Page, figure, table and chapter numbers may differ on updated and revised versions of the prescribed
book. See online version of study guide for updates.

294 GEE2601/1
Study unit 1 11

SHALLOW FOUNDATIONS

11.1 INTRODUCTION

The function of a foundation is to transfer the load of a structure to the soil on which it
is resting. In this sense “structure” refers to any construction or part of it and includes
buildings, bridges, towers, embankments, earth-fill dams, and many more. In the case
of embankments and earth-fill dams the foundations are the bottoms of the structures.
What about the feet of a person standing on the ground? The foundation must transfer
the load to the soil in such a way that the soil is not overstressed, because
overstressing may lead to shear failure in the soil, or to excessive settlement. Both
cause damage to the structure.

There are many types of foundations that can be used to support structures,
depending on the structure's geometry and weight, on the soil properties, or on
economy, and you can read about them in the introductory part of chapter 11 in the
prescribed book. In this study unit we will discuss only shallow foundations. These are
usually defined as foundations no deeper than about four times their width. Shallow
foundations may be divided into the following types:

• Strip foundations support continuous walls or a row of closely spaced columns.


• Pad footings, or spread footings, carry single columns or groups of columns.
• Raft foundations are large pads that support whole structures, usually high-rise
buildings, or they are used on poor soils and collapsing sand.

295 GEE2601/1
Many structures are, or can be, supported by shallow foundations, including houses,
flats, office buildings, factories and retaining walls. The geotechnical engineer and
technician must evaluate the soil profile, soil properties and the structural loads to
decide on the depth below surface at which the foundation is to be constructed, as
well as on the shape and size of the foundation, to prevent excessive settlement or
shear failure of the soil. Shear strength determines the ultimate bearing capacity, and
compressibility governs the settlement.

Since this is a course in geotechnical engineering, we will discuss only the


geotechnical aspects of foundations. The foundation itself is regarded as a structural
element and its design, like concrete thickness and reinforcement, should be done by
structural experts. Chapter 16 (pp. 568 -611*) in the prescribed book covers this study
unit.

11.2 DEFINITIONS

You will come across the following terms in this study unit:

Foundation: An element of a structure which connects it to the ground and transfers


all the loads from the structure to the ground.
Shear failure: An occurrence in which the shear stresses in the soil exceed the shear
strength of the soil.
Ultimate bearing capacity: The maximum load per unit area at which shear failure in
the soil below the foundation does not occur.

11.3 LEARNING OUTCOMES

On completion of this study unit, you should be able to calculate the ultimate bearing
capacity of shallow foundations. More specifically, you should be able to adjust the
bearing capacity calculations to allow for

• foundation depth
• partial submergence

296 GEE2601/1
• inclined loads
• eccentric loads
• drained or undrained conditions
• estimate settlement of foundations on clay and sand, which may be the limiting
factor on allowable bearing pressure

To achieve the learning outcomes in this unit, work through the following topics:

• ultimate bearing capacity of shallow foundations


• effects of founding depth, shape, inclined loads, eccentric loads and submergence
on ultimate bearing capacity
• immediate settlement of shallow foundations on clay
• settlement of shallow foundations on sand

11.4 Ultimate bearing capacity of foundations

Sections 16.1 and 16.2 (pp. 568-571*) in chapter 16 in the prescribed book deal with
this topic. Ultimate bearing capacity of a foundation is defined as the maximum load
per unit area at which shear failure in the soil below the foundation does not occur.
Depending on the density and stiffness of the soil, the extent of the shear failure zone
and the vertical movement of the foundation may be different, as shown in figure 16.1
(p. 570*) in the prescribed book. The following are some characteristics of general
shear failure:

• It occurs over dense sand or stiff cohesive soil.


• It involves total rupture of the underlying soil.
• There is a continuous shear failure of the soil from below the footing to the ground
surface (solid lines on figure16.1).
• When the (load/unit area) is plotted versus settlement of the footing, there is a
distinct load, called ultimate load (Qu) at which the foundation fails.
• The value of Qu divided by the area of the footing is considered to be the ultimate
bearing capacity of the footing (qu).

297 GEE2601/1
• As shown in figure 16.1, a general shear failure rupture occurs and pushes up the
soil on both sides of the footing.
• However, for actual failures on the field, the soil is often pushed up on only one
side of the footing with subsequent tilting of the structure.

11.4.1 Ultimate bearing capacity of a continuous foundation

Sections 16.3 and 16.4 (pp. 571-577*) in the prescribed book cover this topic.
Terzaghi (1943) presented the first theory on ultimate bearing capacity of rough
shallow continuous foundations. An example of continuous foundations is a strip
foundation. He assumed that the shape of the zone of failure in the soil would be as
shown in figure 16.3 (p. 571*) in the prescribed book. His theory is, strictly speaking,
only applicable to foundations on medium dense to dense sand or medium stiff to stiff
cohesive soil. Below the foundation a triangular zone of soil fails and moves down with
the foundation, leading to two zones of radial shear failure adjacent to the triangular
zone and two more triangular zones of failure above these. You can study the
geometries of the different zones in the figure. The soil above foundation level (the
dotted line in the figure) is regarded merely as surcharge on the failing zones.

Other theories were developed later, which differed from Terzaghi's theory only in that
slightly differently shaped failure zones were assumed. Terzaghi's theory is still
regarded as correct and is used extensively, with some modifications to allow for
special conditions. The ultimate bearing capacity for a strip foundation is expressed in
the form

• qu = cNc + qN q + 21 γBN γ (strip foundation) (11.1)

Where:

• qu = Ultimate bearing capacity


• c = Cohesion
• Nc, Nq and NƔ = Bearing capacity factors
• B = Width of foundation

298 GEE2601/1
• q = Column load
• 𝛾𝛾 = Unit weight of soil

The three terms in the equation take into account the contributions of three sources of
resistance to bearing failure. They can be summarised as follows:

• The first term, called the cohesion term, is the contribution of the cohesion of the
soil within and just below the failure zones. Therefore (c) should be the cohesion
of the soil immediately below the foundation. c is used when effective stress
analyses are conducted and cu for undrained analyses.
• The second term is called the depth term, and it is the contribution from the
overburden soil that rests on top of the failure zones. The shear strength of the
overburden soil is ignored in Terzaghi's theory, but the overburden pushes down
on the triangular failure zones on both sides of the foundation and adds to the
shearing resistance of the soil below foundation level, almost like a surcharge
increases active and passive pressures against a retaining wall. The value of q is
calculated as q = γDf, where Df is the depth to the foundation level and γ is the unit
weight of the soil above that level. If there is any chance of the depth of overburden
being reduced by excavation, the minimum possible value of q must be used.
• The third term is the contribution from the frictional resistance of the soil within the
failure zones, and its value depends on the width B of the foundation. It is
commonly known as the width term. The value of γ in this term is that of the soil
immediately below the foundation, in which the failure zones develop.

The three bearing capacity factors (N-factors) in the equation were derived from the
shapes of the assumed failure zones and can be calculated from equations 16.6 to
16.8 (pp. 573-574*) in the prescribed book. It is evident that the factors depend on the
value of the angle of internal friction. φ' is used in effective stress analyses and φu in
total stress analyses. Values of the N-factors for φ between 00 and 450 are listed in
table 16.2 (p. 574*) in the prescribed book. The short-term ultimate bearing capacity
of foundations on clay can be determined by using cu and φu in the calculations.

299 GEE2601/1
Note:

• φ = 0 cannot be used in the equation for Nc.


• The value of Nc = 5.14 in the table was obtained by plotting the equation and
extrapolating to φ = 0.
• Since Nq = 1 and Nγ = 0, the ultimate bearing capacity equation is simplified in this
case to qu = 5,14c u + q = 5,14c u + γD f

11.4.2 Ultimate bearing capacity of a rectangular foundation

Terzaghi's equation was derived for a continuous foundation. It was later modified by
De Beer (1970) to provide for rectangular foundations by introducing shape factors to
multiply by the three terms of the expression. The equation for ultimate bearing
capacity then becomes

qu = cNc Fcs + qN qFqs + 21 γBN γ Fγs


• (11.2)

Where:
• qu = Ultimate bearing capacity
• c' = Cohesion
• Ɣ = Unit weight of soil
• Nc, Nq and NƔ = Bearing capacity factors
• Fcs, Fqs, FƔs = Shape factors

The values of the three shape factors depend on the ratio B:L of the foundation and
expressions for their calculation are found in table 16.3 (p. 575*) in the prescribed
book. A circular foundation may be given the same shape factors as a square
foundation.

300 GEE2601/1
Example 11.1

Compare the ultimate bearing capacity of a 2 m wide continuous foundation for a


double-storey house in Northcliff with that of a 4 m × 2 m rectangular foundation of a
similar house. Both are at 2 m below the surface in a dry soil for which c' = 5 kN/m2, φ'
= 300 and γ = 18 kN/m2.

Solution

• The value of the surcharge is q = γDf = 18 × 2 = 36 kN/m2


• For φ' = 30, obtain the bearing capacity factors from table 16.2 (p. 574*) in the
prescribed book:
• Nc = 30.14; Nq = 18.40; Nγ = 22.4

For the continuous foundation, use equation 11.3 in the prescribed book:

qu = cNc + qNq + 0.5γBNγ = (5 × 30.14) + (36 × 18.40) + (0.5 × 18 × 2 ×


22.40)
= 150.7 + 662.4 + 403.2 (note the relative magnitudes of the three
terms)
= 1 216.3 kN/m2

For the rectangular foundation, determine the shape factors from table 16.3 (p. 575*)
in the prescribed book:
B Nq 2
Fcs = 1 + = 1 + 0,61 = 1,305
L Nc 4
B 2
Fqs = 1 + tan φ = 1 + 0,58 = 1,290
L 4
B 2
Fγs = 1 − 0,4 = 1 − 0,4 = 0,8
L 4

qu = cFcsNc + qFqsNq + 0,5γBFγsNγ


= (5 × 1.305 × 30.14) + (36 × 1.29 × 18.40) + (0.5 × 18 × 2 × 0.8 × 22.40)
= 196.7 + 854.5 + 322.6 = 1 373.72 kN/m2

301 GEE2601/1
11.4.3 Modification of bearing capacity equation for foundation depth and
load inclination

Hansen (1961) proposed a modification to Terzaghi's equation to take into account


the shear strength of the soil above the foundation level. A second set of factors, Fcd,
Fqd and Fγd, are to be multiplied into the terms of the ultimate bearing capacity
equation. The values of these depth factors are calculated from the expressions in
table 16.3 (p. 575*) in the prescribed book. The φ to be used in these expressions is
that of the soil above the foundation level. Some engineers are reluctant to use
Hansen's modification because the soil above foundation level often has low shear
strength and does not really contribute much to the bearing capacity.

A third modification was proposed by Meyerhof (1953) to allow for inclined loads.
Multiplication factors Fci, Fqi and Fγi were introduced and expressions for these are
found in table 16.2 (p. 574*) in the prescribed book. From the expressions it is clear
that the factors are less than unity, which means that a shallow foundation is not very
effective when it comes to inclined loads. Yet, many foundations must carry inclined
loads, for example foundations of retaining walls that are subject to lateral pressures.
The equation for ultimate bearing capacity with all the modifications is given by
equation 16.9 (p. 574*) in the prescribed book.

Example 11.2

Repeat example 11.1 but take the shear strength above foundation level into account,
according to the method by Hansen. Assume that the same shear strength parameters
apply below and above foundation level.

Solution

Calculate the depth factors according to the expressions in table 16.3 (p. 575*) in the
prescribed book:

Df 2
Fcd = 1 + 0,4 = 1 + 0,4 = 1,4
B 2

302 GEE2601/1
Df 2 2
Fqd = 1 + 2 tan φ(1 − sin φ) = 1 + 2 tan 30(1 − sin 30 )
2
= 1,29
B 2
Fγd = 1

For the continuous foundation:

qu = c'NcFcsFcdFci + qNqFqsFqdFqi+ 0,5γB NγFγs FγdFγd


= (5 × 1.4 × 30.14) + (36 × 1.29 × 18.40) + (0.5 × 18 × 2 × 1.0 × 22.40)
= 211.0 + 854.5 + 403.2 (note how the magnitudes of the terms changed)
= 1 468.7 kN/m2

For the rectangular foundation:

qu = cFcsFcdNc + qFqsFqdNq + 0,5γBFγsFγdNγ


= (5 × 1.305 × 1.4 × 30.14) + (36 × 1.29 × 1.29 × 18.40) + (0.5 × 18 × 2 ×
0.8 × 1.0 × 22.40)
= 275.3 + 1102.2 + 322.6 (note how the magnitudes of the terms
changed)
= 1 700.1 kN/m2

Example 11.3

Repeat example 11.2 but assume that the load is inclined at an angle of 15° to the
vertical.

Solution

Use Meyerhof's inclination factors from table 16.3 (p. 576*) in the prescribed book:
2 2
 β   15 
Fci = Fqi = 1 −  = 1 −  = 0,694
 90   90 
2 2
 β  15 
Fγi = 1 −  = 1 −  = 0,25
 φ  30 

303 GEE2601/1
For the continuous foundation:
qu = cFcdFciNc + qFqdFqiNq + 0,5γBFγdFγiNγ
= (5 × 1.4 × 0.69 × 30.14) + (36 × 1.29 × 0.69 × 18.40) + (0.5 × 18 × 2 × 1.0 ×
0.25 × 22.40)
= 145.5 + 589.7 + 101.3 (note how the magnitudes of the terms changed)
= 836.5 kN/m2

For the rectangular foundation:


qu = cFcsFcdFciNc + qFqsFqdFqiNq + 0,5γBFγsFγdFγiNγ
= (5 × 1.305 × 1.4 × 0.69 × 30.14) + (36 × 1.29 × 1.29 × 0.69 × 18.40) + (0.5 × 18 × 2 ×
0.8 × 1.0 × 0.25 × 22.4)
= 190.0 + 760.5 + 80.7 (note how the magnitudes of the terms changed)
= 1 031.2 kN/m2

11.4.4 Net ultimate bearing capacity

The net ultimate bearing capacity is defined as the ultimate vertical pressure that can
be supported by the soil in excess of the pressure of the overburden at foundation
level. A practical way of viewing this is that net ultimate bearing capacity is the increase
in vertical pressure in the soil at foundation level that will bring about shear failure in
the soil. The vertical stress was q = γDf before construction started. When the soil fails
in shear, the vertical pressure is qu. The increase, or net ultimate pressure, is therefore

• qnet(u) = qu – q = qu – γDf (11.3)

11.5 EFFECT OF TOTAL OR PARTIAL SUBMERGENCE

Section 16.5 (pp. 577-578*) in the prescribed book covers this topic. Equation 11.1
was developed by Terzaghi for a foundation in dry soil. If the water table is situated
close to the surface, the effect of submergence must be taken into account when
ultimate bearing capacity is calculated. The main effect is that the submerged part of
the soil is subject to buoyancy, which is important in effective stress calculations, and

304 GEE2601/1
a buoyant unit weight of the submerged soil must be used. Three possible cases are
described in the prescribed book, and averaging of buoyant and bulk unit weights is
suggested over certain depths. A simpler and more conservative approach would be
the following:

• Case I: If the water table is anywhere above foundation level (see figure 16.4 (p.
577*) in the prescribed book), use γ' = γsat – γw for all soils above and below
foundation level to calculate qu.
• Case II: If the water table is below foundation level but within a depth of d = B of
the foundation level, use γ for the soil above foundation level (the surcharge) and
γ' for the soil below foundation level.
• Case III: If the water level is below a depth of d = B of the foundation level, use γ
for all soils. The foundation is not affected by the submergence in this case.

Example 11.4

Recalculate the ultimate bearing capacity of the continuous foundation of example


11.3 for the following conditions:

a) The water table rises to just below foundation level


b) The water table is above foundation level
The submerged part has a unit weight of 20 kN/m3 and the unit weight of water is
taken as 10 kN/m3.

Solution
a) With the water table just below foundation level, case II above applies, and γ'
should be used in the depth term, so that
• q = 2 x (20 – 10) = 20 kN/m2:
• qu = cFcdFciNc + qFqdFqiNq + 0,5γBFγdFγiNγ
= (5 × 1.4 × 0.69 × 30.14) + (20 × 1.29 × 0.69 × 18.40) + (0.5 × 18 × 2 × 1.0 ×
0.25 × 22.40)
= 145.5 + 327.6 + 101.3 (note how the magnitudes of the terms changed)
= 574 kN/m2

305 GEE2601/1
b) With the water table above foundation level, case I above applies and γ' = 10
kN/m3 should be used everywhere.
• qu = cFcdFciNc + qFqdFqiNq + 0,5γBFγdFγiNγ
= (5 × 1.4 × 0.69 × 30.14) + (20 × 1.29 × 0.69 × 18.40) + (0.5 × 10 × 2 ×
1.0 × 0.25 × 22.40)
= 145.5 + 327.6 + 56.3 (note how the magnitudes of the terms changed)
= 529.4 kN/m2

Note how the values of the three terms of the bearing capacity equation were changed
by the various conditions. At this stage it may be interesting to compare the answers
of examples 11.1 to 11.4 to evaluate the overall effects of the different conditions that
applied. From the table below you can see that a rectangular foundation has a larger
ultimate bearing capacity than a continuous foundation, as a result of three-
dimensional effects at shear failure. Including the shear strength of the soil above
foundation level increased the value of qu, but inclination of the load dramatically
reduced qu. Partial or total submergence led to further reduction of the ultimate bearing
capacity. The geotechnical engineer and technician need to be very careful to make
provision for possible negative effects like inclined loads and submergence.

TABLE 11.1: Overall effects of the different conditions.

Continuous foundation Rectangular foundation


Condition
B = 2 m, Df = 2 m L = 4 m, B = 2 m, Df = 2 m
Dry soil Vertical load
Shear above Df ignored qu = 1216 1374
Dry soil Vertical load
Shear above Df included 1469 1700
Dry soil Inclined load ##
Shear above Df included 836 1031
WT at foundation level
Inclined load ## Shear
574 693 #
above Df included

306 GEE2601/1
WT at surface Inclined
load ## Shear above Df
529 657 #
included

# Calculation of these values is not shown. ## Although the load is inclined, qu is the
ultimate vertical bearing capacity.

11.6 FACTOR OF SAFETY AND ALLOWABLE BEARING CAPACITY

Section 16.6 (pp. 578-581*) in the prescribed book covers this topic. The net ultimate
bearing capacity should be divided by a suitable factor of safety to determine the
allowable net bearing capacity, also called the net safe bearing capacity. Factors of
safety of at least 3 are used to allow for soil variability and certainties about the
average shear strength parameters of the soil. If a foundation has to carry a known
bearing stress qf, the equation for FS is

qnet (u ) qu − γD f
• FS = = (11.4)
qnet q f − γD f

If the allowable bearing capacity is desired, it may be calculated from

qu − γD f
• qall = + γD f (11.5)
FS

The value of qall is multiplied by the area of the foundation, to obtain the vertical force
or load that the foundation is allowed to carry, including its own weight

• Qall = qall Area = qall LB (11.6)

Where:

• B = Width of the foundation = smaller dimension


• L = Length of the foundation = larger dimension

307 GEE2601/1
Another way of allowing for uncertainties is to apply the safety factor to the shear
strength parameters c' and tan φ' before calculating qu, similar to slope stability
analyses, and at the same time to increase the load on the foundation by a load factor.
Until this relatively new method has been fully employed in this country, we will stick
to the safety factor method.

Note:

• All bearing capacity safety factor calculations are based on net values, like in
equations 11.4 and 11.5 above.

Example 11.5

The load of steel structure in Bryanston (Johannesburg) rests on a square footing. The
load is 500 kN (including the weight of the footing). The foundation depth is 0.5 m and
the water table may rise to the surface. The saturated unit weight of the soil is 18.075
kN/m3, c' = 30 kN/m2 and φ' = 18°. Use a factor of safety of 2.5 and determine the
required width of the foundation.

Solution

Since the water table may rise to the surface, the ultimate bearing capacity must be
calculated using unit weight = 18.75 – 10 = 8.75 kN/m3. Bearing capacity factors can
be calculated from equations 11.4 to 11.6 or read from table 16.2 (p. 573*) in the
prescribed book. For φ' = 18°:

• Nc = 13.10; Nq = 5.26; Nγ = 4.07

The shape factors are calculated from De Beer's equations in table 16.3 (p. 575*) in
the prescribed book:

• Fcs = 1 + Nq / Nc = 1.40 (because L = B)


• Fqs = 1 + tan18° = 1.32
• Fγs = 1 – 0.4 = 0.60

308 GEE2601/1
The ultimate bearing capacity can now be calculated from equation 11.7 in the
prescribed book, without taking the shear strength of the soil above foundation level
into account:

• qu = cNcFcs + qNqFqs + 0,5γBNγFγs


= (30 × 13.10 × 1.40) + ((8.75 × 0.5) × 5.26 × 1.32) + (0.5 × 8.75 × B × 4.07 ×
0.6)
= 550.2 + 30.4 + 10.68B
= 580.6 + 10.68B

The net ultimate bearing capacity is

• qnet(u) = qu – γDf = 580,6 + 10,68B – 8,75 × 0,5 = 576,2 + 10,68B

The safety factor is defined in terms of net bearing values, according to equation 11.5:

qu − γD f 576,2 + 10,68B
qall = + γD f = + 8,75 × 0,5
FS 2,5

The foundation must carry 500 kN, so that the actual bearing pressure is
• qf = 500/B2, and this must not exceed qall.
500 576,2 + 10,68B
• If qf = qall: = + 4,375
B 2
2,5

Solving B gives the required minimum width of the foundation:


B = 1.44m

The actual bearing pressure is


qf = 500/B2 = 241 kN/m2

Example 11.6

A continuous foundation for a building in Johannesburg is designed to carry a load of


800 kN/m at a foundation depth of 0.7 m in sand. The shear strength parameters of

309 GEE2601/1
the sand are c' = 0 and φ' = 40°. Determine the width of the foundation if a factor of
safety of 3 against shear failure is specified. The water table may rise to foundation
level. Above the water table the unit weight is 17 kN/m3 and below the water table the
saturated unit weight is 20 kN/m3. Ignore the shear strength of the soil above
foundation level.

Solution

For φ'= 40°, table 11.1 in the prescribed book gives Nc = 75.31, Nq = 64.20 and Nγ =
109.41. The ultimate bearing capacity is calculated from equation 11.3 in the
prescribed book:

• qu = cNc + qNq + 0,5γBNγ


= (0) + (17 × 0.7 × 64.20) + (0.5 × (20 – 10) × B × 109.41)
= 764.0 + 547.1B
• qnet(u) = 764.0 + 547.1B – 17 × 0.7 = 752.1 + 547.1B

Actual net bearing pressure is


• qnet = 800/B – 17 × 0.7 = 800/B -11.9

With a factor of safety of 3 on net bearing capacity


• (800/B -11.9) = (752.1 + 547.1B)/3

From which
• B = 1.50 m

11.7 ECCENTRICALLY LOADED CONTINUOUS FOUNDATIONS

Section 16.7 (pp. 581-584*) in the prescribed book covers this. Exclude the part on
foundations with two-way eccentricity. If a foundation is subjected to a moment M in
addition to the vertical load P, the bearing pressure on the soil is not uniform, as shown

310 GEE2601/1
in the figure in the prescribed book. For relatively small moments the pressure is
assumed to vary between a maximum and minimum value, given by

Q  6e 
• q max min = 1 ±  (11.7)
BL  B 

The eccentricity is calculated from e = M/Q. If the moment is large enough that e =
B/6, the minimum pressure will be equal to zero. For an even larger moment, part of
the foundation will lose contact with the soil, in which case the maximum stress will
become

4Q
• qmax = (11.8)
3L(B − 2e)

Figure 16.6(a) (p. 582*) in the prescribed book shows these pressure distributions.
The non-uniform stress distribution is accounted for in bearing capacity analyses by
reducing the width of the foundation by 2e so that the effective width is B′ = B – 2e.
This effective width is used in all subsequent calculations. The load is assumed to be
uniformly distributed over B′, B′ is used to calculate the bearing capacity shape factors
Fcs, Fqs and Fγs, and B′ is used in the width term of the ultimate bearing capacity
equation and in the calculation of the effective foundation area. One exception is that
B, and not B′, is used for the calculation of depth factors Fcd, Fqd and Fγd because the
eccentricity does not influence the shear strength contribution from the soil above
foundation level. Calculation of the safety factor is in terms of net bearing pressures
as before:

qnet (u ) qu − γD f qu − γD f
• FS = = = (11.9)
qnet q f − γD f Q
− γD f
B'
 q − γD f 
• Q all = qall ( B'×1) =  u + γD f  B' (11.10)
 FS 

311 GEE2601/1
Since continuous foundations are considered here, Q or Qall is the load per unit length
of the foundation, in kN/m. Note that a more conservative approach is to compare the
net ultimate bearing capacity with the maximum foundation pressure qmax in the
calculation of the factor of safety, instead of using the reduced width B′. Equation 11.9
is then modified to

qnet (u ) qu − γD f
• FS = = (11.10a)
qnet (max) qmax − γD f

Example 11.7

A retaining wall for a shopping mall parking lot on a cliff in Midrand is supported on a
3 m wide continuous foundation, at 1 m depth in a dry soil for which the unit weight is
18 kN/m3. The resultant load on the foundation is 300 kN/m and its inclination is 20°
from the vertical. The eccentricity of the base reaction is 0.36 m. If the shear strength
parameters of the soil are c' = 10 kN/m2 and φ' = 35°, determine the factor of safety
against shear failure. Ignore shear strength above foundation level.

Solution

Calculate the effective width of the base:


• B' = B – 2e = 3 – 2 x 0.36 = 2.28 m
Find the bearing capacity factors from table 16.2 (p. 574*) in the prescribed book:
• Nc = 46.12; Nq = 33.30 & Nγ = 48.03

Calculate Meyerhof's inclination factors from the expressions in table 16.3 (p. 576*)
in the prescribed book:
• Fci = Fqi = (1 – 20/90)2 = 0.61
• Fγi = (1 – 20/35)2 = 0.18

312 GEE2601/1
From equation 16.9 (p. 575*) in the prescribed book
• qu = (10 × 46.12 × 0.61) + (18 × 1 × 33.30 × 0.61) + (0.5 × 18 × 2.28 × 48.03 ×
0.18)
= 281.3 + 365.6 + 177.4
= 824.3 kN/m2
• qnet×(u) = 824.3 – 18 × 1 = 806.3 kN/m2

The actual net bearing pressure is


• qnet = (300 cos 20°)/2.28 – 18 × 1 = 105.6 kN/m2
• Factor of safety: FS = 806.3/105.6 = 7.63

11.8 SELF-ASSESSMENT ACTIVITIES

You now have the opportunity to apply what you have learnt in this unit. Answer the
following questions:

Activity 11.1: Prescribed book exercises

1. Read the summary (p. 607*) of chapter 16.


2. Do problems 16.1 to 16.5 (pp. 607-608*).
3. Do problems 16.6 to 16.9 (p. 608*).
4. Do problems 16.10 to 16.16 (pp. 608-609*).

Activity 11.2: Multiple-choice questions

1. The load of a structure is transferred to the foundation soil through …


(a) the floor slab.
(b) spread footings.
(c) piles and drilled shafts.
(d) Both (b) and (c).

313 GEE2601/1
2. Overstressing the foundation soil can result in …
(a) excessive settlement.
(b) shear failure of the soil.
(c) Both (a) and (b).
(d) functional failure of the structure.

3. In soil with low load-bearing capacity, it is more economical to construct the entire
structure over …
(a) spread footings.
(b) a mat foundation.
(c) Both (a) and (b).
(d) None of the above.

4. For heavier structures when great depth is required for supporting the load, which
of the following foundations is generally recommended?
(a) spread footings
(b) mat foundation
(c) piles and drilled shafts
(d) Both (b) and (c).

5. Which of the following foundations are generally referred to as shallow


foundations?
(a) spread footings and mat foundations
(b) spread footings and pile foundations
(c) mat foundations and pile foundations
(d) pile and drilled shaft foundations

314 GEE2601/1
6. In general, shallow foundations are those that have a depth of embedment to
width ratio of approximately …
(a) equal to 4.
(b) less than 4.
(c) greater than 4.
(d) None of the above.

7. The ultimate load per unit area of a shallow foundation which will cause shear
failure of the soil supporting the foundation is called …
(a) bearing capacity.
(b) allowable bearing capacity.
(c) ultimate bearing capacity.
(d) None of the above.

8. When the general shear failure of the foundation soil takes place, the failure
surface in soil extends to …
(a) the ground surface.
(b) the base of the footing.
(c) the water table present in the soil.
(d) any arbitrary level.

9. In which of the following failure modes does the failure surface in the foundation
soil not extend to the ground surface?
(a) general shear failure
(b) local shear failure
(c) punching shear failure
(d) Both (b) and (c).

315 GEE2601/1
10. In fairly loose foundation soil, what type of failure mode is expected?
(a) general shear failure
(b) local shear failure
(c) punching shear failure
(d) Both (b) and (c).

11. For foundations of a shallow depth, the ultimate load with a general shear failure
of soil may occur at a foundation settlement of …
(a) 4% to 10% of the depth of foundation.
(b) 4% to 10% of the width of foundation.
(c) 15% to 25% of the depth of foundation.
(d) 15% to 25% of the width of foundation.

12. In Terzaghi's ultimate bearing capacity theory, the effect of soil above the bottom
of the foundation may be assumed to be replaced by an equivalent surcharge
… where the symbols have their usual meanings.
(a) q = 0.5γDf
(b) q = γDf
(c) q = 1.5γDf
(d) q = 2γDf

13. According to Terzaghi's ultimate bearing capacity equation, the ultimate bearing
capacity of a strip footing resting on a clayey ground in undrained condition is

(a) qu = 0.
(b) qu = c'.
(c) qu = 5.14c'.
(d) qu = 5.7c'.

316 GEE2601/1
14. According to Terzaghi's ultimate bearing capacity equation, the ultimate bearing
capacity of square and circular footings resting on a clayey ground in undrained
condition is …
(a) qu = 0.
(b) qu = 1.3c'.
(c) qu = 6.68c'.
(d) qu = 7.41c'.
15. The bearing capacity factor Nγ is given as … where the symbols have their
usual meanings.
(a) Nγ = (Nq – 1) cot φ'
(b) Nγ = (Nq + 1) tan φ'
(c) Nγ = 2(Nq + 1) tan φ'
(d) Nγ = 2(Nq - 1) tan φ'
16. If the angle of friction of the foundation soil is 0°, …
(a) Nq = 0.
(b) Nq = 1.
(c) Nq = 5.7.
(d) Nq = 5.14.

17. If the angle of friction of the foundation soil, φ′ = 0°, …


(a) Nγ = 0.
(b) Nγ = 1.
(c) Nγ = 5.7.
(d) Nγ = 5.14.

18. The general bearing capacity equation considers the following:


(a) shape factors
(b) depth factors
(c) load inclination factors
(d) All of the above.

317 GEE2601/1
19. If the difference between the unit weight of concrete used in the foundation
and the unit weight of soil surrounding the foundation is assumed to be
negligible, then the net ultimate bearing capacity … where the symbols have
their usual meanings.
(a) qnet(u) = qu
(b) qnet(u) = q
(c) qnet(u) = quv - q
(d) qnet(u) = quv + q

20. The water table will have no effect on the ultimate bearing capacity when …
(a) it is located above the base of the footing.
(b) it is located at the base of the footing.
(c) it is located below the base of the footing.
(d) its depth below the base of the footing is greater than the width of the
footing.

21. If the net ultimate bearing capacity of a foundation is 30 t/m2, its net allowable load
bearing capacity will be …
(a) 10 t/m2.
(b) 15 t/m2.
(c) 30 t/m2.
(d) 90 t/m2.

Activity 11.3: Typical exam questions

1. The square footing shown below for a building project in Lynnwood Ridge
(Pretoria) must be designed to carry a 2 400 kN load. Using Terzaghi's bearing
capacity formula and a safety factor of 3, determine the foundation dimension B
in the following two cases:

(a) The water table is at 1 m below the foundation (as shown).


(b) The water table rises to the ground surface.

318 GEE2601/1
2. From the same building project (Lynnwood Ridge, Pretoria) mentioned in Q. 1,
an additional square footing is required for an outside building. Using Terzaghi's
equation and assuming general shear failure, determine the size of this
additional footing to carry a net allowable load of 295 kN. Use the same safety
factor as in Q. 1. Refer to the sketch below.

11.9 SOLUTIONS TO SELF-ASSESSMENT ACTIVITIES

Activity 11.1: Prescribed book exercises

1. Consult the solution book for answers.


2. Consult the solution book for answers.
3. Consult the solution book for answers.

319 GEE2601/1
Activity 11.2: Multiple-choice questions

1. (d)
2. (c)
3. (b)
4. (c)
5. (a); Discussion: (d) is correct for deep foundations.
6. (b); Discussion: (c) is correct for deep foundations.
7. (c)
8. (a)
9. (d)
10. (c)
11. (b)
12. (b)
13. (d)
14. (c)
15. (c)
16. (b)
17. (a)
18. (d)
19. (c)
20. (b)
21. (c)

Activity 11.3: Typical exam questions

1. (a) B =1.33 m; (b) B = 1.42 m


2. B = 0.68 m

320 GEE2601/1
11.10 SUMMARY

This study unit covered bearing capacity of shallow foundations. By now, you should
have a good understanding of the factors that control ultimate bearing capacity of
continuous and spread footings. (Raft foundations and two-way eccentricity of spread
footings are not covered in the course.) You should understand that shear failure of
the soil is not the only way in which a footing may be rendered incapable of supporting
a structure, and that excessive settlement may also be regarded as failure of the
foundation.

The ultimate bearing capacity of shallow foundations/footings has been proved to be


governed by many factors, in addition to the properties of the soil in which they rest.
These factors include foundation shape, foundation size, foundation depth below
surface, submergence and inclination and eccentricity of the applied load. The ultimate
bearing capacity may be very sensitive to relatively small variations in these factors;
you must be very careful when selecting values to be used in the calculations. Special
care should be taken in the determination of the internal friction angle, since the
bearing capacity factors change dramatically with a 1° variation in φ' over the whole
spectrum, but especially at φ' > 30°.

11.11 VIDEO CLIPS, SLIDES AND OTHER WEB-BASED


RESOURCES

For additional explanations on shallow foundations, you can watch some video clips.
Web addresses/links to the video clips are available on the module site under
Additional Resources in a folder named Video clips, slides and other web-based
resources:

• CP5 5 shallow foundations:


https://www.youtube.com/watch?v=RTPXIP-pjs4
• Mod-01 Lec-06 Shallow foundation: bearing capacity –I:
https://www.youtube.com/watch?v=YEltCJUZ0hk

321 GEE2601/1
• Basics of shallow foundation:
https://www.youtube.com/watch?v=frntXHVXkzI

REFERENCE WORKS AND FURTHER READING

Al-Agha, AS. 2015. Basics of foundation engineering with solved problems. Based on
“Principles of foundation engineering. 7th edition”.

Das, BM. 2010. Principles of geotechnical engineering. 7th ed. Connecticut: Cengage
Learning.

Das, BM. 2013. Fundamentals of geotechnical engineering. 4th ed. Connecticut:


Cengage Learning.

Das, BM & Sivakugan, N. 2017. Fundamentals of geotechnical engineering. 5th ed.,


international edition. Connecticut: Cengage Learning.

De Beer, EE. 1970. Experimental determination of the shape factors and the bearing
capacity factors of sand. Geotechnique, 20:387–411.

Hansen, JB. 1961. A general formula for bearing capacity. The Danish Geotechnical
Institute Bulletin 11:38-46. Copenhagen.

Meyerhof, GG. 1953. The bearing capacity of foundations under eccentric and inclined
loads. Proceedings of the III International Conference on Soil Mechanics Found.
Zürich, Switzerland, 1:440-445.

Terzaghi, K. 1934. Large retaining wall tests. Engineering News Record, 1 February,
8 March, 19 April.

Terzaghi, K. 1943. Theoretical soil mechanics. New York: Wiley.

* Page, figure, table and chapter numbers may differ on updated and revised versions of the prescribed
book. See online version of study guide for updates.

322 GEE2601/1
Study unit 1 12

RETAINING WALLS

12.1 INTRODUCTION

We discussed lateral earth pressure in study unit 9 and we derived the theories of
Rankine and Coulomb for calculating active and passive lateral pressures. Study unit
11 dealt with the bearing capacity of shallow foundations subjected to eccentric and
inclined loads. The knowledge you gained from studying those two study units will be
combined in this study unit to investigate the stability of retaining walls. Many other
types of structures are subjected to lateral pressures, for example underground
conduits, culverts and bracing structures for excavations, but their deflections are
limited by their stiffness, geometry, symmetry of loading, or connections to other
structures. As a result, active or passive conditions cannot develop in the soil adjacent
to these structures and lateral pressures have to be determined in a different way.
Analyses of these types of structures are not included in this course.

12.2 DEFINITION

You will come across the following term in this study unit:

• Retaining wall: A relatively rigid wall that is used for supporting soil masses
laterally so that the soil can be retained at different levels on the two sides.
Furthermore, it is a structure designed to restrain soil to a slope that it would not
naturally keep to (typically a steep, near-vertical or vertical slope).

323 GEE2601/1
12.3 LEARNING OUTCOMES
On completion of this study unit, you should be able to investigate the stability of
gravity retaining walls and embedded retaining structures. More specifically, you
should be able to
• explain and apply the fundamental principles of retaining walls and stability
analysis using Rankine’s theory and Coulomb's law
• distinguish between the different possible failure modes of these structures
• determine the factors of safety against failure

To achieve the learning outcomes in this unit, work through the following topics:
• modes of failure of gravity and cantilever walls
• overturning failure of gravity and cantilever walls
• sliding failure of gravity and cantilever walls
• bearing capacity failure of gravity and cantilever walls
• modes of failure of failure of sheet pile walls
• overturning and horizontal stability of cantilevered sheet pile walls
• overturning and horizontal stability of anchored sheet pile walls

12.4 TYPES OF RETAINING WALLS


Chapter 15 (pp. 490-567*) in the prescribed book covers the contents of this study
unit. Study sections 15.1 and 15.2 (pp. 490-493*) in the prescribed book to learn
about the different types of retaining walls. Note that in this chapter, the design does
not cover the structural elements (reinforcement) of the body of the retaining wall.
Vertical or near-vertical slopes of soil are supported by retaining walls, cantilever sheet
pile walls, sheet pile bulkheads, braced cuts and other similar structures. The proper
design of retaining wall structures requires an estimation of lateral earth pressure,
which is a function of several factors, such as

• type and amount of wall movement


• shear strength parameters of the soil
• unit weight of the soil

324 GEE2601/1
The following figure shows a retaining wall of height H, as shown in previous units, for
similar types of backfill:

FIGURE 12.1
A retaining wall of height H, for similar types of backfill (Al-Agha 2015)

As shown in the figure above, there are three types of lateral earth pressure:

• At-rest lateral earth pressure: The wall may be restrained from moving. For
example, a basement wall is restrained from moving due to the slab of the
basement and the lateral earth force in this case can be termed Po.
• Active lateral earth pressure: If the wall is free from its upper edge (retaining
wall), the wall may move away from the soil that is retained with distance "+ΔH”
(i.e. the soil pushes the wall away). This means the soil is active and the force of
this pushing is called active force and termed Pa.

325 GEE2601/1
• Passive lateral earth pressure: For the wall shown above (retaining wall) on the
left side there is soil with a height less than the soil on the right and, as we
mentioned above, the soil on the right will push the wall away, so the wall will be
pushed into the soil on the left (i.e. soil compresses the soil on the left). This means
the soil has a passive effect and the force in this case is called passive force and
termed PP.

Figures 12.2 to 12.5 show the different types of retaining walls.

FIGURE 12.2 FIGURE 12.3


Cantilever wall (Shutterstock 2018) Gravity wall (Shutterstock 2018)

326 GEE2601/1
FIGURE 12.4 FIGURE 12.5
Counterfort wall (Shutterstock 2018) Cantilever L wall (Shutterstock 2018)

12.5 APPLICATION OF LATERAL EARTH PRESSURE THEORIES


TO RETAINING WALLS

Study section 15.4 (pp. 494-496*) in the prescribed book. To apply the somewhat
idealised lateral earth pressure theories by Rankine and Coulomb to real walls, some
simplifying geometry assumptions are needed, as shown in figure 15.4(a), (b) and (c)
in the prescribed book (pp. 494-495*).

Rankine's theory can only be used when the wall-to-soil interface is vertical. This
condition is complied with by applying the lateral pressure to a vertical plane through
the heel of the wall, as if this plane were the interface. The volume of soil between this
vertical plane and the wall is considered to be part of the wall for purposes of stability
analyses. Note that the effective height of the wall is increased by this simplification
when the soil surface is inclined.

One advantage of Coulomb's theory is that it can be applied when the soil-to-wall
interface is not vertical, but the interface must be a single plane. In the case of gravity
walls this is achieved by ignoring any protrusions from the wall, like the foundation
slab in figure 15.3(c) (pp. 494-495*) in the prescribed book, when Pa is calculated.

327 GEE2601/1
12.6 MODES OF FAILURE OF A RETAINING WALL

Failure may be defined as any event that renders the retaining wall, or an element of
the wall, unsuitable for the purpose for which it was designed. Several modes of failure
have been identified in the case of gravity and cantilever walls:

• overturning of the wall


• sliding on its base
• bearing capacity failure
• development of a deep-seated failure surface below the wall
• excessive deformation in any direction (rotational, vertical, horizontal)
• erosion of the retained soil as a result of an inadequate drainage system
• structural failure of an element of the wall (bending or shear failure)

We will discuss overturning of the wall, sliding on its base and erosion of the retained
soil as a result of an inadequate drainage system later in this study unit. We discussed
bearing capacity failure in study unit 11. Development of a deep-seated failure surface
below the wall can be analysed by means of slope stability theory as we discussed in
study unit 10, and we covered excessive deformation in any direction in study units 6
and 11. Structural failure of an element of a wall can be designed against by structural
engineers using input from this unit.

12.7 STABILITY ANALYSIS

Stability analysis is an important step in the design process of every retaining wall
structure. To successfully design a retaining wall that is stable, it is important that you
analyse all the forces (lateral, vertical and imposed loads) that act on it (retaining wall)
so as to size your wall to be able to withstand these forces. In the sections that follow,
you will learn more about the typical forces that act on a retaining wall and how to
check for stability by checking your retaining wall for overturning, sliding and bearing
capacity failure. You will cover the following:

• forces that act on a retaining wall

328 GEE2601/1
• how to check for overturning
• how to check for sliding along the base
• how to check for bearing capacity failure

12.7.1 Forces acting on a retaining wall

Figure 12.6 below shows a cantilever retaining wall. The following forces act on the
wall:

• Active lateral soil force Pa calculated by Rankine's method and acting on the
vertical plane through the heel of the wall. The force is resolved into its horizontal
and vertical components, Ph and Pv. Since Rankine assumed that Pa is parallel to
the soil surface, the components are
Ph = Pa cos α
Pv = Pa sin α
• Passive lateral soil force Pp. This force is usually ignored in stability analyses
because it may be reduced, or even destroyed, by removal of soil from the area in
front of the toe.
• The weight Ws of the soil between the vertical plane through the heel and the wall.
• The weight Wb of the wall foundation.
• The weight Ww of the wall stem.
• The base reaction R on the underside of the foundation, resolved into a horizontal
component Rh, and a vertical component Rv. The values of two components and
their positions can be calculated from equilibrium considerations.

329 GEE2601/1
FIGURE 12.6
Stability analysis of retaining wall

12.7.2 Check for overturning

Study section 15.5 (pp. 496-498*) in the prescribed book. The value of the vertical
component, Rv, of the foundation reaction can be calculated from vertical equilibrium
considerations. In the case of the wall in figure 12.1 the result is

• Rv = Pv + Ws + Ww + Wb (12.1)

From moment equilibrium of the wall, taking moments about A and omitting Pp

Σ(overturning moments) = Σ(resisting moments)

• Rvx + PhH'/3 = PvB + Wsxx +Wwxw + Wbxb (12.2)

330 GEE2601/1
The position x of the reaction Rv is determined by solving equations 12.1 and 12.2. If
the combination of moments is unfavourable, for instance when Ph is very high, and
the margin of safety against overturning is low, the reaction Rv will be positioned close
to A, or x will be small. When the wall actually falls over, it will rotate about A and x
becomes zero. The safety factor against overturning may then be defined as the ratio
between the total resisting moment and the total overturning moment, with Rvx = 0.

Pv B + W s x s + W w x w + Wb x b
FS (overturning) =
PhH' / 3 (12.3)

12.7.3 Check for sliding along the base

Read through section 15.6 (pp. 498-500*) in the prescribed book. From horizontal
equilibrium of the wall in figure 12.6, the horizontal component Rh of the foundation
reaction must be equal to the horizontal component Ph of the active lateral soil force.
Pp is again not included.

The shear strength of the soil in contact with the foundation must be sufficient to resist
Rh, otherwise the wall will slide on its base. The shear strength of the soil is τ

• τ = σ' tanφ' + c'


If the shear strength is multiplied by the area of the foundation, which is B m2 per m, it
gives the available shear force T:

• T = σ'B tanφ' + c'B = (ΣV) tanφ' + c'B


where ΣV is the sum of all the vertical forces = Pv + Ws + Ww + Wb = Rv

The factor of safety against sliding is

• FS (sliding ) =
(ΣV ) tan φ'+Bc' = R v tan φ'+Bc' (12.4)
Ph Ph

331 GEE2601/1
The expression above assumes that the full shear strength of the soil in contact with
the foundation is available to resist sliding at the interface since concrete is poured
into the foundation trench and the bottom of the foundation should be rough. If in doubt,
use reduced values of the shear strength parameters.

12.7.4 Check for bearing capacity failure


Read through section 15.7 (pp. 500-502*) in the prescribed book. From equilibrium
considerations it was determined that the foundation reaction is eccentric as well as
inclined. The inclination of the foundation load can be determined from the values of
the components Rv and Rh of the base reaction.

Both the eccentricity and inclination of the foundation load will reduce the ultimate
bearing capacity of the foundation, as was discussed in chapter 11. If a water table is
within a distance B below the foundation, its presence will have to be taken into
account too.

The eccentricity is determined from the value of x obtained from equation 12.2.

• e = (B/2) – x (12.5)

The value of e may be positive or negative, depending on the magnitudes of the


applied moments. With a wide foundation, the relatively large weight Ws and lever arm
xs of the soil resting on the foundation slab may result in a negative eccentricity, which
means that Rv is positioned on the side of the centreline away from A. But the moment
by Ph may be sufficiently large to move Rv over to the other side of the centreline, and
e would be positive.

Example 12.1
This example is identical to example 15.1 (pp. 502-503*) in the prescribed book, but
some modifications are made to the calculations. The most important modification will
be to ignore the contribution of the soil above foundation level at the toe in the
calculations of the factors of safety. This soil may be loose and of low shear strength
and cannot be relied on.

332 GEE2601/1
Solution

Rankine's method is used to calculate the active force on the vertical through the heel
of the wall, and the force is parallel to the soil surface:

• Pa = 161.40 kN/m
• Pv = Pa sin 10° = 28.03 kN/m
• Ph = Pa cos 10° = 158.95 kN/m

The passive pressure acting on the toe is ignored.


From the calculations in the prescribed book we have the following values:
• Total vertical force = ΣV = 470.4 kN/m
Total resisting moment about C from vertical forces = ΣMR = 1129.0 kNm/m
Overturning moment about C from Ph = 379.3 kNm/m = MO

Check for overturning:


ΣMR 1129,0
• FS(overturning) = = = 2,98
ΣM O 379,3

Factor of safety against sliding:


Sliding is resisted by forces of friction and cohesion along the base. Assume that the
interface friction angle is ⅔φ' = 17° and that interface cohesion is ⅔c' = 27 kN/m2. The
available resisting force is

• ΣV tan 17° + B × 27 = 470,4 × tan 17° + 4 × 27 = 251,8 kN/m

The force that promotes sliding is


Ph = 158,9 kN/m
251,8
• FS(sliding) = = 1,58
158,9

333 GEE2601/1
The factor of safety against sliding is low, but the passive force on the foundation slab
provides an additional margin of safety. A key below the foundation will further improve
the factor of safety.

Bearing capacity failure

The position of the vertical reaction force is determined from equation 12.2:
ΣM R − ΣM O 1129,0 − 379,3
• x= = = 1,594m
ΣV 470,4
• e = B/2 – x = 4/2 – 1,594 = 0.406 m
• B' = B – 2e = 3.188 m

Ultimate bearing capacity is calculated without taking shear strength of the soil above
foundation level into account (which is different from the method followed in the
prescribed book). Bearing capacity factors are obtained from table 16.1 (p. 573*) in
the prescribed book. The inclination of the load is β = tan-1 (Ph/ΣV) and the inclination
factors are determined from the expressions in table 15.2 in the prescribed book.

qult = cNcFci + qNqFqi + 0,5γB'NγFγi


= 40 × 14.83 × 0.628 + (1.5 × 19) × 6.4 × 0.628 + 0.5 × 19 × 3.188 × 5.39 × 0
= 574.1 kN/m2

Calculation of the factor of safety against bearing capacity failure is based on the
equivalent width. (Note that net values are not used in the prescribed book and
calculations are based on qmax.) The merits of the different approaches were discussed
in study unit 11.

• qnet = (ΣV)/B' – γD = 470,4 / 3,188 – 19 x 1,5 = 119,0 kN/m2


• qnet(ult) = 574,1 – 19 x 1,9 = 545,6 kN/m2
545,6
• FS(bearing capacity) = = 4,58
119,0

334 GEE2601/1
Example 12.2

Consider example 15.2 (pp. 506-508*) in the prescribed book. The calculations are
presented in the prescribed book and will only be discussed here briefly.

Solution

The interface between the retaining wall and the soil is sloped at 15°, which calls for
Coulomb's method of lateral earth pressure calculation. The protruding foundation slab
is ignored in the calculation of Pa. Pa acts at an angle of δ = 2/3φ' above the normal to
the interface, and the normal is inclined at 15° to the horizontal as a result of the
inclined interface. The sum of these angles must be used to calculate the vertical and
horizontal components of Pa.

Note:

• In the determination of the factor of safety against sliding, the foundation interface
shear resistance is taken as 2/3 of the shear strength parameters of the foundation
soil.
• It is again recommended that the passive resistance be omitted from the
calculations, unless the shear strength of the soil is verified and it can be
guaranteed that the soil will stay intact for the entire life of the wall.

12.8 DESIGNING FOR UNDRAINED CONDITIONS

The backfill behind retaining walls is usually granular, in which case only drained
conditions have to be considered. The discussions and derivations above assumed
effective stress conditions throughout. Occasionally a gravity or cantilever retaining
wall has to be constructed on saturated clay, which means that undrained conditions
below the foundations have to be designed for in the short term.

The granular fill will always be drained and the value of Pa based on effective stresses.
But in the clay below the foundation total stresses should be used in the short term.

335 GEE2601/1
The ultimate bearing capacity will be in terms of total stresses and so will the
calculation of F(sliding). For the saturated clay φu = 0, so that

(ΣV ) tan φ u + Bc u Bc u
• FS (sliding )(undrained ) = = (12.6)
Ph Ph

12.9 STABILITY OF EMBEDDED RETAINING WALLS

Embedded retaining walls differ from cantilever and gravity retaining walls in that the
stability of the structure depends entirely on the passive lateral pressure of the soil, or
some external means of support. The walls are usually steel sheet piling embedded
below the surface by driving or vibration. After installation of the sheet pile wall, the
levels of the soil on its two sides are changed by backfilling, or excavation, or both.
The wall deflects away from the higher (“backfilled”) side towards the lower
(“excavated”) side, allowing active and passive limit states to develop in the soil.
Depending on the flexibility of the piling, bending deflection adds to the development
of the limit states.

Several modes of failure have been identified in the case of embedded walls:
• overturning of the wall, if not anchored
• instability of the embedded part due to insufficient length
• overstressing of the anchor, if present
• development of a deep-seated failure surface below wall
• excessive deformation in any direction (rotational, vertical, horizontal)
• quick conditions which may develop as a result of high hydraulic gradients
• structural failure of the wall usually in bending

In most cases the soil surfaces are level and the walls vertical, so Rankine's method
is ideal for determining lateral soil pressures. The discussion will be limited to granular
in situ soil and backfill and only static pore water pressures will be included in the
analyses.

336 GEE2601/1
12.9.1 Cantilever sheet pile walls

If the wall is relatively low, it can be supported by the passive resistance of the soil.
The initial geometry and deflected position of the wall are shown in figure 12.7(a), and
the lateral soil pressure diagrams are shown in figure 12.7(b) below.

FIGURE 12.7
(a) Cantilever sheet pile geometry and deflected position,
(b) Lateral earth and hydrostatic pore water pressure diagrams (not to scale)

The sheet pile wall rotates at point C, which is close to the bottom end of the wall. The
actual position of C is determined by assuming that CD = 20% of BC. Above C the
wall experiences active lateral pressure on the “backfilled” side and passive lateral
pressure on the “excavated” side, but below C the pressure on the “backfilled” side is
passive and on the “excavated” side it is active. The reversal of pressures is as a result
of the rotation about C. The wall is kept in moment equilibrium by the fixing moment
from the opposing passive pressures above and below C. Horizontal equilibrium must
also be satisfied.

The geotechnical engineer and technician must evaluate the factor of safety with
regard to passive pressure for a given penetration depth. The procedure is best
explained by means of an example.

337 GEE2601/1
Example 12.3

A sheet pile is driven into a granular soil to a depth of 3.6 m and then backfilled with
the same soil to a height of 2.5 m. The water table is at 1.0 m below the original ground
level. The soil has the following properties: γ = 17 kN/m3 above the water table, γsat =
20 kN/m3 below the water table, c' = 0 and φ' = 30°. Determine the factor of safety on
the passive forces.

Solution

Figure 12.8
Cantilever sheet pile geometry and provisional lateral earth
pressure diagrams (not to scale) for example 12.3

The geometry and assumed shapes of the lateral pressure diagrams are shown in
figure 12.8. Since it is assumed that the distance from C, which is the point of rotation,
to the tip is 20% of the penetration depth, C must be at 0.2 × 3.6/1.2 = 0.6 m above
the tip, or 3.0 m below the original ground surface.

Calculate the values of Ka and Kp:


• Ka = tan2 (45° – φ'/2) = tan2 (45° – 35°/2) = 0.271
• Kp = tan2 (45° + φ'/2) = tan2 (45° + 35°/2) = 3.690

338 GEE2601/1
Calculations of the lateral pressures above C are shown in tables 12.1 and 12.2 below:

TABLE 12.1: Backfilled side calculations

Lateral
Depth from Vertical pressure σ v'
Ka/Kp pressure σa'
surface (m) (kN/m2)
(kN/m2)
0.0 0 0,271 0
3,5 3,5 × 17 = 59,5 0,271 16,12
5,5 3,5 × 17 + (20 – 10) × 2,0 = 79,5 0,271 21,54
5,5 79,5 3,690 293,35
6,1 3,5 × 17 + (20 – 10) × (2,6) = 85,5 3,690 315,50

TABLE 12.2: Excavated side calculations

Depth from Vertical pressure σ v' Lateral pressure σa'


Ka/Kp
surface (m) (kN/m2) (kN/m2)
0 0 3,69 0
1,0 1,0 × 17 = 17 3,69 62,73
3,0 1,0 × 17 + (20 – 10) × 2,0 = 37 3,69 136,53
3,0 37 0,271 10,03
6,1 3,5 × 17 + (20 – 10) × (2,6) = 85,5 3,690 315,50

Figure 12.9 shows the lateral pressures on the wall, and the forces associated with
each area of the pressure diagrams are indicated as well. Note the hydrostatic
pressures on the two sides.

339 GEE2601/1
Figure 12.9
Lateral earth pressure diagrams (not to scale) and
resultant lateral forces for example 12.3

The value of SF is determined from moment equilibrium. The obvious point about
which to take moments is C, and to simplify the analysis somewhat, it is assumed that
the forces 7 and 8 below C actually act at C, and not at approximately 0,3 m below C.
As a result, these forces do not contribute to the moment. The error involved is small.

Calculation of forces and moments is shown in the table below. A positive force is from
right to left and a negative force from left to right. Passive forces are not fully mobilised
and include a factor of safety.

TABLE 12.3: Forces and moments

Force Moment
Value (kN/m) Lever arm (m)
no. (kN.m/m)
1 0,5 × 16,12 × 3,5 = 28,21 3,5/3 + 2 = 3,167 89,33
2 16,12 × 2,0 = 32,24 2,0/2 = 1,0 32,24
3 0,5 × (21,54 – 16,12) × 2,0 = 5,42 2,0/3 3,61
4 - 0,5 × 62,73 × 1,0 = - 31,36/FS 1,0/3 + 2,0 -73,17/FS
5 - 62,73 × 2,0 = -125,46/FS 2,0/2 = 1,0 -125,46/FS
6 - 0,5 × (136,5 – 62,73) × 2,0 = -73,77/FS 2,0/3 -49,18/FS

340 GEE2601/1
The factor of safety against overturning is determined from moment equilibrium:

• 89,33 + 32,24 + 3,61 – 73,17/FS – 125,46/FS – 49,18/FS = 0


ΣMRe sisting 73,17 + 125,46 + 49,18
• FS = = = 1,98
ΣMOverturning 89,33 + 32,24 + 3,61

The next step is to verify horizontal equilibrium. The sum of the forces below C must
be equal to or greater than the sum of the forces above C. If this is not the case,
penetration should be increased.

Table 12.4: Horizontal equilibrium

Force no. Value (kN/m)


1 28,21
2 32,24
3 5,42
4 - 31,36/1,98 = -15,84
5 - 125,96/1,98 = - 63,62
6 - 73,77/1,98 = - 37,26
7 182,26/1,98 = 92,05
8 -6,5

• Sum of forces 1 to 6 above C = -50,85 kN/m (left to right)


• Sum of forces 7 and 8 below C = 85,55 kN/m (right to left)

Sufficient force can be generated by the passive pressure below C to balance forces
above C. This satisfies the criterion for horizontal stability.

12.9.2 Anchored sheet pile walls

When sheet pile walls are high, the bending moments induced by the active pressures
in cantilevered piles become excessive and very heavy sheet pile sections have to be
used. High cantilevered walls, or low shear strength of the soil, may also result in large
penetration depths to satisfy equilibrium with an adequate factor of safety. Both cases
may result in uneconomical designs. If some form of lateral support is supplied near
the top of the wall, the situation will be greatly improved. Bending moments are

341 GEE2601/1
reduced and penetration depths decreased. Support may be provided by one or two
rows of equally spaced cables held in place by anchors in the soil some distance away
or external support may be provided by rows of struts.

The mode of deflection of an anchored sheet pile wall is different from that of a
cantilevered wall. Rotation is about the anchor level, and it is assumed that lateral
deformation is sufficient for active lateral pressure to develop over the entire
“backfilled” side and passive pressure over the entire “excavated” side. Figure 12.10
shows the geometry and the assumed lateral pressure distribution of an anchored
sheet pile wall with hydrostatic pore water pressures on both sides.

Figure 12.10
(a) Anchored sheet pile geometry and deflected position
(b) Lateral earth and hydrostatic pore water pressure diagrams (not to scale)

The factor of safety on the passive lateral pressure is determined from moment
equilibrium about the anchor level. The anchor force per running metre is determined
from horizontal equilibrium once the factor of safety is known.

342 GEE2601/1
Example 12.4

An anchored sheet pile wall is 13.5 m long. The soil surfaces on opposing sides differ
by 7.5 m. A row of anchors is attached to the wall at 1.5 m from the top. On the
backfilled side the water table is at 4.5 m below the surface and on the excavated side
it is at 3 m below the surface. Assume static pore water pressure below the water
tables. Above the water tables the unit weight of the soil is 18 kN/m3 and below the
water table the saturated unit weight is 20 kN/m3. Lateral soil pressure coefficients are
Ka = 0.333 and Kp = 3.00. Determine the factor of safety on the passive forces as well
as the required anchor force per running metre of the wall.

Solution

FIGURE 12.11
Geometry and provisional lateral earth and hydrostatic
pore water pressure for example 12.4

The geometry and provisional lateral pressure diagrams are shown in figure 12.11.
The values of the active and passive pressures at certain depths will now be
calculated.

343 GEE2601/1
TABLE 12.5: Backfilled side calculations

Depth from Vertical pressure σ v' Lateral pressure σa'


Ka
surface (m) (kN/m2) (kN/m2)
0 0 0.333 0
4,5 4,5 × 18 = 81 0.333 27
13,5 4,5 × 18 + (20 – 10) × 9 = 171 0.333 57
0 0 0.333 0

TABLE 12.6: Excavated side calculations

Depth from Vertical pressure σ v' Lateral pressure σa'


Ka
surface (m) (kN/m2) (kN/m2)
0 0 3,00 0
3,0 3,0 × 18 = 54 3,00 162
6,0 3,0 × 18 + (20 – 10) × 3,0 = 84 3,00 252

The lateral earth pressures and hydrostatic pore water pressures are shown in figure
12.12. Also shown in the figure are the forces associated with each part of the pressure
diagrams.

FIGURE 12.12
Lateral earth and hydrostatic pore water pressure diagrams
for example 12.4 (not to scale)

344 GEE2601/1
The values of the forces, their lever arms and moments about the anchor point are
calculated in the following table. The available passive forces are not fully mobilised
and must include the factor of safety.

TABLE 12.7: Forces, their lever arms and moments about the anchor point

Force Moment
Value (kN/m) Lever arm (m)
no. (kNm/m)
1 0,5 × 27 × 4,5 = 60,75 2/3 × 4,5 - 1,5 = 1,5 91,1
2 27 × 9,0 = 243 9,0/2 + 3,0 = 7,5 1 882,5
3 0,5 × (84 – 54) × 9,0 = 135 2/3 × 9,0 + 3,0 = 9,0 1 215,0
4 0,5 × 90 × 9,0 = 405 2/3 × 9,0 + 3,0 = 9,0 3 645,0
5 - 0,5 × 162 × 5,0 = -81/FS 2/3 × 3,0 + 6,0 = 8,0 -648,0/FS
6 -162 × 3,0 = -486/FS 0,5 × 3,0 + 9,0 = 10,5 -5 103,0/FS
7 -0,5 × (252 – 162) × 3,0 = -135/FS 2/3 × 3,0 + 9,0 = 11,0 -1 485,0/FS
8 -0,5 × 30 × 3,0 = -45 2/3 × 3,0 + 9,0 = 11,0 -495,0

The factor of safety against rotating about the anchor is calculated from moment
equilibrium:

• 91.1 + 1822.5 + 1215.0 + 3646.0 – 648.0/FS – 5103.0/FS – 1006.5/FS – 495 = 0


648.0+5103.0+1485.0
• 𝐹𝐹𝐹𝐹 = = 1.14
91.1+1882.5+1215.0+3646.0−495.0

The factor of safety is rather small as a result of the huge moment caused by the water
pressure on the backfilled side. Horizontal equilibrium is maintained by the anchor
force T:

• 60.75 + 243 + 135 + 405 – 81/1.14 – 486/1.14 – 135/1.14 – 45 – T = 0


• T = 183.0 kN/m

345 GEE2601/1
12.10 SELF-ASSESSMENT ACTIVITIES

You now have the opportunity to apply what you have learnt in this unit. Answer the
following questions:

Activity 12.1: Prescribed book exercises

1. Do problems 15.1 and 15.2 (p. 559*). Use Rankine's method to calculate active
lateral pressure on the vertical through the heel of the cantilever retaining wall.
Ignore passive pressure in all calculations.
2. Do problems 15.3 and 15.4 (p. 559*). Calculate factors of safety with regard to
overturning, sliding and bearing capacity failure.
3. Do problem 15.5 (p. 559*). Calculate factors of safety in respect of overturning,
sliding and bearing capacity failure and compare the factors of safety with those
obtained in the previous problem.
4. Do problems 15.6, 15.7 and 15.8 (p. 560*).

Activity 12.2: Multiple-choice questions

1. The retaining walls provide a permanent lateral support to …


(a) vertical slopes of soil.
(b) near-vertical slopes of soil.
(c) Both (a) and (b).
(d) slopes of soil.

2. The vertical faces of the cuts made during construction of basements of


buildings in developed areas or underground transportation facilities at shallow
depths below the ground surface should be protected by a …
(a) retaining wall.
(b) temporary bracing system.
(c) raft.
(d) None of the above.

346 GEE2601/1
3. Which of the following walls depends mainly on its own weight for stability?
(a) gravity retaining wall
(b) cantilever retaining wall
(c) counterfort retaining wall
(d) All of the above.

4. Which of the following walls is not economical for supporting high vertical slopes
of soil?
(a) gravity retaining wall
(b) cantilever retaining wall
(c) counterfort retaining wall
(d) Both (b) and (c).

5. Cantilever retaining walls are made of …


(a) plain cement concrete.
(b) reinforced cement concrete.
(c) stone masonry.
(d) brick masonry.

6. Counterfort retaining walls are similar to …


(a) gravity retaining walls.
(b) cantilever retaining walls.
(c) Both (b) and (c).
(d) None of the above.

7. Which of the following walls consists of a thin stem and a base slab?
(a) gravity retaining wall
(b) cantilever retaining wall
(c) counterfort retaining wall
(d) Both (b) and (c).

347 GEE2601/1
8. At regular intervals, the thin vertical concrete slabs called counterforts that tie
the stem and the base slab together in the counterfort retaining wall reduce …
(a) the height of the wall.
(b) shear force.
(c) the bending moment.
(d) Both (b) and (c).

9. The design of retaining walls requires …


(a) unit weight, cohesion and angle of friction of the soil retained behind the
wall.
(b) unit weight, cohesion and angle of friction of the soil below the base of the
wall.
(c) Both (a) and (b).
(d) None of the above.

10. With the lateral earth pressure known, the retaining wall as a whole is checked
for …
(a) overturning about its toe and sliding failure along its base.
(b) bearing capacity failure of the base and its settlement.
(c) overall stability.
(d) All of the above.

11. When designing retaining walls, a trial section with some of the dimensions is
assumed. This initial design step is called …
(a) dimensioning.
(b) proportioning.
(c) sectioning.
(d) designing.

12. The minimum width of the top of the stem of retaining walls is about …
(a) 0.1 m.
(b) 0.3 m.

348 GEE2601/1
(c) 0.6 m.
(d) 1.0 m.

13. The minimum depth of soil in the front of retaining walls is about …
(a) 0.1 m.
(b) 0.3 m.
(c) 0.6 m.
(d) 1.0 m.

14. The base width of retaining walls of height H is generally …


(a) 0.1H to 0.3H.
(b) 0.3H to 0.7H.
(c) 0.5H to 0.7H.
(d) 0.5H to H.

15. The minimum factor of safety with regard to overturning is generally …


(a) 1.
(b) 1.5 to 2.
(c) 2 to 3.
(d) 1.5 to 4.

16. The minimum factor of safety with regard to sliding is generally …


(a) 1.
(b) 1.5.
(c) 3.
(d) 4.

17. Which is the best option to increase the resistance to sliding of the retaining
wall?
(a) Increase the base width of the wall.
(b) Increase the height of the wall.
(c) Construct the base key.
(d) None of the above.

349 GEE2601/1
Activity 12.3: Typical exam question

The cross-section of a cantilever retaining wall is shown below. Calculate the factor of
safety in respect of overturning, sliding and bearing capacity.

12.11 SOLUTIONS TO SELF-ASSESSMENT ACTIVITIES

Activity 12.1: Prescribed book exercises

1. Consult the solution book for answers.


2. Consult the solution book for answers.
3. Consult the solution book for answers.
4. Consult the solution book for answers.

Activity 12.2: Multiple-choice questions

1. (c)
2. (b)

350 GEE2601/1
3. (a)
4. (a)
5. (b) Discussion: (a), (c) and (d) are correct for the gravity retaining walls.
6. (b)
7. (d)
8. (d)
9. (c)
10. (d)
11. (b)
12. (a) Discussion: The top of the stem of any retaining wall should be no less than
about 0.3 m wide for proper placement of concrete.
13. (c)
14. (c)
15. (b)
16. (b) Discussion: (c) is correct for factor of safety against bearing capacity failure.

Activity 12.3: Typical exam questions

Ka = 0.3495; Pa = 161.2 kN; MR = 1 132.88 kN/m; V = 470.11 kN; FSOT = 2.99; FSS =
2.72; e = 0.4m; qmax = 188.04 kN/m2; qmin = 47 kN/m2

12.12 SUMMARY

In this study unit you should have learnt to apply the theories of lateral earth pressure
to real retaining structures. Two types of structures were discussed. Gravity and
cantilever retaining walls utilise their own weight, or that of a volume of soil, to remain
stable against the lateral forces, whereas embedded walls have to rely on the passive
resistance of the soil for stability against active forces and unbalanced pore water
pressures. In the case of gravity and cantilever walls, the stability analyses also
include bearing capacity failure, and the theory of ultimate bearing capacity was
applied.

351 GEE2601/1
12.13 VIDEO CLIPS, SLIDES AND OTHER WEB-BASED
RESOURCES

For additional explanations on retaining walls, you can watch some video clips. Web
addresses/links to the video clips are available on the module site under Additional
Resources in a folder named Video clips, slides and other web-based resources.

• Design of retaining wall:


https://www.youtube.com/watch?v=mUWCnANvJdE

• How to build a retaining wall:


https://www.youtube.com/watch?v=mp5OLKynS3k

• Design of retaining wall (contd.):


https://www.youtube.com/watch?v=6LZiPNApaB8

REFERENCE WORKS AND FURTHER READING

Al-Agha, AS. 2015. Basics of foundation engineering with solved problems. Based on
“Principles of foundation engineering, 7th edition”.

Das, BM. 2010. Principles of geotechnical engineering. 7th ed. Connecticut: Cengage
Learning.

Das, BM. 2013. Fundamentals of geotechnical engineering. 4th ed. Connecticut:


Cengage Learning.

Das, BM & Sivakugan, N. 2017. Fundamentals of geotechnical engineering. 5th ed.,


international edition. Connecticut: Cengage Learning.

Terzaghi, K. 1934. Large retaining wall tests. Engineering News Record, 1 February,
8 March, 19 April.

* Page, figure, table and chapter numbers may differ on updated and revised versions of the prescribed
book. See online version of study guide for updates.

352 GEE2601/1
Study unit 1 13

PILE (DEEP) FOUNDATIONS

13.1 INTRODUCTION

Pile (deep) foundations are used to support major structures when highly
compressible, weak, expansive or collapsible soils are encountered near the surface,
large horizontal or uplifting forces are involved, or scouring is expected. Since you are
likely to become involved in piled foundations, you should acquire some knowledge
about the bearing capacity of piles in sand and clay.

Drilled shafts and caissons are forms of deep foundations too, but because of their
size they can carry much larger forces than piles. Their design is usually done by
experts and is not included in this course. The same applies to laterally loaded piles
and pile groups.

Pile foundations are usually costlier. Despite their cost, their use is often necessary to
ensure structural safety. In this study unit, we will cover the following areas:

• various types of piles and their characteristics


• installations
• load-bearing capacities
• estimation of loads
• pile load testing

353 GEE2601/1
13.2 DEFINITIONS

You will come across the following terms in this study unit:

• Friction piles: A type of pile that is used to transmit the structural load to the soil
gradually. The resistance to the applied load is derived from the frictional
resistance developed at the soil-pile interface.
• Pile foundation: A vertical structural member made of steel, concrete or timber,
driven or drilled deep into the ground to transfer loads from a structure to the soil.
• Point-bearing pile: A type of pile that is used to transmit the structural load to
underlying bedrock or a stronger soil layer.

13.3 LEARNING OUTCOMES

On completion of this study unit, you should be able to explain the bearing capacity of
single piles in sand and clay. More specifically, you should be able to

• explain the interaction of a single pile with the soil to develop a resistance to vertical
loads
• identify and explain the different methods of analysis
• identify various types of piles and their characteristics, installations and pile load
testing
• perform calculations that involve bearing capacity of single piles in sand and in clay
under drained and undrained conditions

The contents of this study unit are covered in detail in chapter 18 (pp. 643-711*) in
the prescribed book. To achieve the learning outcomes, study chapter 18 in the
prescribed book.

354 GEE2601/1
13.4 TYPES OF PILES, PILE CHARACTERISTICS, PILE
INSTALLATION, LOAD TRANSFER MECHANISM

Pile foundations are generally needed in special circumstances. The following are
some situations in which piles may be considered:

• when the upper soil is weak and highly compressible to support a load
• when there are horizontal forces
• when the site has expansible and collapsible soils
• when there are uplifting forces, especially on foundations which are below the
water table
• when there is soil erosion

Figure 18.1 (p. 644*) in the prescribed book best illustrates the conditions for use of
pile foundations. Sections 18.1 to 18.3 (pp. 643-653*) in the prescribed book deal
with the need for pile foundations as well as the types of piles and their structural
characteristics. The types of piles that are common in practice as well as the
construction industry are

• steel piles
• concrete piles
• timber piles
• composite piles (different types of materials)

Figures 13.1 to 13.4 below show the different types of piles.

355 GEE2601/1
FIGURE 13.1 FIGURE 13.2
Steel piles (Shutterstock 2017) Concrete piles (Shutterstock 2017)

FIGURE 13.3 FIGURE 13.4


Timber piles (Shutterstock 2017) Composite piles (different materials)

13.5 ESTIMATION OF PILE CAPACITY

Sections 18.7 to 18.8 (pp. 659-662*) in the prescribed book deal with the estimation
of pile capacity. As indicated by equation 18.8 (p. 659*) in the prescribed book, the
ultimate bearing capacity of a pile is the sum of the point-bearing capacity and the
shaft resistance. The latter is commonly known as the shaft friction, although in clay it
will actually be adhesion. The remainder of this section will be on methods of
estimating the magnitudes of point-bearing capacity and skin friction in sand and clay.

356 GEE2601/1
13.5.1 Load-carrying capacity of the pile point

The ultimate bearing capacity of the pile point can be calculated from an equation
similar to the one used for ultimate bearing capacity of shallow foundations subjected
to vertical loads.

qu = cNc* + qN *q + γDN*γ
(13.1)

Where the bearing capacity factors include the necessary shape and depth factors.

It is assumed that the weight of pile is equal to the weight of the soil it displaces, so
calculations are not performed in terms of net bearing capacity. Since the width of a
pile is small, the last term in equation 13.1 may be omitted from the calculations. If the
pile point has a cross-sectional area Ap, the load-carrying capacity is

(
Q p = A p qp = A p cNc* + qN *q ) (13.2)

Where:

• Ap = Area of the pile tip


• c = Cohesion of the soil supporting the pile tip
• qp = Unit point resistance or ultimate bearing capacity of pile point
• q = Effective vertical stress at the level of the pile tip
• Nc*, Nq* = Bearing capacity factors for the piles

13.5.1.1 Pile point in sand

If the pile is in cohesionless sand, equation 13.2 is modified to

Q p = A p qp = A p q' N *q (13.3)

Overburden q is replaced by q’ because calculations must be performed in terms of


effective stress. Figure 18.12 (p. 661*) in the prescribed book shows the relationship
between the bearing capacity factor N*q and the angle of internal friction.

357 GEE2601/1
Meyerhof (1976) found that arching effects in the sand cause overburden pressure q’
in the vicinity of the pile tip to become constant beyond a certain depth. To account for
this, the value of qp = q’Nq* in equation 13.3 should be limited to

qp < 50N*q tan φ' kN/m2 (13.4)

Meyerhof (1976) also proposed an empirical relationship between qp and the corrected
standard penetration resistance in the vicinity of the pile point. Equation 13.5 below
shows that qp is again limited to a maximum of 400Ncor for large depths.

L
qp = 40Ncor < 400Ncor
D kN/m2 (13.5)

If the pile is driven into non-plastic silts, the upper limit should be taken as 300Ncor.

13.5.1.2 Pile point in saturated clay

For piles in saturated clay, in undrained conditions, equation 13.2 simplifies to

Q p = Nc* c u A p = 9c u A p
(13.6)

Where:

cu = Undrained cohesion of the soil below the pile tip

13.5.2 Shaft resistance

Shaft resistance is the product of the friction or adhesion between the pile shaft and
the perimeter area of the shaft. Since soil properties may vary with depth, for example
when the soil is layered, the expression for shaft resistance is

Q s = Σ(p∆Lf ) (13.7)

Where

• p = Perimeter = πD
• ∆L = Increment of pile length over which f is taken constant
• f = Friction or adhesion over the length ∆L

358 GEE2601/1
13.5.2.1 Shaft resistance in sand

In sand the downward movement of the pile shaft is resisted by friction between the
pile shaft and the sand with which it is in contact. Since effective stresses are always
used in sand, the value of f in equation 13.7 is given by

f = Kσ 0 ' tan δ (13.8)

Where

• K = Coefficient of lateral earth pressure


• σ0’ = Average effective vertical stress over the length ∆L under consideration
• δ = Soil-pile interface friction angle

Recommended values of K and δ are given in the prescribed book. You should note
that many other values of K and δ have been proposed by different people, for example
δ = 20° for steel piles and 0.75φ’ for concrete piles, K = 1,0 for loose sand and 2,0 for
dense sand, etc.

As was the case with the point-bearing capacity, arching causes the vertical pressure
around the pile to increase up to a maximum value and it is assumed that σ0’ does not
increase after a depth of about 15D where D is the pile diameter. As a result, the skin
friction f too does not increase beyond this depth.

The results of standard penetration tests may also be used to estimate the average
skin friction in sand, as indicated by equations 18.18 to 18.21 (pp. 662-663*) in the
prescribed book.

13.5.2.2 Shaft resistance in clay

The method of analysis depends on the drainage condition in the clay. If undrained
conditions prevail, the α-method is used. In this method the value of f in equation 13.7
is given by

f = αcu (13.9)

359 GEE2601/1
Where:

• α = Adhesion factor
• cu = Average undrained cohesion over the length ∆L

The value of factor α is a function of the type of clay, pile material and method of
installation of the pile and is assumed to vary with cu as shown in figure 18.15 (p.
664*) in the prescribed book. Slightly different relationships between α and cu have
been proposed by others. The α-method is difficult to apply in practice since there is
considerable scatter in the plot of cu with depth.

From measurements it was found that the pore water pressure usually dissipates very
quickly after installation of the pile in clay, and that the analysis of shaft resistance may
be performed in terms of effective stresses, similar to the analysis in sand. The β-
method allows for this type of analysis, and the value of f in equation 13.7 is calculated
from

f = βσ0’ = K tan φR’σ0’ (13.10)

Where:

• φR’ = Effective remoulded angle of internal friction of the clay around the pile
• K = K0 = Coefficient of lateral earth pressure at rest for the remoulded clay

The value of K0 for normally consolidated and overconsolidated clays can be


calculated from equations 18.34 and 18.35 (p. 667*) in the prescribed book. Note that
the remoulded shear strength parameter φR’ is used because installation of the pile by
drilling or driving completely remoulds the clay with which it is in contact. Since there
is no arching in clay, there is no limit on the value of σ0’.

13.5.3 Allowable pile capacity

The ultimate load-carrying capacity of a pile is the sum of the point load-carrying
capacity and the shaft resistance.

Qu = Qp + Qs (13.11)

360 GEE2601/1
The allowable load-carrying capacity is

Qu
Q all =
FS (13.12)

Where:

• Qall = Allowable load-carrying capacity for each pile


• FS = Factor of safety

Did you know?

• Micropiles, also called mini piles, are often used for underpinning.
• Sheet piling is a form of driven piling using thin interlocking sheets of steel to obtain
a continuous barrier in the ground.
• Soldier piles or Berlin walls are constructed of wide steel H-sections spaced
about 2 m to 3 m apart and are usually driven prior to excavation. As the excavation
continues, horizontal timber sheeting is inserted behind the H pile flanges.

Example 13.1

A 15 m concrete pile with diameter 250 mm is installed in saturated clay. The


undrained shear strength of the clay can be taken as cu = 50 kN/m2 over the first 5 m,
cu = 75 kN/m2 from 5 m to 10 m and cu = 100 kN/m2 below 5 m. If a factor of safety of
2.5 is specified, determine the allowable bearing capacity of the pile.

Solution

• Calculate pile point area: Ap = πD2/4 = π × (0.25)2/4 = 0.049 m2


• Calculate pile perimeter: p = πD = π × 0.25 = 0.785 m

Determine values of α:
• cu = 50 α = 0,88
• cu = 75 α = 0,63
• cu = 100 α = 0,50

361 GEE2601/1
Calculate pile point resistance from equation 13.6:
• Qp = 9cuAp = 9 ×100 × 0,049 = 44 kN

Calculate pile shaft resistance for the three sections from equations 13.9 and 13.7:
Qs = Σ p∆Lαcu
= 0.785 × 5 × 0.88 × 50 + 0.785 × 5 × 0.63 × 75 + 0.785 × 5 × 0.50 × 100
= 173 + 185 + 196 = 554 kN

• Ultimate load capacity: Qu = Qp + Qs = 44 + 554 = 598 kN


• Allowable load capacity: Qall = Qu/FS = 598 / 2.5 = 239kN

Example 13.2

A 15 m concrete pile in saturated normally consolidated clay has a diameter of 250


mm. Near the tip of the pile the undrained shear strength of the clay is 88 kN/m2. A
water table is at 5.0 m. Above the water table the bulk unit weight of the clay is 16
kN/m3 and the saturated unit weight is 21 kN/m3. The average remoulded shear
strength of the clay is φR’ = 21°. Use the β method to calculate the ultimate pile
capacity.

Solution

Calculate pile properties: Ap = 0.049 m2 and p = 0,785 m as in example 13.1.


Calculate end capacity: Qp = 9cuAp = 9 × 88 × 0.049 = 39 kN
Calculate shaft resistance:
f = (1 - sin φR’) tan φR’ σ0’
= (1 – sin 21°) × tan 21° × σ0’
= 0.2463 σ0’

From surface to WT at 5,0 m: average σ0’ = 2.5 × 16 = 40 kN/m2


f = 0.2463 × 40 = 9.85 kN/m2
Qs = f p DL = 9.85 × 0.785 × 5.0 = 39 kN

362 GEE2601/1
From WT to tip: average σ0’ = 5 × 16 + 5 × (21 -10) = 135 kN/m2
f = 0.2463 × 135 = 33.23 kN/m2
Qs = 33.23 × 0.785 × 10.0 = 261 kN
Qs = 39 + 261 = 300 kN

Ultimate pile capacity:


Qu = Qp + Qs = 39 + 300 + 339 kN

Example 13.3

A pile in sand is 18 m long and its diameter is 300 mm. A water table is at 3.0 m. Above
the water table the sand has a dry density of 16 kN/m3 and the saturated density is 19
kN/m3. The average value of φ’ is 35°. Determine the allowable pile capacity if the
factor of safety is 2.8.

Solution

Calculate pile properties:


• Ap = π/4 x 0,3002 = 0,0707 m2
• p = π x 0,300 = 0,9425 m

Calculate point capacity:


From figure 18.12 (p. 661*) in the prescribed book Nq* = 140 for φ’ = 35°
• q’ = 16 × 3 + (19 – 10) × 15 = 183 kN/m2

From equation 13.3


Qp = Ap qp = Ap q’ Nq* = 0,0707 × 183 × 140 = 1 811 kN

But equation 13.4 limits qp to 50 Nq* tan φ’


Therefore, Qp = 0.0707 x 50 x 140 x tan 350 = 346 kN

363 GEE2601/1
Calculate shaft capacity:
• Assume δ = 0.7 φ’ = 0.7 x 350 = 24.50
• Assume K = 1.2 (1 – sin φ’) = 1.2 x (1 – sin 350) = 0.512

Vertical stress is limited to its value at 15 D = 15 x 0,300 = 4.5 m


• σ’lim = 3 × 16 + 1.5 × (19 – 10) = 61.5 kN/m2

From surface to WT at 3.0 m: average σ’ = 1.5 × 16 = 24.0 kN/m2


From equation 13.8
f = K σ’ tan δ
= 0,512 × 24.0 × tan 24.5°
= 5.6 kN/m2
Qs = p f DL
= 0.9425 × 5.6 × 3.0
= 16 kN

From WT at 3.0 m to 15D at 4.5 m:

Average σ’ = 3.0 × 16 + 0.75 × 9

= 54.75 kN/m2

From equation 13.8:


f = K σ’ tan δ
= 0.512 × 54.75 × tan 24.5°
= 12.8 kN/m2

Qs = p f DL
= 0.9425 × 12.8 × 1.5
= 18 kN
From 4.5 m to tip at 18.0 m: limiting
σ’ = 61.5 kN/m2

364 GEE2601/1
From equation 13.8:
f = K σ’ tan δ
= 0,512 × 61.5 × tan 24.50
= 14.35 kN/m2

Qs = p f DL
= 0.9425 × 14.35 × 13.5
= 183 kN

Total Qs = 16 + 18 + 183
= 217 kN

Ultimate pile capacity:


Qu = Qp + Qs
= 346 + 217
= 563 kN

Allowable pile capacity:


Qall = 563/2.8
= 21 kN

Example 13.4

A number of 12 m piles with diameter 300 mm are installed in sand. To determine the
consistency of the sand, standard penetration tests were performed and the averages
for the site were

• 0 -2 m Ncor = 12
• 2–6m Ncor = 18
• 6 – 16 m Ncor = 21

Determine the allowable pile capacity if a safety factor of 3.0 is required.

365 GEE2601/1
Solution

Calculate pile properties:

π 2 π
• Ap = D = 0,30 2 = 0.0707 m2
4 4
• p = πD = π × 0.30 = 0.9435 m

Calculate pile point capacity:

From equation 13.5 we have for pile end capacity


L
• qp = 40Ncor < 400Ncor
D
12
• q p = 40 × 21× = 33600 kN/m2
0,3

But qp is limited to
• q p = 400 N cor = 400 × 21 = 8400 kN/m2

• Qp = qpAp = 8400 x 0,0707 = 593 kN

Calculate the pile shaft capacity. From equation 18.18 (p. 662*) in the prescribed
book, we have, for frictional resistance of low displacement piles: f = Ncor and from
equation 13.2 in the prescribed book:

Qs = p∆Lf

• 0 – 2 m: Qs = 0.9425 × 2.0 × 12 = 22.6 kN


• 2 – 6 m: Qs = 0.9425 × 4.0 × 18 = 67.9 kN
• 6 – 12 m: Qs = 0.9425 × 6.0 × 21 = 118.8 kN

Total:
• Qs = 22.6 + 67.9 + 118.8 = 209 kN

Calculate ultimate and allowable pile capacity:


• Qu = Qp + Qs = 593 + 209 = 802kN Qall = Qu/FS = 802/3 = 267 kN

366 GEE2601/1
13.6 PILE LOAD TESTS

Read section 18.13 (pp. 682-687*) in the prescribed book. If a heavy structure is
supported on piles, each pile is an extremely important element. Should any one pile
fail because of an inadequate design, there is a high probability that neighbouring piles
may also be close to failure for the same reason. These piles may then not be able to
carry the additional load imposed on them by the failure of the first one, and they may
fail as well, with serious consequences to the structure. It was evident from the
discussion of the components of ultimate pile resistance that there is a fair amount of
uncertainty about the values of the different coefficients, like K, α, and β,. The variation
of soil properties with depth, as well as horizontally, adds to the ambiguity, so after
theoretical design, engineers prefer to perform load tests on a number of piles.

Piles are usually tested to about twice their required allowable loads in order to verify
that the working loads will be carried without failure or excessive settlement. It is not
uncommon that additional piles have to be installed when testing reveals inadequate
capacity. Piles installed specifically for the purpose may be tested to failure to get a
better estimate of the values of the coefficients and/or strength of the soil to be used
in the design.

13.7 SELF-ASSESSMENT ACTIVITIES

You now have the opportunity to apply what you have learnt in this unit. Answer the
following questions:

Activity 13.1: Prescribed book exercises

1. Carefully work through worked examples 18.1 to 18.9 (pp. 670-679*).


2. Redo problem 2 with a water table at 6 m and a saturated unit weight of 21.5
kN/m3.
3. Redo problem 3 with a water table at 7 m. The unit weight above and below the
water table is 21 kN/m3.

367 GEE2601/1
Activity 13.2: Typical exam questions
1. Determine the ultimate load capacity of an 800 mm diameter concrete bored pile
for a building project in Mahikeng (North West) as shown in the figure below.

2. As a contractor for a building project in Soweto, you have to construct a concrete


pile which is 20 m in length and 360 mm x 360 mm in cross-section. The pile will
be fully embedded in sand with a unit weight of 16.8 kN/m3 and ϕ=30°. You are
given also Nq∗ = 56.7.
(a) Calculate the ultimate load (Qp), by using Meyerhof’s method.
(b) Determine the frictional resistance (Qs), if k = 1.3 and δ = 0.8ϕ.
(c) Estimate the allowable load carrying capacity of the pile (use FS = 4).

13.8 SOLUTIONS TO SELF-ASSESSMENT ACTIVITIES

Activity 13.2: Typical exam questions

1. Qu = 2 310.94 kN when using λ−method or 1 770.28 kN when using α−method.


2. (a) QL = 212.13 kN; (b) Qs = 1308kN; c) Qal = 380 kN)

368 GEE2601/1
13.9 SUMMARY

After working through the contents of this study unit, you should be able to estimate
the capacity of piles in sand and clay. You should also have the insight to realise that
there is a lot of uncertainty and that a pile testing programme is as much a part of the
pile design operation as the soil survey or laboratory testing programme.

13.10 VIDEO CLIPS, SLIDES AND OTHER WEB-BASED


RESOURCES

For additional explanations on deep foundations, you can watch some video clips.
Web addresses/links to the video clips are available on the module site under
Additional Resources in a folder named Video clips, slides and other web-based
resources.

• Deep foundations, underpinning:


https://www.youtube.com/watch?v=EVUXeeVOZKI
• Pile foundation casing method:
https://www.youtube.com/watch?v=lvFptrZGex0&t=22s
• Deep foundation – introduction:
https://www.youtube.com/watch?v=SZefeLiaiIE
• CP5 6 Deep foundations:
https://www.youtube.com/watch?v=QdosO_pW07s

REFERENCE WORKS AND FURTHER READING

Das, BM. 2010. Principles of geotechnical engineering. 7th ed. Connecticut: Cengage
Learning.

Das, BM. 2013. Fundamentals of geotechnical engineering. 4th ed. Connecticut:


Cengage Learning.

369 GEE2601/1
Das, BM & Sivakugan, N. 2017. Fundamentals of geotechnical engineering. 5th ed.,
international edition. Connecticut: Cengage Learning.

Meyerhof, GG. 1976. Bearing capacity and settlement of pile foundations. Journal of
the Geotechnical Engineering Division, 102(3):197-228.

370 GEE2601/1

You might also like